Bar Prep Final Exam Spring 2021

¡Supera tus tareas y exámenes ahora con Quizwiz!

An environmental activist group vehemently opposed the planned expansion of an oil pipeline that would take it through a wildlife preserve. Despite long periods of protests and media coverage, the oil company was nevertheless granted all of the necessary state and federal permits and permissions to go ahead. Deciding that nonviolent protests were not enough, a splinter faction of the environmentalist group decided to take a more hands-on approach. Late one night when nobody was present, the faction detonated blasting caps on a nearby mountain that caused a small avalanche above the oil company's supply cache. The falling rocks destroyed material and machinery and blocked necessary roads into and out of the wildlife preserve, costing the oil company millions of dollars and months of delay while it had to clear and rebuild many roads. Thereafter, the state legislature enacted a law restricting the rights of members of the environmentalist group in several ways, one of which was excluding members of the group from holding elected office in the state or working for a government agency or contractor. A representative justified the legislation on the grounds that "ours is a peaceful state, and these verminous scum clearly have no place here." Several members of the group who did not participate in the blasting were soon informed that they would no longer be allowed to remain in elected positions, including a city council member and a county school board official. If the members challenge the constitutionality of the legislation in court, what is the likely outcome? (A) The members will prevail because the law is a bill of attainder. (B) The members will prevail because they did not engage in violence as a member of the group. (C) The state will prevail because it acted within its police power. (D) The state will prevail because the group is a subversive organization.

(A) The members will prevail, because the law is a bill of attainder. A bill of attainder punishes named individuals or easily ascertainable members of a group without the benefit of judicial trial. Article I, Section 9, Clause 3 of the U.S. Constitution states: "No bill of attainder...shall be passed"; and Article I, Section 10, Clause 1 provides: "No state shall pass any Bill of Attainder." These apply to states as well as the federal government. The state legislation in this question is a bill of attainder because it amounts to legislative punishment for membership in the environmental activist group. Members of the group, such as those in question, are prohibited from holding elective office or being employed by the government or government contractors without a judicial determination of their right to serve. The legislation is therefore unconstitutional.

A college professor is on trial for the murder of her husband. The prosecution alleges that the professor paid one of her students to kill the victim. The student told his girlfriend the day after the husband's death, "We're rich! I just knocked off the old man and made $100,000 for five seconds' work." The student was a bit too exuberant, however, and a neighbor outside overheard every word. The student mysteriously disappeared before trial. Over the professor's objection, the neighbor testified at trial as to the student's remarks. Why is the neighbor's testimony regarding the student's remarks admissible despite the professor's objection? (A) It is a statement with circumstantial guarantees of trustworthiness. (B) It is a statement by an opposing party. (C) It is a statement against penal interest. (D) It is an excited utterance.

C) It is a statement against penal interest. If a declarant makes a statement against his penal interest and is unavailable for trial, the statement will qualify as a statement against interest and will be admitted as an exception to the hearsay rule. The facts indicate the student has mysteriously disappeared. The student's statement also meets this exception's criteria that it was against the student's penal interest when he said it and that a reasonable person would not have made the statement if it were not true. Therefore, this is the best answer.

The plaintiff owned a very successful seafood restaurant. At the end of the previous season, the plaintiff had become concerned about competition from a new restaurant approximately two miles away. Now, at the beginning of this new season, the plaintiff was particularly concerned, because the clams and scallops he was receiving from his supplier were not up to their usual standard. The plaintiff believed that the diminished quality was due to changes in water temperatures and environmental conditions, which the plaintiff expected to improve by the start of summer. However, even well into the summer season, the plaintiff found that the seafood continued to be subpar. Early in July, he hired a secretary, who was previously employed by the supplier, to work in his office. Shortly after the secretary began working for the plaintiff, she told him that the owner of the new restaurant was an old high school buddy of the president and owner of the supplier. The secretary told the plaintiff that the owner of the supplier was a silent partner in the new restaurant, and that at a meeting in March, the secretary overheard them discussing a plan to supply inferior seafood to the plaintiff's restaurant, in order to hurt the plaintiff's reputation and increase business at the new restaurant. The plaintiff immediately instructed his staff to keep tallies of how many meals containing seafood supplied by the supplier were sent back to the kitchen as a result of customer complaints. In a subsequent action by the plaintiff against the supplier for interference with business relations, the plaintiff moved to introduce the tallies into evidence. Are the tallies admissible? (A) Yes, because they constitute a past recollection recorded. (B) Yes, because they are a business record. (C) No, because they are hearsay not within any exception. (D) No, because there is no proof that the meals were sent back because of the inferior seafood.

C) No, because they are hearsay not within any exception. Hearsay is an out-of-court statement offered into evidence to prove the truth of the matter asserted. Here, the tallies are clearly written testimony being presented for the purpose of proving that the supplier was supplying the plaintiff's restaurant with inferior seafood. Therefore, the tallies are inadmissible hearsay if they are not subject to a recognized hearsay exception. Moreover, because the tallies were clearly created for the purpose of the plaintiff's litigation and not in the ordinary course of the plaintiff's business, they do not come within the business records exception to the hearsay rule.

A baseball fan purchased two tickets for a World Series baseball game. The fan contacted his best friend and invited him to go to the game. The friend, who was also a baseball fanatic, eagerly agreed. The fan told the friend that the game started at 7:00 p.m. and that he would pick him up at about 5:00 p.m. so they could get there early to watch batting practice. They were driving to the game together when the fan sped up to cross an intersection while the traffic signal was changing from amber to red. As he reached the intersection, the fan was traveling at 50 m.p.h., although the posted speed limit was 25 m.p.h. Simultaneously, a car entered the intersection on red and collided with the fan's vehicle. The friend suffered a broken pelvis in the collision. This jurisdiction has adopted the following "modified" comparative negligence statute: "A negligent plaintiff is entitled to obtain a recovery provided plaintiff's negligence is not equal to or greater than that of the defendant's; otherwise no recovery is permitted." Suppose the friend brings suit against the driver of the car that entered the intersection on the red light to recover damages for his injury. A jury returned a special verdict with the following findings: (1) the fan was 55% negligent in speeding; (2) the driver was 45% negligent in driving through the red light; and (3) the friend suffered $100,000 in damages. As a result, the court should enter a judgment in favor of the friend for what amount? (A) $100,000. (B) $55,000. (C) $45,000. (D) Nothing, because the fan was more negligent than the driver.

(A) $100,000. On all standardized examinations (whether it be the MBE, SAT, LSAT, or GRE), the examiner's main goal is to hide the correct answer. The examiners try to accomplish this by employing "distracters" and "red herrings." Here, the comparative negligence statute is the distracter. A comparative negligence statute only applies when there is a negligent plaintiff and one or more negligent defendants. Here, there were two negligent parties (the fan and the driver), but neither of them is the plaintiff. The plaintiff (the friend) was just a passenger who was riding in one of the cars and did nothing negligent. The comparative negligence statute, therefore, is irrelevant. The two negligent parties (the fan and the driver) are joint tortfeasors who can be held jointly and severally liable for the full amount of the plaintiff's damages. This answer choice is correct because the friend may recover 100% of the damages from the driver, who could then seek contribution from the fan.

On New Year's Eve, a man negligently hit a woman with his car. The woman was rushed to the hospital and sent into surgery. The surgeon, who was summoned from a nearby party, operated on the woman while slightly intoxicated. As a result, the surgeon failed to remove a drain tube, causing the woman harm and requiring a second operation to remove. The woman has now brought suit against the man for her injuries. At trial, testimony showed that the woman suffered $100,000 in damages caused to her legs from the accident and $50,000 from complications surrounding the drain tube. At trial, an expert testified that the surgeon's efforts fell below the standard of care that should have been exercised in an operating room as a result of his intoxication. The jury also found that the woman was 25% responsible for the accident. How much should the woman recover from the man? (A) $75,000, or the amount of her injuries from the accident minus her own negligence. (B) $100,000, or the full amount of her injuries from the accident. (C) $125,000, or the woman's injuries from the accident as well as the doctor's negligence, minus her own negligence. (D) $150,000, or the full amount of her injuries for both the accident and the doctor's negligence.

(A) $75,000, or the amount of her injuries from the accident minus her own negligence. While the general rule is that subsequent medical malpractice is considered a foreseeable intervening act which does not sever the original tortfeasor's liability, this does not apply where the medical care was performed recklessly or where medical malpractice was committed deliberately. In such a case, the intervening act is certainly not considered foreseeable. A doctor performing surgery while intoxicated would not considered foreseeable. As such, the doctor's malpractice is a superseding cause which severs the man's liability, making him liable for only the injuries resulting from the accident. Additionally, the majority of jurisdictions have adopted a pure comparative negligence scheme, whereby a plaintiff's recovery will be reduced by her own percentage of fault. Thus, the woman would be able to recover $75,000 from the man, representing her injuries attributable to just the car accident minus her own allocation of fault.

A homeowner said to a carpenter, "My porch is need of repair. I think the boards are cracked. Can you do the job for $5,000?" The carpenter replied, "I'd be happy to, but $5,000 seems a little low. Make it $6,000 and you got a deal." The homeowner then remarked, "Deal." Three days later, the carpenter purchased the necessary materials, then drove his pickup truck to the homeowner's home and unloaded the materials and equipment needed to perform the job. When the homeowner looked out his window and saw what was transpiring, he immediately ran outside and exclaimed, "Stop! The deal's off. I decided to repair the porch myself." In an action by the carpenter against the homeowner for breach of contract, which of the following is the most accurate? (A) A bilateral contract was formed before the carpenter purchased the materials. (B) A unilateral contract was formed when the carpenter started to perform. (C) No contract was formed, because the carpenter did not start to perform. (D) No contract was formed, because the carpenter rejected the homeowner's terms.

(A) A bilateral contract was formed before the carpenter purchased the materials. A bilateral offer, which seeks a return promise rather than the performance of the requested act as acceptance, becomes enforceable as soon as the offeree has promised to perform the act. Thus, this answer choice is correct. The homeowner and carpenter had a valid, enforceable agreement as soon as the two agreed to each other's terms.

A buyer ordered 1,000 widgets from a seller for immediate delivery. The seller responded by shipping 800 widgets rather than the full 1,000 ordered because the seller did not have an adequate inventory and thought it better to ship something rather than nothing. Which of the following is the most accurate statement? (A) A contract was formed when the seller shipped the widgets. The UCC governs contracts for the sale of goods. (B) A contract will be formed once the widgets arrive at the buyer's address. (C) No contract exists, because the seller shipped nonconforming goods. (D) No contract exists, because the seller did not cure the nonconforming tender immediately.

(A) A contract was formed when the seller shipped the widgets. The UCC governs contracts for the sale of goods. Under the UCC, an offer can be accepted by the offeree communicating acceptance or by the prompt shipment of the goods. When the seller ships nonconforming goods, this operates as an acceptance of the offer and a simultaneous breach of contract.

On December 15, a landlord and tenant entered into a lease for a period of one year, beginning at the start of the calendar year, with rent to be paid on the first of each month. One month before the end of the lease term, the tenant called the landlord and the two agreed to renew the lease for another year at the same terms. As of January 1, what type of tenancy do the landlord and tenant have? (A) A term of years. of the lease with no notice required. (B) A periodic tenancy. (C) A tenancy at-will. (D) A tenancy at sufferance.

(A) A term of years. A tenancy for a term of years has a definite beginning and end, and no notice is required to terminate the lease. The initial lease was such a tenancy, terminating at the end of the calendar year with no notice required. When the landlord and tenant agreed to another one-year lease on the same terms, the parties either extended the duration of the lease or created another term of years for the following calendar year. Either way, the lease remained one for a term of years, which will terminate at the end of the lease with no notice required.

A heavyset man with long red hair robbed a liquor store. Thereafter, a man was arrested and charged with armed robbery. At the man's trial, the owner of the liquor store was called to testify. He admitted that he was unable to identify the man, who now had a shaven head, as the robber. The prosecuting attorney then handed the owner six photographs. He proposed to testify, over defense objections, that he had previously told the prosecuting attorney that picture #4, admittedly a picture of the man before he shaved his head, was a picture of the person who robbed his store. How should the court rule on the owner's proffered testimony? (A) Admissible, as a prior identification by the witness. (B) Admissible, as a past recollection recorded. (C) Inadmissible, as hearsay not within any recognized exception. (D) Inadmissible, because it is a violation of the man's right of confrontation.

(A) Admissible, as a prior identification by the witness. FRE 801(d)(1)(C) defines as nonhearsay a prior statement by a witness of identification of a person made after perceiving him, if the person making the identification is available to testify at the trial or hearing and subject to cross-examination concerning the statement. There is no requirement that the witness first be impeached, nor does a prior identification require that the identifying witness make a positive in-court identification. This choice is therefore correct, because the owner had previously identified the man in picture #4.

A defendant was on trial for attempted murder. The alleged victim was called by the prosecution to testify. During her testimony, the victim recounted the incident and described how the defendant savagely beat her with a baseball bat. She was not asked by the prosecution whether she made any statements during the attack. After the victim was excused and left the witness stand, the prosecution called another witness to testify. The witness proposes to testify that when the beating stopped, the victim screamed, "I'm dying. Don't let the defendant get away with this." Upon objection by the defendant's attorney, how will the court rule on the witness's proffered testimony? (A) Admissible, as an excited utterance. (B) Admissible, as a dying declaration. (C) Inadmissible, as hearsay not within any recognized exception. (D) Inadmissible, because the victim was not questioned about the statement before being excused.

(A) Admissible, as an excited utterance. Under FRE 803(2), an excited utterance is defined as a statement relating to a startling event or condition made while the declarant was under the stress of excitement caused by the event or condition. The witness's testimony of the victim's statement, "I'm dying. Don't let the defendant get away with this," related to the savage beating (the startling event) and was made when the beating stopped (while the victim was still under the stress of excitement). The testimony will be admissible substantively as an excited utterance.

After being passed over for a promotion, an aeronautic engineer became a whistleblower. He contacted a government agency to state that the company for which he worked was submitting false safety reports on newly built aircraft. When the company learned that the engineer was leaking this information to the government agency, he was fired from his job. Afterward, the engineer sued the company for wrongful termination of employment. During the discovery stage of litigation, the engineer was deposed by the company's attorney. In his deposition, the engineer stated that the company submitted false safety reports to the government agency to cover up structural defects in its aircraft. A pilot was injured when one of the company's airplanes he was piloting crashed. The pilot asserted a strict product liability tort against the company. At trial, the pilot sought to introduce into evidence portions of the engineer's deposition from his wrongful termination lawsuit against the company. Assume that the engineer is unavailable to testify at trial. Upon objection by the company, how should the trial court judge rule on the engineer's deposition? (A) Admissible, as former testimony. (B) Admissible, as a vicarious statement. (C) Inadmissible, as hearsay not within any recognized exception. (D) Inadmissible, because the company did not have the opportunity to cross-examine the engineer on the liability issue for which the statement is now being offered.

(A) Admissible, as former testimony. Usually, former testimony questions will involve the same parties where each had the opportunity to interrogate the witness at the earlier trial. In this question, we have a plaintiff (who was not party to the first suit) seeking to introduce former testimony against the same defendant. According to Lilly, "using the testimony against the same defendant, which was a party to both suits and had an opportunity at the first trial to interrogate the witness, appears to fit comfortably within the exception." Evidence, p. 286. On the other hand, Lilly points out that if a plaintiff seeks to use all or part of the former testimony against a new defendant (who did not have an opportunity to interrogate the witness), then the testimony should be excluded. Here, because the testimony is being used against the same defendant, it is admissible.

A plaintiff sued a defendant for injuries suffered in a fall on the sidewalk of the defendant's home. The plaintiff's complaint alleged that the walk was covered by a thick sheet of ice, which had been negligently left there for several days by the defendant. In his answer, the defendant set forth that the ice formed overnight and that the plaintiff fell before the defendant had a chance to remove it. During the trial, a physician, whose office was next door to the defendant's home, testified that he saw the plaintiff fall and that the ice, which had been there for several days was at least two inches thick. On cross-examination, counsel for the defendant asked the physician the following question: "During your treatment of the plaintiff on the day in question, is it not true that he told you his fall resulted from a loss of equilibrium after he suffered from dizziness?" Upon objection by the plaintiff's attorney, how should the court rule on the physician's testimony? (A) Admissible, because it is reasonably pertinent to diagnosis or treatment. (B) Admissible, because the physician had personal knowledge of the plaintiff's injuries. (C) Inadmissible, because the defendant's counsel failed to lay a proper foundation. (D) Inadmissible, because the physician's testimony is not relevant to prove that the plaintiff's alleged injuries are false or exaggerated.

(A) Admissible, because it is reasonably pertinent to diagnosis or treatment. The Federal Rules of Evidence allows not only statements of past symptoms and medical history as exceptions under the hearsay rule, but also the cause or source of a patient's past physical condition, insofar as it is reasonably pertinent to diagnosis or treatment. In this regard, the plaintiff's statements to the physician would be admissible as reasonably pertinent to diagnosis and/or treatment regarding the cause or source of his injuries.

A fan attended a minor league hockey game in his hometown. Unfortunately, he was only able to obtain tickets in the visitor's section. While at the game, he became involved in an altercation with a fan of the visiting team. When the fan cheered for a home team goal, the visiting fan turned around and threatened to kill the home fan if he didn't shut up. The home fan pulled a knife out of his pocket and stabbed the visiting fan in the arm. At his trial for aggravated assault, the home fan wants to introduce a statement from a witness who was standing next to the visiting fan at the game. The statement, which the witness had made earlier in the game when the home fan cheered for the home team, was, "You'd better watch out. At a hockey game last week, the visiting fan put two guys in the hospital when they wouldn't shut up. One of them had 33 stitches after the visiting fan bashed his head against the steps." Assume that the witness's statement is being offered as proof of the effect it produced in the home fan's mind. In this regard, how will the statement most likely be found? (A) Admissible, because the testimony is not hearsay as it is not being offered for its truth. (B) Admissible, as a present sense impression. (C) Inadmissible, as hearsay not within any recognized exception. (D) Inadmissible, because the statement is self-serving.

(A) Admissible, because the testimony is not hearsay as it is not being offered for its truth. This answer choice is correct, because the witness's statement that the visiting fan had put two men in the hospital at a hockey game the week before is being offered to show its effect on the defendant's state of mind, rather than being offered to prove the truth of the assertion. Thus, the witness's statement bears upon the reasonableness of the home fan's fear to justify his self-defense plea.

In a written agreement, a landlord leased his home to a tenant for a term of three years, ending on December 31 of the third year, at the rate of $1,000 per month. The lease provided that the tenant could sublet and assign the lease. The tenant lived in the home for one year and paid the rent promptly each month. After one year, the tenant leased the home to a student for one year at a rent of $1,000 per month. The student took possession of the home and lived there for six months, but because of her unemployment, she never made any rent payments. After six months, the student abandoned the home, which then remained vacant for the balance of that year. The tenant retook possession of the home at the beginning of the third and final year of the lease term, but made no further rent payments to the landlord. At the end of the lease term, the landlord brought an action against both the tenant and the student to recover $24,000, the unpaid rent for the second and third years of the lease term. Assuming the landlord is successful, how will the court rule regarding damages? (A) Against the tenant individually for $24,000, and no judgment against the student.A sublease occurs when a tenant transfers to a third person (the subtenant, or sublessee) less than all of his rights, title, and interest in the leased premises. (B) Against the tenant individually for $18,000, and against the student individually for $6,000. (C) Against the tenant individually for $12,000, and against the tenant and student jointly and severally for $12,000. (D) Against the tenant individually for $18,000, and against the tenant and student jointly and severally for $6,000.

(A) Against the tenant individually for $24,000, and no judgment against the student.A sublease occurs when a tenant transfers to a third person (the subtenant, or sublessee) less than all of his rights, title, and interest in the leased premises. A subtenant does not come into privity of estate with the original landlord. Therefore, absent an express assumption of the duty to pay rent by the subtenant, the landlord may not sue the subtenant directly for unpaid rent. Here, the student was a sublessee, because his lease was for only one of the remaining two years of the lease term. As such, the landlord has no cause of action against the student, and must instead recover all of the unpaid rent from the original tenant.

A woman was walking her dog one day around her neighborhood. As she passed a neighbor's house, the neighbor's dog charged out from the side of the property and attacked the woman. A passerby helped get the neighbor's dog off the woman, though she was seriously injured from the attack. The woman later brought a claim against the neighbor for her medical bills. At trial, the neighbor stated that he was diligent in keeping his dog in the backyard, and therefore, someone must have let it out, which was not his fault. The neighbor then filed a motion for judgment as a matter of law. How should the court rule on the neighbor's motion? (A) Denied, because the dog was within the control of the neighbor at the time of the injury. (B) Denied, because the keeper of an animal is strictly liable for injuries that it causes. (C) Granted, because the neighbor testified that he was diligent in keeping his dog in the backyard. (D) Granted, because the woman cannot prove that someone else did not let the neighbor's dog out of the backyard.

(A) Denied, because the dog was within the control of the neighbor at the time of the injury. Even though there is no proof that the neighbor did not personally let the dog out, he is nevertheless responsible for keeping the dog safely confined. According to the doctrine of res ipsa loquitur, the incident could not have happened without negligence, and so the elements of duty and breach will be established.

A plaintiff brought an action in federal court for assault and battery against a diverse defendant claiming $15,000 in actual damages, $12,500 in pain and suffering, $12,500 in lost wages, and $36,000 in punitive damages. The defendant made a motion to remand the case to state court. How should the court proceed? (A) Deny the request, because the amount in controversy is satisfied. (B) Deny the request, because a plaintiff can aggregate multiple claims against a single defendant. (C) Grant the request, because punitive damages cannot be used towards the amount in controversy for a case arising out of diversity. (D) Grant the request, because although punitive damages are counted towards the amount in controversy, they cannot account for the majority of the total amount.

(A) Deny the request, because the amount in controversy is satisfied. Punitive damages can be used in calculating the amount in controversy provided that they are available for the particular claim being asserted. As punitive damages are allowed in most tort cases, this answer choice is correct.

One night, two teenagers, ages 17 and 18, were walking home from their jobs at a pizza shop. As they passed a large house, the older boy told the younger one that as a kid, he had worked in a hotel for the family that owned both that house and the hotel in which he worked. He also told the younger boy that when he had worked at the hotel, rumor had it that the hotel owner skimmed most of the cash off the top of his businesses, and that he had accumulated more than $500,000 in cash, which he kept hidden in the basement of his home. The younger boy grew extremely excited at the thought of all that cash there for the taking. They spontaneously decided to sneak into the house and steal the money. They jumped the fence surrounding the property and crept through the backyard of the house. They were attempting to jimmy the downstairs window when they heard the owner yelling, "Hey, who's there?" Frightened, the boys began running away, chased by the incensed homeowner. However, before he could catch up to them, the homeowner suffered a heart attack from the excitement and extreme physical exertion and died. If the teenagers are charged with homicide, they will likely be found guilty of what crime? (A) Felony murder. (B) Involuntary manslaughter. (C) First-degree murder. (D) No crime, because they intended no death.

(A) Felony murder. Felony murder is an intentional or accidental killing proximately caused during the commission or attempted commission of a serious or inherently dangerous felony, including burglary, arson, robbery, rape, and kidnapping (known through the mnemonic "BARRK"). The resulting death must be a foreseeable consequence of the defendant's actions. Courts have generally been very liberal in applying the foreseeability requirement, and for purposes of felony murder most deaths are considered foreseeable. Under the common law, the crime of burglary consisted of the breaking and entering of the dwelling house of another at nighttime with the intent to commit a felony therein. This definition has since been expanded to include entry into the curtilage, structure, or area immediately surrounding the dwelling. Here, as the teenagers jumped the fence of the homeowner's property and attempted to jimmy the window with the intent to commit larceny therein, they committed burglary. Because it is foreseeable that a burglary victim may chase after the burglars, it is likely that the homeowner's death would be considered a foreseeable outcome of the teenagers' act. As such, both boys will likely be found guilty of felony murder.

A man who had recently lost his job decided to seek revenge by committing a fake robbery at his former place of employment. The defendant entered the sporting goods store, picked up a ski mask and replica gun, and approached the clerk. The man pointed the gun at the clerk and demanded the money from the register. Terrified, the clerk closed the register and ran into the back room. The man panicked, ran back to his car, and drove off. As he was approaching a traffic light, he hit the gas and sped through the intersection, causing a truck to swerve out of the way and run over a pedestrian standing on the sidewalk. The pedestrian was rushed to the hospital, but died in surgery as a result of his injuries. This jurisdiction criminalizes speeding in an intersection. Which of the following is the most serious crime for which the man can be convicted? (A) Felony murder. (B) Involuntary manslaughter, because no robbery occurred. (C) Misdemeanor manslaughter, because speeding in an intersection is a crime. (D) Robbery, because the truck swerving into the pedestrian was a superseding cause.

(A) Felony murder. The pedestrian's death was unintentional, but did occur as a foreseeable result during the commission of the robbery (as the man had yet to reach a place of safety, terminating the felony). It is immaterial if the man's car actually hit the pedestrian or if the truck hit the pedestrian, because the man will still be held criminally responsible for the death.

The defendant and the victim have been rivals for years. Their rivalry began when they were teenagers and competed against each other on rival sports teams at schools on opposite sides of the town. Later, the victim broke up the defendant's marriage by having an affair with the defendant's wife. The defendant and the victim are now in rival motorcycle gangs, and each is trying to control the local drug trade. The defendant just learned that the victim has been dating his little sister, and she is now addicted to drugs. To make matters worse, the defendant also learned that after the victim gives the defendant's sister drugs, he passes her around to members of his gang so that they can "do whatever they want with her." The defendant is completely enraged by the victim's conduct. He calls the victim and asks if they can meet for a few beers to discuss a merger of their respective drug trade organizations. The victim agrees, so they meet and talk for an hour or so. Later, the defendant offers the victim a ride home. When they pull up in front of the victim's house, the defendant shoots the victim in the back of the head, stating, "That's for my little sister." The victim dies instantly. What is the most serious crime of which the defendant can be found guilty? (A) First-degree murder. (B) Second-degree murder. (C) Voluntary manslaughter. (D) Involuntary manslaughter.

(A) First-degree murder. First-degree murder includes intent-to-kill murder committed with premeditation and deliberation, felony murder, and in some jurisdictions, murder accomplished by lying in wait, poison, terrorism, or torture. If a murderer does any reflection or premeditation, even if the reflection is cursory and brief, he may be guilty of first-degree murder. An intentional killing may be mitigated to voluntary manslaughter (also known as a heat-of-passion killing) if the killing was mitigated by adequate provocation or other circumstances negating malice aforethought. The time period between the heat of passion and the fatal act must be short enough that a reasonable person would not have had time to cool off. Here, although the defendant was provoked by the victim's harmful acts to the defendant's sister, there was a significant cooling off period between the time the defendant learned of the victim's actions and the time he killed the victim. Moreover, the facts indicate that the murder was premeditated, as the defendant planned to take the victim out for drinks on the pretense of discussing business then drive him home and shoot him. Therefore, it is unlikely that the defendant's act will be mitigated to voluntary manslaughter, and the defendant will most likely be found guilty of first-degree murder.

A landlord leased an office building to a tenant for 10 years. The tenant always paid his rent on time. After four years, the tenant assigned his interest in the building to medical practice. The medical practice paid the rent on time for the first six months, but then stopped paying the rent. After being in the office building for a year, the medical practice decided to downsize and assigned the lease to a publisher. The publisher never paid any rent. One year later, the landlord sued the tenant, medical practice, and publisher for the accrued unpaid rent. From whom should the landlord recover damages? (A) From the tenant or the medical practice for six months' rent, and from the tenant or the publisher for one year's rent. (B) From the tenant or the medical practice for six months' rent, and from the tenant, medical practice, or publisher for one year's rent. (C) Only from the tenant for six months' rent, and only from the publisher for one year's rent (D) Only from the tenant, for all of the unpaid rent.

(A) From the tenant or the medical practice for six months' rent, and from the tenant or the publisher for one year's rent. Where a tenant assigns all of his leasehold interest to an assignee, such assignment does not release the original tenant from his contract obligations to the landlord under the terms of the lease absent a novation, which is a separate agreement discharging a contractual party from liability. This is true even when the assignee subsequently reassigns the lease to a new assignee. As such, the tenant will be liable for any unpaid rent. However, the medical practice will only be liable for the unpaid rent that accrued during its one year of possession. When a lease is assigned to a new tenant, the assignee comes into privity of estate with the landlord, but not privity of contract. If the assignee then reassigns the property to someone else, the middle tenant is no longer in privity with the landlord at all, and is therefore not liable for any unpaid rent of the subsequent assignee unless the assignor expressly assumed the duty to pay rent. Therefore, the medical practice will not be liable for the publisher's unpaid rent; only the original tenant and the publisher will be liable for this amount.

An elderly couple, through a Washington State travel agent, purchased passage for a seven-day cruise. The couple paid the fare to the agent, who forwarded the payment to the cruise line's headquarters in Miami, Florida, where the cruise line was incorporated and conducted all of its business operations. The cruise line then prepared and sent the tickets to the couple. The back of the tickets stated, "It is agreed by the passenger and carrier that all disputes in connection with this contract shall be litigated before a court in the State of Florida." The couple boarded the ship in Los Angeles, California. While the ship was in international waters off the Mexican coast, the couple was injured when a faulty light fixture fell on their bed. The couple sued the cruise line in Washington, claiming that their injuries were caused by negligence. The cruise line moved to dismiss, arguing it was not subject to the Washington court's jurisdiction. How should the court rule? (A) Grant the motion, based on the forum selection clause. (B) Grant the motion, because the suit was based on a contract formed in Florida. (C) Grant the motion, because the cruise line lacked sufficient contacts with Washington. (D) Deny the motion.

(A) Grant the motion, based on the forum selection clause. If a contract is otherwise valid and enforceable, a person may consent to personal jurisdiction within a state. Here, the contract made between the couple and the cruise line stated, "All disputes in connection with this contract shall be litigated before a court in the State of Florida." Consequently, Florida, not Washington, had jurisdiction over this claim [Carnival Cruise Lines Inc. v. Shute, 499 U.S. 585 (1991)].

A woman was driving her van along a public road one night. A police officer who was driving behind the woman, decided to make a random stop of the woman's vehicle to check her license and registration. The officer pulled the woman's van over to the side of the road and then walked up to the driver's side of the vehicle. When he came alongside the driver's window, the officer asked the woman for her identification. As the woman was thumbing through her wallet, the officer shone his flashlight into the van and spotted a plastic bag containing marijuana lying on the floor under the back seat. The officer then arrested the woman and charged her with possession of marijuana. At the woman's trial for illegal possession of a controlled substance, her attorney moved to suppress the use of the marijuana as evidence. How should the judge rule on her motion? (A) Granted, because the marijuana was the fruit of an illegal search. (B) Granted, because the police officer did not have probable cause or a reasonable suspicion to believe that the woman's van contained a controlled substance. (C) Denied, because the marijuana was in plain view when the police officer shone his flashlight inside the van. (D) Denied, because the marijuana was in an automobile.

(A) Granted, because the marijuana was the fruit of an illegal search. A random stop of a vehicle on the highway where the officer has no suspicion of wrongdoing is unconstitutional because it leaves too much discretion to the police officer [Delaware v. Prouse, 440 U.S. 648 (1979)]. And, applying the fruits of the poisonous tree doctrine, no evidence seized as a result of a Fourth Amendment violation may be admitted at trial [Wong Sun v. United States, 371 U.S. 471 (1963)]. Note: The rule suppressing fruits of an illegal search applies not only to objects found, but also to verbal statements obtained because of the original tainted search or as a result of an illegal arrest [Brown v. Illinois, 422 U.S. 590 (1975)].

A man took two people hostage and arranged for them to play a game. The man blindfold the first hostage and strapped a gun, which was pointed at the first hostage's head, to the second hostage's hand. The man told the second hostage that if he did not pull the trigger, that the man would shoot the second hostage. Not wanting to die, the second hostage pulled the trigger, killing the first hostage. If the second hostage is charged with the death of the first hostage, how should the jury find the second hostage? (A) Guilty of murder. (B) Guilty of manslaughter. (C) Not guilty, because the second hostage was under duress. (D) Not guilty, because the killing was done out of necessity

(A) Guilty of murder. The 2nd hostage intentionally pulled the trigger knowing that the gun was pointed at the first hostage's head. He chose to kill rather than to be killed, and he would therefore be guilty of murder. Note that duress is a defense to everything except murder, because one person's life is not more important than anyone else's.

A man recently moved to a new town and fell in love with its charm. A month later, deciding to get as involved as possible, the man submitted his name for an upcoming election for city council. However, the city refused to put his name on the ballot, citing a statute requiring residents to have lived within the town boundaries for at least 60 days prior to being considered for candidacy. The man sued the city, claiming that the statute violated the Due Process Clause. How should the court rule? (A) In favor of the city, because under a rational basis analysis, the statute is rationally related to a legitimate state interest. (B) In favor of the city, because under a strict scrutiny analysis, the statute is necessary to further a compelling state interest. (C) In favor of the man, because under a rational basis analysis, the 60-day residency requirement is unconstitutional. (D) In favor of the man, because under a strict scrutiny analysis, the 60-day residency requirement is unconstitutional.

(A) In favor of the city, because under a rational basis analysis, the statute is rationally related to a legitimate state interest. While the right to vote is typically considered a fundamental right subject to strict scrutiny, government regulations of ballot access by candidates based on age, duration of residency, or payment of filing fees require only a minimum rational basis scrutiny. Because the statute in question here is setting a durational residency requirement on a person's being able to get their name on the ballot as a candidate, it will be subject to rational basis, not strict scrutiny.

A state held an election in which there were many charges of voter fraud and vote manipulation. Concerned about voting fraud, the state legislature passed a bill providing that any resident of the state who wished to vote in a state election could not vote until he or she had lived within the state for at least one year. The governor of the state signed the bill into law. A group of new residents who wished to vote in a state election but were prohibited under the new law brought suit to enjoin the enforcement of the statute. How should the court rule? (A) In favor of the plaintiffs, because under a strict scrutiny analysis, a one-year residency requirement is unreasonable. (B) In favor of the plaintiffs, because under a rational basis analysis, a one-year residency requirement is unreasonable. (C) In favor of the state, because under a strict scrutiny analysis, the statute is necessary to further a compelling state interest, namely to prevent voter fraud. (D) In favor of the state, because under a rational basis analysis, the statute is rationally related to legitimate state interest, namely to prevent voter fraud.

(A) In favor of the plaintiffs, because under a strict scrutiny analysis, a one-year residency requirement is unreasonable. Durational residency requirements should receive strict scrutiny analysis. Here, a residency requirement of one year will not pass strict scrutiny. Therefore, the statute will be found unconstitutional.

A state provided for the free distribution of textbooks to all public and private school students, certified teachers, and offered vouchers and tuition grants for families wishing to send their children to private schools. One private school that offers elementary and secondary education in the state denies admission to all children of Canadian descent. Two children whose family originated from Toronto, and who had lived in the state for several generations, were denied admission to the private school. The parents subsequently filed an action against the state challenging the distribution of vouchers and tuition grants to the school was unconstitutional. What of the following is the strongest argument in favor of finding the challenged public aid unconstitutional? (A) Issuing tuition grants to this private school furthers segregation. (B) It is unconstitutional for a state to provide aid to private schools. (C) The Constitution forbids private, as well as public, bias in education. (D) Tuition grants for private schools do not serve any legitimate educational function.

(A) Issuing tuition grants to this private school furthers segregation. SCOTUS has held that public aid given to private, segregated schools--including the free distribution of textbooks, exclusive use of public facilities, or the issuance of tuition grants--is unconstitutional [Norwood v. Harrison, 413 U.S. 455 (1973)]. The majority opinion stated that a state cannot give significant aid to schools that practice racial or other invidious discrimination.

On October 1, a seller mailed a letter to a buyer offering to sell a specified quantity of shirts at list price. The buyer received the seller's offer on October 2. The next day, the buyer mailed the seller a letter of rejection. The buyer then changed his mind and decided to accept the seller's offer. On October 4, the seller sent the buyer a letter revoking his original October 1 offer. On October 5, the buyer emailed the seller indicating that he wished to accept the seller's offer. The seller read the email on October 6. On October 7, the buyer received the seller's letter of revocation. The following day, the seller received the buyer's rejection. The seller subsequently refused to sell the shirts to the buyer, and the buyer sued for breach of contract. Assume that the buyer's email complies with the Statute of Frauds. Which of the following is most accurate? (A) Judgment for the buyer, because he accepted in a reasonable manner and before receiving notice of the seller's revocation. (B) Judgment for the buyer, because his acceptance was effective upon dispatch. (C) Judgment for the seller, because he revoked his offer before receiving the buyer's acceptance. You Answered (D) Judgment for the seller, because the buyer did not accept in a proper manner.

(A) Judgment for the buyer, because he accepted in a reasonable manner and before receiving notice of the seller's revocation. This is a classic case of the mailbox rule, but with a twist. Because the buyer sent a rejection prior to sending his acceptance, the mailbox rule will not apply, and so the acceptance will not be effective upon dispatch. Instead, the first communication to reach the seller will control. Therefore, as soon as the seller read the email from the buyer accepting the offer, the buyer's acceptance was effective.

A woman was employed as a state trooper. Although the state provides both sexes with equal pay and benefits, the state has adopted a policy that prohibits the assignment of female officers to its special undercover narcotics division. This is a moderate risk position that sometimes involves violent encounters with drug dealers. Since the special narcotics division was first established, five undercover agents have been killed in the line of duty. It is because of the state's concern with the safety and well-being of its female officers that it has adopted such a policy. The woman, who desired to be a member of the narcotics division, filed an application for assignment as a special drug agent. After she was rejected for the position, the woman sued the state in federal court to enjoin enforcement of its stated policy on the grounds that it is unconstitutional. As a matter of constitutional law, which of the following results in this suit is most appropriate? (A) Judgment for the woman, because the facts asserted do not demonstrate that the particular classification contained in this policy is substantially related to the advancement of important state interest. (B) Judgment for the woman, because the terms and conditions of state government employees are privileges or immunities of state citizenship that may not be abridged by the state on the basis of gender. (C) Judgment for the state, because it is within a state's police power to insulate the terms and conditions of governmental employment. (D) Judgment for the state, because the state has articulated a rational basis for this classification and, therefore, a court may not substitute its judgment for that of responsible state officials.

(A) Judgment for the woman, because the facts asserted do not demonstrate that the particular classification contained in this policy is substantially related to the advancement of an important state interest. Gender classifications are subject to a middle-tier standard of review. The burden of persuasion is on the government to demonstrate that the classification (or discrimination) is substantially related to an important governmental interest. This answer choice correctly states the applicable standard of review for gender classifications.

One evening, the defendant was playing horseshoes with his son in the front yard of his house. One of the horseshoes went too far and landed on the neighbor's lawn, breaking the neighbor's bird bath. Immediately after hearing the crash, the neighbor stepped outside and began yelling about the bird bath. The neighbor said that the defendant's son was going to be a lousy sport, just like his dad. Infuriated, the defendant hurled the horseshoe in his hand at the neighbor's head. The horseshoe hit the neighbor in the temple, killing him instantly. Of what crime can the defendant be found guilty? (A) Murder, because the defendant's actions were malicious. (B) Murder, because the defendant was the initial aggressor. (C) Voluntary manslaughter, because the neighbor's insults caused the defendant to react violently. (D) Involuntary manslaughter, because the defendant did not intend to kill his neighbor.

(A) Murder, because the defendant's actions were malicious. All forms of murder require malice, such as intent to inflict serious bodily injury, wanton and willful misconduct, or depraved-heart murder. Here, there is no indication that the defendant intended to kill his neighbor. While his neighbor died as a result of the defendant's purposeful act, the act of throwing a horseshoe at someone could be construed as an attempt to kill, or even as an attempt to inflict serious bodily injury. As such, the defendant could be convicted of murder.

An off-duty police officer came home early from work one day. He walked up to his bedroom and, upon opening the door, saw his wife and another man in bed together. Visibly upset, the husband grabbed his service revolver and shot the man to death. He then went to a local bar, had several drinks, returned home, and killed his wife. The husband is guilty for which of the following crimes? (A) Murder of both his wife and the other man. (B) Voluntary manslaughter of both his wife and the other man. (C) Murder of the other man, and voluntary manslaughter of his wife. (D) Murder of his wife and voluntary manslaughter of the other man.

(D) Murder of his wife and voluntary manslaughter of the other man. The husband killed the other man while in the heat of passion. There was adequate provocation, which will reduce this intentional killing from murder to manslaughter. However, the man did not initially kill his wife while under this provocation. Instead, he went to a bar, had several drinks, and then returned home to kill his wife. A reasonable person would have cooled off in this period. Therefore, the murder of the wife will not be reduced to manslaughter.

A man owned a home security business. In addition to selling personal defense devices, he also trained dogs to attack strangers at night. He sold and leased his guard dogs to various business and factory owners, who used the guard dogs to frighten away intruders from entering their premises at night. One evening, the man was in the backyard of his home training three of his guard dogs. The backyard was enclosed with a chain-link fence and a latched gate that prevented the dogs from running out. After the training session, the man opened the gate and permitted the dogs to run loose in his front yard. Minutes later, a bicyclist got a flat time in from of the man's home and was attacked by several of the dogs. The bicyclist suffered severe injuries and died as a result of the attack. At trial, it was conceded that the man's dogs attacked a postal worker two weeks prior to this incident. For what crime should the man should be found guilty? (A) Murder, because the man's actions were malicious. (B) Involuntary manslaughter, because the man did not command the dogs to attack the bicyclist. (C) Voluntary manslaughter, because the dogs could have reasonably believed the bicyclist was an intruder. (D) Reckless endangerment, because the dogs killing the victim was a superseding cause.

(A) Murder, because the man's actions were malicious. Depraved-heart murder occurs when a defendant's conduct creates an unjustifiable, very high degree of risk of death or serious bodily injury to another, even though unaccompanied by any intent to kill or to do serious bodily injury, and such conduct actually causes the death of another. Because the man had trained his guard dogs to attack at night and then opened the gate to let them run loose, he will be criminally responsible for the death of the bicyclist. This is so especially in light of the fact that he was aware that these dogs had previously attacked postman postal worker just two weeks prior to this incident.

A night watchman became annoyed at his next-door neighbor who was remodeling his house. The noise from the jackhammering was so loud that the night watchman could not fall asleep. The nightwatchman knocked on the door, asked the neighbor to keep it down, and the neighbor simply replied, "Sucks to be you!" Angered, the night watchman went home and got his service revolver. Not intending to shoot anyone, he fired a bullet through the open first-floor window at the jackhammer, as he merely wanted to put an end to the noise. The bullet, however, ricocheted off the jackhammer and struck a construction worker in the head, killing him. The jurisdiction makes it a misdemeanor to discharge a firearm in public. For what crime should the night watchman be found guilty? (A) Murder. (B) Involuntary manslaughter. (C) Voluntary manslaughter. (D) Discharge of a firearm in public.

(A) Murder. The night watchman is guilty of depraved-heart murder. Firing a bullet into a room that the defendant knows is occupied by several people constitutes the kind of reckless indifference to human life that will make a criminal defendant liable for murder, even though the killing was unintentional. Therefore, this answer is correct.

A defendant had an argument with a patron in a bar over a game of pool. As they were quarreling, the defendant pulled out his knife, intending only to frighten the patron. The knife was six inches long, and the defendant held it out towards the patron. The patron immediately retreated from the pool table, telling the defendant that he was a hemophiliac and that he did not want any trouble. Wanting to have some fun at the patron's expense, the defendant drew near and swung the knife in the patron's direction. The defendant got a little too close to the patron and unintentionally nicked the patron's arm. The bar was far from a hospital, and by the time the patron arrived at the emergency room, he had died from his injury. What is the most serious crime for which the defendant could be convicted? (A) Murder. (B) Voluntary manslaughter. (C) Involuntary manslaughter. (D) Battery.

(A) Murder. While the defendant only intended to frighten the patron (which would be an assault) and did not intend to kill him or to inflict serious bodily harm, there is malice, because he was aware that the knife could cut the patron, who was a hemophiliac. As such, the defendant could be convicted of murder.

A breeder and owner of vicious guard dogs trained his dogs to attack strangers at night. He often sold and leased his guard dogs to various business and factory owners who used the guard dogs to frighten away intruders from entering their premises at night. One evening, the breeder was in the back yard of his home training three of his guard dogs. The backyard was enclosed with a chain-link fence and a latched gate that prevented the dogs from running out. After the training session, the breeder opened the gate and permitted the dogs to run loose in his front yard. Minutes later, a man was walking along the sidewalk in front of the breeder's house when he was attacked by one of the dogs. The man suffered severe injuries and died as a result of the attack. The breeder should be found guilty of what crime? (A) Murder. (B) Involuntary manslaughter. (C) Voluntary manslaughter. (D) Reckless endangerment.

(A) Murder. According to LaFave, "extremely negligent conduct, which creates what a reasonable man would realize to be not only an unjustifiable but also a very high degree of risk of death or serious bodily injury to another though unaccompanied by any intent to kill or do serious bodily injury and which actually causes the death of another" constitutes depraved-heart murder. See Criminal Law, p. 541. Since the breeder had trained his guard dogs to attack at night and then opened the gate to let them run loose, he will be criminally responsible for the killing of the man. Such conduct on the breeder's part is more extreme than the gross or criminal negligence standard sufficient for involuntary manslaughter. The breeder will be liable on the theory of depraved-heart murder.

A defendant wished to see his high school basketball team win the state championship. During an important game, the defendant pulled out a gun and shot at the leg of a key player on the opposing team. The defendant intended only to inflict a slight wound so that the opposing player would be unable to complete the game. When the defendant fired the shot, he unintentionally hit a player on his own high school team in the chest, killing him instantly. What is the most serious crime of which the defendant can be convicted? (A) Murder. (B) Voluntary manslaughter. (C) Involuntary manslaughter. (D) Battery.

(A) Murder. At common law, murder is the unlawful killing of another human being with malice aforethought. Malice is found where there was intent to kill, intent to inflict serious bodily harm, reckless indifference to human life, or a death caused during the commission of a dangerous felony. Although the defendant may have intended only to cause a slight wound, his actions did, in fact, cause the death of another human being. Such a killing is murder unless justified, excused, or mitigated. The fact that the defendant shot the "wrong" basketball player would not change the outcome, because he was aware (or should have been aware) that the shooting created a serious risk of death or grave bodily harm. Therefore, the malice is present, and the defendant can be convicted of murder, since there is no mitigation, justification, or excuse.

A defendant hated his boss, who had recently demoted him to a less prestigious position. Late one afternoon, the defendant saw his boss walking down the hallway. The defendant pulled out a gun and fired four shots at his boss. Although none of the bullets directly hit his boss, one of the shots ricocheted against a wall and struck the boss in the head, killing him instantly. What is the most serious crime of which the defendant can be convicted? (A) Murder. (B) Voluntary manslaughter. (C) Involuntary manslaughter. (D) Battery.

(A) Murder. Here, the facts do not state what the intent of the defendant was in this situation. He hated his boss, had recently been demoted, and was armed while at work, but the facts are not clear that the defendant actually wanted to kill the boss, as opposed to just wound. However, under the deadly weapons doctrine, an inference of intent to kill is raised through the intentional use of any instrument which, judging from its manner of use, is calculated to produce death or serious bodily injury. Because the defendant hated his boss and fired four shots at him, bad aim notwithstanding, an inference of intent to kill can be drawn here. An argument could also be made for depraved-heart murder or even intent to inflict serious bodily injury, but sufficient facts exist to satisfy inferring an intent to kill.

The plaintiff, a citizen of State A, went on a business trip to State B and stayed in a hotel. While there, he received a chemical burn while using the hot tub. He filed a tort suit in State A state court against the hotel and the technician (an independent contractor) who serviced the hot tub and pool. The man alleged a violation of State B law and sought $100,000 in damages. The hotel and the technician are citizens of State B for the purposes of diversity jurisdiction. Two weeks after the hotel and technician were served with process, the hotel's lawyer sought to remove the case to federal court, but the technician did not wish to remove the action. Two months later, the plaintiff learned that the technician followed the maintenance instructions as provided by the manufacturer of the hot tub. The plaintiff then joined the manufacturer of the hot tub as a defendant. Several weeks after being served, the manufacturer, a resident of State A, wished to remove the case to federal court and persuaded the technician and hotel to join in a notice of removal. The manufacturer filed a notice of removal exactly four weeks after being served, and the hotel and technician filed a notice that same day joining in the removal. May the suit be removed under these circumstances? (A) No, because a case can only be removed that could have originally been filed in federal court. (B) No, because the defendants waited too long to initiate their removal. (C) Yes, because the defendants have the right to remove the action within 30 days after a reason for removal become apparent. (D) Yes, because the plaintiff added the manufacturer to prevent the case from being removed.

(A) No, because a case can only be removed that could have originally been filed in federal court. Removal is proper only when the plaintiff could have originally brought the case in federal court. Here, this case involves a sufficient amount in controversy, but because the plaintiff and the manufacturer are from the same state, there is not complete diversity.

A Hawaiian plaintiff sued a California defendant for personal injuries which resulted in $100,000 in damages after the defendant negligently pushed her down some stairs while in California. The plaintiff argued that the defendant was subject to personal jurisdiction in Hawaii because the defendant took the bar exam in Hawaii several years earlier, and spent two of his last three family vacations in Hawaii. The defendant argued that the sum total of his time in Hawaii was to take the bar (in which he immediately went inactive and only paid yearly dues), to vacation on those two occasions (which were paid for by his company, as he was awarded attorney of the year both of those years), and to rent a car while he was on vacation. Is the defendant subject to personal jurisdiction in Hawaii? (A) No, because although the defendant had several contacts, even in the aggregate, they are insufficient to establish personal jurisdiction over him. (B) No, because the defendant has shown his intent not to remain in Hawaii and has only entered at limited times and with a limited purpose each time. (C) Yes, because the defendant, by way of his taking the bar exam, has purposely availed himself of the jurisdiction. (D) Yes, because entering a contract with a hotel for lodging and with a rental car company is implied consent which subjects him to personal jurisdiction.

(A) No, because although the defendant had several contacts, even in the aggregate, they are insufficient to establish personal jurisdiction over him. Although the defendant had several contacts with the forum state of Hawaii, the claim must be related to that contact. For jurisdiction to be constitutional under a "minimum contacts" analysis, three conditions must be satisfied: 1) the defendant must have established a minimum contact with the forum state; 2) the claim against the defendant must be related to that contact; and 3) the exercise of jurisdiction must not offend traditional notions if fair play and substantial justice. In this case, the plaintiff's claim is against the California defendant for pushing the plaintiff down the stairs while in California. Under minimum contacts analysis, the claim against the defendant must be related to that contact, which is not the case here.

A State A plaintiff sued a State C defendant in federal court in State A for personal injuries that occurred in State B which caused an alleged $100,000 in damages. The plaintiff argued that the defendant was subject to personal jurisdiction in State A because the defendant took the bar exam in State A several years earlier, and took several vacations to State A in the last 10 years. Additionally, on the latest vacation, the defendant rented a car. The defendant argued that the sum total of his time in State A was to take the bar (which he failed and never took again) and to vacation on two occasions. Is the man subject to personal jurisdiction in State A? (A) No, because although the man had several contacts, even in the aggregate, they are insufficient to establish personal jurisdiction over him. (B) No, because the man has shown his intent not to remain in State A and has only entered at limited times and with a limited purpose each time.An intent to remain is not required. (C) Yes, because renting a car is implied consent which subjects him to personal jurisdiction. (D) Yes, because the man, by way of his taking the bar exam, has purposely availed himself of the jurisdiction.

(A) No, because although the man had several contacts, even in the aggregate, they are insufficient to establish personal jurisdiction over him. While the number of times entered and purpose of each trip are useful in balancing the sufficient nature of contacts, it is much easier to look at the intentionality surrounding taking the bar exam. It is also important to note that the contacts are unrelated to the lawsuit. The defendant could actually have many, many more contacts with the state, but if they are unrelated to the lawsuit, they would still not be enough, because general jurisdiction is only available for natural persons who are residents of the state, or non-natural persons who are "essentially at home" in the state. While the man did take the bar exam in the state, he did not pass and never retook the exam. As such, this contact, even with the later vacations added to it, would not be sufficient for general personal jurisdiction.

After an airplane crash at a small airfield, an inspector for the Federal Aviation Administration investigated the crash. In her initial visit, the investigator took pictures of the scene and made some notes after conducting interviews. One particular interview was with a novice pilot who happened to be at the airfield when the crash occurred. He recalled to the investigator seeing something wrong with the left wing of the crashed plane before it took off, but that he didn't have a chance to warn the pilot. He went on to say that "even a novice like him would have done a quick safety check and spotted the issue before taking off." The investigator then compiled her findings and submitted them in a report. Her conclusion was that the pilot failed to conduct himself as a reasonably prudent pilot. A ground worker at the airfield who had been burned trying to rescue the pilot brought a civil case against the pilot's estate. The ground worker wanted to introduce the novice pilot's statement from the investigator's report. The estate's attorney objected. Is the novice pilot's statement admissible? (A) No, because it is hearsay not falling within any exception. (B) No, because the estate does not have an opportunity to cross-examine the investigator or the novice pilot. (C) Yes, as a business record, because the FAA routinely conducts such investigations. (D) Yes, because it is contained inside of a public record compiled by a government agent under duty of law.

(A) No, because it is hearsay not falling within any exception. Rule 803(8) creates a hearsay exception for most public records and reports. Properly certified official records from public offices are generally admissible if they are routine, factual, based on personal knowledge of public officials, and appear reasonably reliable. Investigative reports, reports with recommendations, and one-time reports prepared for a narrow purpose are generally admissible except against a defendant in a criminal case. No witness is required if the document is certified; it will be considered self-authenticated under Rule 902. However, statements by private citizens may not be included in official records unless they separately fall within a hearsay exception [D.W.S. v. L.D.S., 654 N.E.2d 1170 (Ind. Ct. App. 1995) (report from welfare department based on interviews of people who were under no duty to report did not qualify; person making report must be public employee with personal knowledge)]. Here, because the statement of the novice pilot is not independently admissible, it will not be admitted simply because it was written in a public record.

A state enacted a statute that required that 50% of the employees of any business that was located in the state and that employed more than five people be residents of the state. A man worked for a department store in the state as a shoe salesman and commuted the five miles to his job from a neighboring state. The department store had 10 employees, eight of whom lived outside of the state. The man was informed by his supervisor that he was one of the three nonresident employees who would be fired pursuant to the new statute. The man properly filed an action challenging the ordinance on constitutional grounds. Is the state statute constitutional? (A) No, because it unfairly discriminates in favor of residents of the state against nonresidents as to the basic right to employment. (B) No, because it authorizes employers to arbitrarily dismiss some nonresidents while retaining others. (C) Yes, because a state is permitted to treat nonresidents differently from residents whenever it sees fit. (D) Yes, because the man was discharged in compliance with the ordinance and was not otherwise discriminated against by his employer.

(A) No, because it unfairly discriminates in favor of residents of the state against nonresidents as to the basic right to employment. The state is discriminating against nonresidents with respect to one of the privileges and immunities of citizenship protected by Article IV, Section 2--the right to pursue a livelihood. This clause protects nonresidents from being discriminated against by state or local governments with respect to certain basic rights or essential activities of citizenship. Among these are the right to own property, equal access to medical care and the courts, and the right to earn a living. Here, the state statute discriminates against nonresidents. When discrimination against the employment rights of nonresidents is found to exist, the law will be struck down unless the responsible state or local government can show such discrimination is proper because the nonresidents are a peculiar source of the evil the law was designed to redress. For instance, if the state were experiencing high unemployment because nonresidents were taking all the jobs, a court could find that the ordinance was constitutional. However, no such showing has been made here.

A teenage boy and his friend were playing paintball at the friend's father's farm. The paintballs were designed to break on impact, spraying paint but not physically injuring anyone. The boy pointed his paintball gun at the friend and fired a paintball at him. The paintball missed the friend and instead hit a horse that was grazing nearby. The horse, startled and in pain, started bucking and kicked up a rock that hit the friend in the head, causing him to lose vision in his right eye. In a negligence action against the boy, will the friend recover? (A) No, because shooting the gun was not the proximate cause of the friend's injury. (B) No, because shooting the gun was not the actual cause of the friend's injury. (C) Yes, because paintball is an adult activity. (D) Yes, because the boy shot the gun at his friend, which is sufficient for recovery under transferred intent.

(A) No, because shooting the gun was not the proximate cause of the friend's injury. Here, it was not foreseeable that shooting the paintball gun at the friend would cause the horse to be injured, kicking up a rock that then hit the friend. Because there is no proximate cause, there can be no action in negligence.

A State A resident received a job offer on the other side of the country. He decided to take the offer and move to State C. He went to his local Brand Z car dealer in State A to shop for a new Brand Z. He asked the dealer for advice, stating that he was thinking about driving across the country. Based on the dealer's assertions that the new Brand Z Model X "was perfect for all kinds of weather," the man bought the Model X. On the third day of his trip, some 1,000 miles from either State A or State C, he was involved in an accident in State B. The car slid out of control and smashed into a fence. Due to the nature of his injuries, the man never made it to State C or took the job. The man sued the State A car dealer for $100,000 in a State B state court, alleging a design defect in the car. Does the State B court have jurisdiction over the car dealer? (A) No, because the car dealer does not have sufficient minimum contacts with State B. (B) No, because the car was sold in State A to a resident of State A and the car dealer is a resident of State A. (C) Yes, because the car dealer knew the vehicle was likely to travel all around the country. (D) Yes, because as a commercial seller, the car dealer purposefully availed itself of interstate commerce, and so being sued in State B would not offend traditional notions of fair play and substantial justice.

(A) No, because the car dealer does not have sufficient minimum contacts with State B. Where a party is not a citizen of a state, the court may still establish personal jurisdiction over that party if he has a minimum number of contacts with the state. A minimum contacts analysis looks at the nature of the defendant's relationship with the forum state and whether the defendant would have reasonably expected to be called into court in the forum state. If the defendant purposely involves himself in transactions within the state, then he receives the benefits and privileges of that state's laws, and so it is fair that he is exposed to the jurisdiction of that forum state as well. Importantly, if a defendant purposely availed himself of the benefits of the forum state, sufficient minimum contacts for personal jurisdiction would exist. Here, the car was the only connection between the dealer and State B. While the dealer may have had some abstract knowledge that the car could travel the country, he did not know that the car was going to State B, and importantly, he did not purposefully direct any of his actions toward State B.

A landlord and a tenant entered into a lease for a period of one year for an apartment. Three months into the lease, the floor collapsed under the living room and part of the bedroom due to work that the landlord was doing in the apartment below. When the landlord told the tenant that it would take at least three months to repair the floors, the tenant moved out. When the tenant refused to pay rent the next month, the landlord sued the tenant. Is the tenant obligated to continue paying rent? (A) No, because the covenant of quiet enjoyment has been breached. (B) No, because the tenant has been actually evicted from the property. (C) Yes, because although the covenant of quiet enjoyment has been breached, the tenant is only entitled to a rent abatement. (D) Yes, because the landlord is working to repair the apartment.

(A) No, because the covenant of quiet enjoyment has been breached. Every lease contains a covenant of quiet enjoyment (express or implied). A landlord is deemed to have breached this covenant if the landlord actually or constructively evicts a tenant. A constructive eviction occurs when: (1) the landlord's act substantially and permanently interferes with the tenant's use and enjoyment of the premises; and (2) the tenant moves out. In this case, both of these factors are present. When a tenant has been constructively evicted, he may treat the lease as terminated and withhold rent.

A man's mother has lived with him for 20 years, and for the past several months she has been suffering from terminal cancer. Tired of her constant demands, the man begins to add a small amount of poison to his mother's liquid medication each day. When the mother dies, the medical examiner determines that cancer would have killed her within a few days, but that her death was hastened by the poison. If the man is prosecuted for murder, what should the result be? (A) The man should be convicted because his actions showed a depraved indifference to the value of human life. (B) The man should be acquitted because his mother was ill and in pain. (C) The man should be acquitted because his mother would have died almost immediately from cancer. (D) The man should be acquitted because the man's actions alone would not have caused the death.

(A) The man should be convicted, because his actions showed a depraved indifference to the value of human life. The act of introducing a poison into the mother's medication does show a reckless indifference to the value of her life. While the facts do make it clear that the mother would have died very soon anyway, the son's actions hastened that death and were done in a way that demonstrated an indifference to whatever life his mother still had left. Because these actions will meet the standard for malice, he will be convicted of her murder.

Three friends were driving home together from a local tavern. The driver had not consumed any alcoholic beverages at the tavern. On their way home, the three friends observed a man wandering haphazardly across the busy highway. One friend remarked, "I hope that guy makes it home. If he's not careful, he could get killed." The driver said, "That's his problem. I have to get home. I can't be bothered to stop and help him." After being dropped off at his home, the friend felt pangs of guilt and decided to go back and help the man that he'd seen. The friend asked his brother, who had not been drinking, to drive back to find and help the man. When the brothers returned to the area where the man had been, they found him lying on the shoulder of the highway, obviously injured. They helped the man into their car and the brother drove the man to the hospital. When they arrived at the hospital, the man was admitted with a broken leg. The leg had been broken when the man was wandering on the highway and was struck by a vehicle that had left the scene. The broken leg was the man's only injury. If the man sues the driver of the car the friend had been in on the way home from the tavern for damages for his injuries, will he prevail? (A) No, because the driver was under no duty to help the man. (B) No, because the man was involved in an abnormally dangerous activity. (C) Yes, because a reasonable person would have helped the man. (D) Yes, because the driver breached his duty to the man.

(A) No, because the driver was under no duty to help the man. The man, seeking to establish the driver's liability in negligence, must first prove that the driver owed a legal duty to the man. The element of duty is usually described as an obligation recognized by law that requires the defendant to conform to a certain standard of conduct to protect others against unreasonable risk. The traditional rule is that there is no affirmative duty to aid or protect a plaintiff who is at risk of injury unless such action is undertaken by a defendant. Here, given that the driver did not owe any duty to protect the man, the man cannot prevail in a negligence action against the driver.

A famous football player had his dog stolen from his yard. He immediately went to social media and implored anyone with details to contact him. He put a description of the dog, where it was last seen, and said that he would be forever indebted to whomever found his dog. One particular fan immediately went out looking for the dog in the area surrounding the player's home. A few days passed and the player became desperate, so he offered a $5,000 reward for finding the dog. After several days of looking, the fan finally saw the dog tied to a bench. He untied the dog and went onto the player's website to contact him. The fan then saw the reward offer and returned the dog, demanding the reward. The player refused to pay the fan the $5,000. Is the player liable to pay the fan the reward money? (A) No, because the fan did not know of the reward before he found the dog. (B) No, because the player was unaware that the fan had accepted the offer. (C) Yes, because the fan found the dog as requested. (D) Yes, because the fan learned of the reward before he returned the dog.

(A) No, because the fan did not know of the reward before he found the dog. A unilateral contract is created when an offeror seeks acceptance by performance, rather than by a promise in return. An offer in a unilateral contract cannot be revoked once performance has begun. Here, however, the fan undertook performance not based on the reward money, but based on the request for help. Further, the fan did not learn of the reward offer until after he had already performed by finding the dog. Therefore, his performance did not act to accept the player's unilateral offer.

A pedestrian was walking in front of a hotel in the downtown area when a chair was thrown from an unidentified window. The chair struck the pedestrian on the head, knocking her unconscious. When the pedestrian recovered consciousness, she found herself in a nearby hospital with a doctor in attendance. An examination revealed that the pedestrian had suffered a concussion and severe head lacerations. A subsequent investigation revealed that the chair had, in fact, been thrown from a window at the hotel. The pedestrian sued the hotel for negligence. At trial, it was conceded by the hotel that it was one of their chairs, that it came from a room on the 12th floor, and that the hotel was fully occupied on the night of the incident. However, none of the hotel's employees or guests admitted culpability for the incident. Will the doctrine of res ipsa loquitur enable her to recover? (A) No, because the furnishings of the room are within the control of the guests. (B) No, because the hotel is not vicariously liable for the tortious conduct of its employees. (C) Yes, because a chair is not usually thrown from a window in the absence of someone's negligence. (D) Yes, because it is undisputed that the chair came from the hotel.

(A) No, because the furnishings of the room are within the control of the guests. The doctrine of res ipsa loquitur allows a plaintiff to recover when there is strong circumstantial evidence that the defendant was negligent, even though the plaintiff cannot identify exactly what the defendant did that was negligent. The doctrine originated in Byrne v. Boadle [159 Eng. Rep. 299 (1863)], a British case involving a person who was walking down the street when he was suddenly struck on the head by a barrel of flour. Figuring that the barrel must have come from the warehouse located next to the spot where he was hit by the barrel, the injured person sued the owner of the warehouse. Although the plaintiff could not show exactly what anyone did that was negligent, the court said that the situation spoke for itself, and that negligence could be inferred because it seemed that a barrel would not fall unless one of the defendant's employees had been negligent in some way. A different situation, with a different result, arose in Larson v. St. Francis Hotel [188 P.2d 513 (Cal. Ct. App. 1948)], where a plaintiff was struck by a chair thrown by an unidentified person from an unidentified window of the defendant's hotel. The plaintiff was unable to recover because the building in question was a hotel. It was not clear that the chair was thrown by an employee of the hotel, rather than a guest. The chair, thus, was not under the exclusive control of the hotel and its employees, and there was no sound basis for inferring that the hotel or its employees must have been negligent. This question is like the Larson case, rather than the Byrne case, so res ipsa loquitur does not apply, and the plaintiff will lose.

A border patrol officer found packages of heroin glued between the soles of gym shoes being exported from Colombia into the United States. Narcotics agents arranged for a controlled delivery of the boxes to the address listed on the packages. After the boxes were delivered to the address, the house was put under surveillance. One week later a woman driving a car with expired plates arrived at the address and began loading the boxes into the trunk of the car. As she drove away the car was stopped by the narcotics agents and the driver was arrested. The agents searched the entire vehicle without first obtaining a warrant. The box was opened and the heroin was found concealed in the soles of the gym shoes. At her trial on charges of possession of heroin and conspiracy, the defendant moved to suppress the search of the trunk. Should the motion be granted? (A) No, because the officers had probable cause to search the trunk. (B) No, because the search was conducted incident to a lawful arrest. (C) Yes, because the warrantless search was limited to the wingspan of the driver. (D) Yes, because the agents entrapped the defendant by arranging the controlled delivery.

(A) No, because the officers had probable cause to search the trunk.

A plaintiff and defendant were playing football. The plaintiff knew that the defendant had a reputation for being an extremely competitive player. During the game, the plaintiff's eye was severely injured when the defendant's finger came into contact with it while trying to catch a pass. The plaintiff sought medical attention, and the doctor diagnosed him with a scratched cornea. The plaintiff subsequently brought a negligence action against the defendant. Will the plaintiff prevail? (A) No, because the plaintiff impliedly consented to this type of injury. (B) No, because the plaintiff voluntarily played the game. (C) Yes, because the defendant's actions fell below the standard of care. (D) Yes, because this type of physical injury is beyond the scope of consent.

(A) No, because the plaintiff impliedly consented to this type of injury. A plaintiff is said to assume the risk of injury from a defendant's negligence if the plaintiff expressly or impliedly consents to undergo the risk created by the defendant's conduct. The plaintiff will be barred from recovery, or his recovery will be reduced, under the assumption of risk doctrine if the defendant establishes that the plaintiff had knowledge of and appreciated the nature of the danger involved. In some circumstances, a court will determine that a defendant has no obligation to be non-negligent toward the plaintiff because of the nature of the activity in which they are engaged. The most common context is sports. Here, the plaintiff knew or should have known that such an injury could occur. Thus, even if the defendant was negligent, the plaintiff will likely not prevail.

Police officers received a tip from a reliable informant that a night club was operating a drug distribution business out of the basement in the building where the club was located. During a police surveillance of the building, the officers observed a patron leaving the club by the basement stairs and placing what appeared to be a weapon inside his raincoat. The officers stopped the patron on the street who denied that he was ever in the basement of the night club. At that point the officer, fearing for his safety, patted down the outer clothing and felt small tube-shaped vials in the defendant's shirt pocket. The defendant was placed under arrest. The substance in the vials field-tested positive for cocaine. The defendant filed a motion to suppress the cocaine. At the suppression hearing, the arresting officer testified that based on his 20 years of experience, he concluded that the tube-shaped objects were glass vials used by drug dealers to sell narcotics. If the court believes the officer's testimony, should the evidence of the cocaine be suppressed? (A) No, because the stop was reasonable in duration and the officer had a reasonable suspicion that the defendant was armed and dangerous. (B) No, because the officer had probable cause to believe that crime had been committed. (C) Yes, because the scope of a pat down in a Terry stop is limited to a search for weapons. (D) Yes, because the informant supplied no information about the patron to police before the stop.

(A) No, because the stop was reasonable in duration and the officer had a reasonable suspicion that the defendant was armed and dangerous. SCOTUS held in Terry v. Ohio [392 U.S. 1 (1968)] that a police officer may legally stop a person without probable cause if the officer has a reasonable and articulable suspicion of criminal activity. If at the time of the stop, the officer reasonably believes that the person may be armed and dangerous, the officer is authorized to conduct a protective frisk to locate any weapon that may endanger the officer's safety. The scope of a protective frisk is limited to a pat-down of the defendant's outer clothing. However, the Supreme Court also held in Minnesota v. Dickerson [508 U.S. 366 (1993)] that the officer may, during the course of a protective frisk, reach inside the defendant's clothing and seize any item that the officer reasonably believes is a weapon or contraband based on its "plain feel." This is known as the plain feel doctrine.

A man inherited a large estate with a log cabin that had been constructed many generations ago using wood from the surrounding old-growth forest. For over a century, the family had been exploiting the timber on the property by permitting companies to enter and remove trees at a sustainable rate, although never from a five-acre area around the log cabin. Not wanting to give up his life in the city, the man leased the log cabin and the five acres around it to a tenant. In the third year of the lease, a serious rainstorm caused part of the roof and walls of the cabin to collapse. The tenant, wanting to retain the locally sourced nature of the cabin, cut down several trees nearby to use in making repairs. While the repairs kept the log cabin from becoming unusable, the fact that old-growth trees were used for the repairs caused a net loss in the value of the property due to how long it would take for them to be replaced. When the man learned of this, he sued the tenant for the loss in value to the property and the value of the trees that had been cut down. Will the tenant be required to reimburse the man? (A) No, because the tenant used the trees to repair the property. (B) No, because of the open mines doctrine. (C) Yes, because the tenant committed voluntary waste. (D) Yes, because the tenant committed ameliorative waste.

(A) No, because the tenant used the trees to repair the property. A tenant is not allowed to do anything that adversely affects the future interest that follows the present possession; doing so is called waste. In the case of a lease, this means that the tenant cannot adversely affect the landlord's interest in the property. When a tenant voluntarily commits an act that has more than a trivial injurious effect on or change in the property, the tenant has committed voluntary waste. However, despite the prohibition on voluntary waste, natural resources may be consumed in three situations: (1) for the repair and maintenance of the property; (2) with permission of the grantor; or (3) under the open mines doctrine. Because the tenant cut down the trees in order to repair and maintain the property, he will not be required to reimburse the man for their value.

A landlord lived with his two tenants as roommates in a townhouse. The lease agreements specified that each tenant had private access to one of the bedrooms and access to the shared common areas of the house. One day, due to poor upkeep, one of the pipes burst under the kitchen sink, flooding the kitchen and rendering the entire room unusable. The tenants salvaged what they could of their property from the kitchen and complained to the landlord to fix it, but despite constant promises that he would get to it, the kitchen was never repaired. Are the tenants entitled to withhold their next month's rent payments? (A) No, because they are entitled only to a rent abatement. (B) No, because they did not move out of the townhouse. (C) Yes, because they have been constructively evicted from part of the premises (D) Yes, because the landlord failed to maintain the kitchen in good repair.

(A) No, because they are entitled only to a rent abatement. Where the landlord fails to maintain the common areas used by all tenants such that the tenant breaches the covenant of quiet enjoyment, this will be considered a constructive eviction. When the tenants are only constructively evicted from part of the premises, there has been a partial constructive eviction. Under the majority rule, if a tenant is constructively evicted from part of the premises, the tenant may receive a rent abatement, but is not relieved of all liability for rent. Therefore, the tenants are not entitled to withhold their entire rent payment, but may receive an abatement.

A fitness instructor wanted to open a new gym in a city neighborhood known locally as "Fitnessville," because of its many gyms, fitness studios, and juice bars. The man purchased an empty lot and submitted the appropriate plans to the city authorities for a building permit. The building inspector, backed by the city zoning and planning commission, denied the instructor's application for a building permit, stating the following reasons in a written decision: (1) the local neighborhood could not sustain any further traffic entering and exiting the area; (2) the neighborhood already had 12 gyms of a similar nature, four of which would be on the same street and block as the proposed gym location; and (3) the location of the entrance to the planned gym was too close to a dangerous intersection. The fitness instructor requested an opportunity before the city zoning and planning commission to address each of these reasons for denying his building permit, but the city authorities replied that the decision was final. The fitness instructor then brought an appropriate action challenging the constitutionality of the city's decision. Which of the following provides the fitness instructor with the strongest argument? (A) Procedural due process. (B) The Contract Clause. (C) The Privileges and Immunities Clause of the 14th Amendment. (D) The Takings Clause

(A) Procedural due process. Under the 5th & 14th Amendments, citizens are protected against the deprivation of life, liberty, or property without the due process of the law. Procedural due process guarantees notice & the right to be heard.

A man has been arrested and charged with homicide for the intentional killing of a neighbor. At trial, the prosecution argued that the man committed murder, and the defendant, in turn, claimed that he should be found guilty of voluntary manslaughter. Regarding voluntary manslaughter, which of the following statements is most accurate? (A) Provocation is adequate if it would cause a reasonable person to lose self-control. (B) Provocation is adequate if it would cause the defendant to lose self-control. (C) As long as the man was still laboring under the heat of passion from the incident, it is irrelevant whether a reasonable person would have cooled off. (D) A showing by the man that he was intoxicated would reduce a charge of murder to manslaughter.

(A) Provocation is adequate if it would cause a reasonable person to lose self-control. Voluntary manslaughter involves the intentional killing of another in the heat of passion brought about by adequate provocation. Provocation is adequate if it would cause a reasonable person to lose self-control.

A defendant was arrested for assault. He knew that he was a "three-strike" candidate and did not want to spend the rest of his life in prison, so he asked for an opportunity to set up his boss, whom he claimed sells a substantial amount of professionally stolen merchandise. The police agreed to work with the defendant and sent him to the boss's condo with marked merchandise that contained transmitters. When the defendant left his boss's condo, he no longer had the merchandise. He was under surveillance from the time he left the police van until he went in the front door of the condo, and again from the moment, he left the condo until he returned to the van. The police immediately move in to arrest the boss for receiving stolen property. They knock and announce their presence three times, but there is no answer. They then ram the door and enter, finding the marked stolen property and numerous other items of expected stolen property, but not the boss, who had exited by the back stairwell to take his dog for a walk. The boss was subsequently arrested upon his return. At trial, the boss's attorney moves to suppress the seized property. The court should take what action? (A) Suppress the evidence, because there were no exigent circumstances. (B) Suppress the evidence, because the police had no warrant. (C) Admit the evidence, because the police knocked and announced their presence before entering. (D) Admit the evidence, because they had evidence against the boss for the offense.

(A) Suppress the evidence, because there were no exigent circumstances. This is correct, because there was no reason to believe that the boss had any idea about the sting operation or that the evidence would be moved or destroyed while they obtained an arrest warrant for the boss or a search warrant for his apartment. Without exigent circumstances, the police must obtain a warrant.

A plaintiff wanted to sue two defendants for damages stemming from the defendants' reprinting of her novel in violation of copyright laws. The defendants reprinted and sold the novel in the Northern District of Illinois. One defendant resides in the Eastern District of Michigan, while the other resides in the Western District of Illinois. The plaintiff resides in the Northern District of Iowa. Where would venue be proper for this action? (D) The Northern District of Illinois, the Western District of Illinois, the Northern District of Iowa, or the Eastern District of Michigan. (C) The Northern District of Illinois, the Western District of Illinois, or the Eastern District of Michigan. (B) The Western District of Illinois or the Eastern District of Michigan. (A) The Northern District of Illinois.

(A) The Northern District of Illinois. Pursuant to 28 U.S.C Section 1391, venue is appropriate: (1) where any defendant resides, provided that all defendants are within the same state; (2) in the district where a substantial portion of events occurred; or (3) in the district where any defendant can be found, if there is no other district in which the action could be brought. Here, the action was based on copyright, a federal question. Venue is appropriate in the Northern District of Illinois because that is where the defendants reprinted and sold the novel. Because the defendants live in different states, the plaintiff cannot take advantage of the rule allowing a plaintiff to sue in any district where a defendant resides.

On October 1, a seller mailed a letter to a buyer offering to sell a specified quantity of shirts at list price. The buyer received the seller's offer on October 2. The next day, the buyer mailed the seller a letter of acceptance. The buyer then changed his mind and decided to reject the seller's offer. On October 5, the buyer mailed a letter of rejection to the seller. On October 4, the seller sent the buyer a letter revoking his original October 1 offer. On October 6, the seller received the buyer's acceptance. On October 7, the buyer received the seller's letter of revocation. The following day, the seller received the buyer's rejection. The buyer then demanded that the seller sell him the shirts at the specified price, and the seller refused. The buyer has sued for breach of contract. Judgment should be for whom? (A) The buyer, because he mailed his acceptance before mailing his rejection. (B) The buyer, because the seller received the buyer's acceptance before receiving the buyer's rejection. (C) The seller, because the buyer's dispatch of both a rejection and an acceptance evidences a lack of mutual assent. (D) The seller, because he mailed his revocation before receiving notice of the buyer's acceptance.

(A) The buyer, because he mailed his acceptance before mailing his rejection. This is a classic application of the mailbox rule. Under the mailbox rule, when the acceptance by mail is effective upon dispatch as long as the acceptance is properly posted. The fact that there was a later rejection and/or revocation is irrelevant, because the contract was formed on October 3, then the buyer mailed his acceptance of the seller's offer.

A seller sent an email to a potential buyer offering to sell his patio furniture to her for $5,000. The buyer immediately responded via email asking whether the offer included the umbrella that was sitting in the garage. The seller emailed back: "No, it does not; just what is sitting on the patio. If you want an umbrella, you will have to buy one." The buyer then ordered an umbrella that would fit on the table and matched the color of the chairs. Later that day, the buyer replied to the seller: "I accept your offer." The seller then wrote back, "I have changed my mind, I've decided to keep the furniture." If the buyer sues the seller claiming there was a valid contract, who is likely to prevail? (A) The buyer, because she accepted the seller's offer within a reasonable amount of time and before the seller revoked his offer. (B) The buyer, because her ordering the umbrella is a sufficient act in reliance on the seller's offer. (C) The seller, because the offer lapsed after the buyer did not accept the original offer. (D) The seller, because the buyer's initial email was a counteroffer which terminated her power of acceptance.

(A) The buyer, because she accepted the seller's offer within a reasonable amount of time and before the seller revoked his offer. The UCC governs contracts for the sale of goods. An offer was made by the seller via email. It was not a firm offer, because it did not specify that it would remain open for a period of time. Therefore, it is revocable either through lapse of time or a rejection or counteroffer. It appears from the facts that this conversation is taking place on the same day, so the offer would not be revoked due to lapse of time. The buyer notified the seller of her intent to accept before the seller revoked the offer, and within a reasonable time. Therefore, a contract was formed.

A construction company was doing repairs and replacing portions of a sidewalk and railing next to a lake. The construction crew started tearing out the old sidewalk and railing but stopped work when it started to get dark. The construction crew left without putting up a warning sign or barrier around the work area. A few hours later, a jogger came along the sidewalk. Not realizing the construction work was in progress there, the jogger stumbled and fell at the spot where the construction crew had torn up the sidewalk and railing. The jogger fell into the lake. As the jogger was attempting to stay afloat, he began screaming, "Help! Help! I can't swim. I'm drowning." His screams attracted the attention of a person who was passing on his bicycle. The cyclist immediately hurried to assist the jogger. As the cyclist was leaning over the edge of the lake, trying to help the jogger get out of the water, he lost his balance and fell into the lake. Both the jogger and cyclist suffered serious bodily injuries before they were pulled out of the water by police. In a negligence action by the cyclist to recover for his personal injuries, what is the probable outcome? (A) The construction company will be held liable, because the cyclist's attempt to rescue the jogger was foreseeable. (B) The construction company will be held liable, because the construction company would be strictly liable to anyone injured by the failure to put adequate warnings or barriers around the site of the sidewalk repairs. (C) The construction company will not be held liable, because the cyclist assumed the risk by leaning over the edge of the lake. (D) The construction company will not be held liable, because the construction company could not foresee that anyone would be hurt while trying to rescue someone from the lake.

(A) The construction company will be held liable, because the cyclist's attempt to rescue the jogger was foreseeable. The rescue doctrine recognizes that if a defendant's negligence puts someone in danger, it is foreseeable that another person will come to the rescue of the person who is in danger. If the rescuer is injured, the rescuer can hold liable the defendant whose negligence created the need for the rescue attempt. As Judge Cardozo stated in Wagner v. International Railway Co. [133 N.E. 437 (N.Y. 1921)], "Danger invites rescue."

The owner of a lakefront house had leased the house to a renter for the past three summers. Last year, on October 1, the owner sent the renter a letter stating, "Because you have been such a great tenant, I will rent you the house on the same terms as last year. However, I need your answer by February 1." The renter, uncertain as to when he would be taking his annual vacation, decided to put the letter aside for the time being, intending to respond to it by February 1. On January 15, the owner received an offer from the renter's cousin to rent the house at a higher price than the renter had paid the prior year. The owner accepted. On January 20, after learning about his cousin's rental, the renter telephoned the owner and said, "I have decided that I want the house for the summer." The owner replied that he had already accepted an offer to rent the house from the renter's cousin. The renter then filed a lawsuit seeking to enforce the contract. If the renter is unsuccessful in his lawsuit, which of the following provides the best reason? (A) The contract with the cousin indirectly revoked the offer to the renter. (B) The contract with the cousin terminated the offer to the renter. (C) There was no mutual assent, because the renter was unsure whether he wanted to vacation at the lake. (D) A common law offer cannot extend beyond three months without consideration.

(A) The contract with the cousin indirectly revoked the offer to the renter. An offer is a manifestation of intent by the offeror to be bound by the contract that is communicated to the offeree with definite and certain terms. Here, the owner wrote, "I will rent you the house on the same terms as last year. I need your answer by February 1," thereby demonstrating his intent to be bound by a contract. Although the letter itself did not specify the terms, the reference to "the same terms as the previous year" is sufficiently definite to form an enforceable offer. The owner's letter also included the stipulation that he must have a response from the renter by February 1. Generally, promises to keep an offer open are enforceable and irrevocable as long as the offeree pays separate consideration for the option. In this case, the renter did not pay separate consideration for the option, and, as such, the owner was free to revoke the offer at any time before February 1. When the renter learned of the fact that the owner had rented the lakefront house to his cousin, this indirectly revoked the offer to the renter, and he could no longer accept the offer on January 20.

A defendant was charged with felony murder as a result of his setting fire to the victim's house. The victim was upstairs sleeping when the fire was set, and he died in his bedroom during the fire. If the defendant can prove the facts to support his defense, which of the following assertions would least likely remove liability for felony murder? (A) The defendant did not intend to kill the victim. (B) The defendant was insane when he set the fire. (C) The defendant was coerced by another to set the fire. (D) The victim died in his sleep before the fire spread to his bedroom.

(A) The defendant did not intend to kill the victim. Lack of intent to kill would least likely relieve the defendant of liability for felony murder. The common law felony-murder rule provides that one whose conduct brought about a death in the commission of a felony was guilty of murder. For a defendant to be found guilty of felony murder, he need not have the intent to kill the victim. Thus, choice (A) would not be a good defense.

A city police department needs to hire an undercover agent to infiltrate a racist militant group whose members are all white. An African American officer applied for the assignment but was rejected even though he had extensive undercover experience. A much less experienced white officer was chosen for the position. The department admitted that race was the determinative criteria in filling the position. If the officer who did not receive the position sued, what would be the most likely result? (A) The department's decision will be upheld as necessary to achieve a compelling interest. (B) The department's decision will be upheld as rationally related to a legitimate state interest. (C) The department's decision will be overturned because race cannot be used as a criterion in government job assignments. (D) The department's decision will be overturned as a violation of the Equal Protection Clause of the Fourteenth Amendment.

(A) The department's decision will be upheld as necessary to achieve a compelling interest. The primary criterion for the government job assignment in this case is race. Under the Equal Protection Clause, that classification is only valid when necessary to achieve a compelling interest. This is one of the rare cases in which this standard is met. An African American police officer would be totally ineffective penetrating an organization in which race is the primary criterion for membership. And therefore the racial classification in this case will be upheld.

A police officer saw a van that had a bumper sticker with the symbol of a rock band that is known to have fans that regularly take recreational drugs during the concert as a way to enhance their enjoyment of the show. The van entered a restaurant parking lot and the driver and her friend walked into the restaurant. The policeman decided to search the van for drugs. He opened the van door in the back which was unlocked and started looking inside. He saw a bag of marijuana. The police officer arrested both the driver and her friend when they came back. What is the most likely outcome of the driver's and her friend's motions to suppress the evidence? (A) The driver's motion will be granted, but the friend's will not. (B) The friend's motion will be granted, but the driver's will not. (C) Both motions will be granted as to the marijuana, even though the marijuana was in plain view. (D) Neither motion will be granted.

(A) The driver's motion will be granted, but the friend's will not. The search was clearly illegal in that the officer did not obtain a warrant and there were no exigent circumstances or other applicable exceptions to the warrant requirement. Thus, the driver's motion to suppress the marijuana obtained during the search of her van will be granted. The driver's friend, however, has no standing to challenge the search of the driver's van, and, therefore, the marijuana will not be excluded as against her.

A defendant is wandering down the street in a drunken stupor, periodically stepping off the sidewalk onto the street and into the path of oncoming traffic. A police officer saw the defendant and decided he should help. The officer stopped the defendant and asked his name and how he was doing. The defendant mumbled something about how he "loved her" and how he "shouldn't have done it." Figuring that the defendant was simply drunk, the officer ignored his remarks and decided the best course of action would be to take away any remaining alcohol from the defendant so that he could sober up. The officer reached into the defendant's coat pockets and, to his surprise, found a small pistol. "What are you doing with this, man?" the officer asked. When the defendant did not reply, the officer became angry. "I asked you a question," the officer said. "You'd better answer it." The defendant then told him that he had killed his girlfriend and dumped her body in a drainage ditch. The officer arrested the defendant and read him his rights. At the defendant's murder trial, the state moved to admit the evidence of the gun. How is the court likely to rule? (A) The evidence will be suppressed because the officer had no reason to believe that the defendant was armed. (B) The evidence will be suppressed because the Terry stop of the defendant was illegal. (C) The evidence will be admitted because there was reasonable suspicion for the Terry stop. (D) The evidence will be admitted because guns constitute a threat to public safety.

(A) The evidence will be suppressed, because the officer had no reason to believe that the defendant was armed. A Terry stop has 2 parts: the stop & the frisk. For a constitutional stop, an officer must have a reasonable suspicion that criminal activity may be "afoot." During the course of that stop, if the officer has a reasonable and articulable suspicion that the suspect is armed and dangerous, he may conduct a pat-down frisk search for concealed weapons. Here, the officer's Terry stop of the defendant was likely valid, in that the defendant's drunken weaving along the sidewalk was bizarre enough to suggest that criminal activity was afoot. However, following this stop, there was nothing to suggest that the defendant was armed and dangerous. By reaching into the defendant's pocket, the officer conducted an actual, and unlawful, search.

A man is walking down the street and is weaving down the street in a drunken stupor. A police officer saw the man and asked, "Do you need help?" The man, who reeked of alcohol, said, "Tired. I am going home." The officer ignored his remarks and decided the best course of action would be to take away any remaining alcohol from the defendant so that he could sober up. The officer reached into the defendant's coat pockets and, to his surprise, found a small pistol. "What are you doing with this, man?" the officer asked. When the defendant did not reply, the officer became angry. "I asked you a question," the officer said. "You'd better answer it." The defendant then told him that he had robbed a liquor store. The officer arrested the defendant and read him his rights. At the defendant's robbery trial, the state moved to admit the evidence of the gun. How is the court likely to rule? (A) The evidence will be suppressed because the officer had no reason to believe that the defendant was armed. (B) The evidence will be suppressed because the Terry stop of the defendant was illegal. (C) The evidence will be admitted because there was reasonable suspicion for the Terry stop. (D) The evidence will be admitted because guns constitute a threat to public safety.

(A) The evidence will be suppressed, because the officer had no reason to believe that the defendant was armed. A Terry stop has two parts: the stop and the frisk. For a constitutional stop, an officer must have a reasonable suspicion that criminal activity may be "afoot." During the course of that stop, if the officer has a reasonable and articulable suspicion that the suspect is armed and dangerous, he may conduct a pat-down frisk search for concealed weapons. Here, the officer's Terry stop of the defendant was likely valid, in that the defendant's drunken weaving along the sidewalk was bizarre enough to suggest that criminal activity was afoot. However, following this stop, there was nothing to suggest that the defendant was armed and dangerous. By reaching into the defendant's pocket, the officer conducted an actual, and unlawful, search.

Paul, a citizen of Idaho, and Greg, a citizen of Oregon, sued Darryl, a citizen of Montana, for breach of contract occurring in Montana. The action was filed in the U.S. District Court in Oregon, and Darryl was improperly served. Upon learning of the suit, Darryl made a successful motion to transfer the case to the U.S. District Court in Montana. Which of the following is true? (C) The federal court in Oregon should have denied Darryl's motion to transfer the case because he was improperly served. (A) The federal court had the authority to transfer the action to Montana. (B) The federal court in Oregon should have dismissed the action because venue in the U.S. District Court in Oregon was improper. (D) The federal court improperly transferred the action to Montana.

(A) The federal court had the authority to transfer the action to Montana. The action was not properly commenced in the U.S. District Court in Oregon, because the defendant does not reside in Oregon, and the cause of action did not arise there. However, under the applicable law, the action may be transferred to the U.S. District Court in Montana (where the sole defendant was domiciled).

A homeowner purchased a new vacuum cleaner. A few days later, the homeowner received a severe electric shock while using the vacuum cleaner. The homeowner realized that there was a short in the wiring of the vacuum cleaner. The homeowner called the store that sold the vacuum cleaner and arranged to return it for a refund. The homeowner was busy, however, and would not have time to make a trip to the store for several days, so he put the vacuum cleaner in a corner in his living room to store it until he had a chance to take it to the store. The next day, the homeowner had an old friend from out of town arrive for a three-day visit. The homeowner had a spare room and loved to have overnight guests, so he had invited this old friend to come for a visit and was excited to see her. Shortly after the guest's arrival, however, the homeowner had to go to work for a few hours, so he told his friend to make herself comfortable in the house while he went to his office. While the homeowner was away from the house, his friend got hungry and made herself a snack. After finishing the snack, she realized that she had dropped a lot of crumbs on the carpet. Wanting to be a polite guest, she decided to clean up the mess herself before the homeowner returned. She saw the vacuum cleaner in the corner, plugged it in at an electric outlet, and started to vacuum up the crumbs when she received a very strong electric shock as a result of the vacuum cleaner's faulty wiring. If the guest asserts a claim against the homeowner, what is the most likely outcome? (A) The guest will recover because the homeowner knew about the hazardous condition of the vacuum cleaner and yet failed to warn the guest. (B) The guest will recover because the homeowner had a duty to make a reasonable inspection of the home to discover any unknown dangers. (C) The guest will not recover, because the homeowner did not create the risk of harm. (D) The guest will not recover, under the family purpose doctrine.

(A) The guest will recover, because the homeowner knew about the hazardous condition of the vacuum cleaner and yet failed to warn the guest. A guest who is present for social rather than business purposes is a licensee. Even though the person is invited and urged to come to the property, the guest is not an invitee. For a licensee, the possessor of the property owes no duty of inspection and affirmative care to make the premises safe for his visit. Usually, the guest understands when he comes that he is to be placed on the same footing as one of the family, and he must take the premises as the occupier himself uses them, without any inspection or preparation for his safety; he also understands that he must take his chances as to any defective conditions unknown to the occupier, and is entitled at most to a warning of dangers that are known. Here, the homeowner, therefore, can be held liable for failing to warn the guest about a known danger.

A landlord was the owner in fee simple of a tract of land on which was situated a large office building. The landlord leased the land and building thereon to a tenant for a term of seven years, commencing on August 1, 2001 and terminating at midnight on July 31, 2008. The lease contained the following provision: "The tenant covenants to pay the rent of $750 per month on the first day of each month." After three years of the lease had expired, the tenant assigned the entire balance of the lease period to an attorney, who took immediate possession of the leased property. Then, in 2005, the attorney assigned his leasehold interest to a doctor. The doctor went into possession but failed to pay any rent for two months. After the doctor failed to make his rental payments for the first two months of his lease, the landlord brought suit against the tenant to recover for the unpaid rent. For whom should the court enter judgment? (A) The landlord, because the tenant's contractual obligation under the lease survived the assignments. (B) The landlord, because he did not object to the assignments. (C) The tenant, because only the doctor, as assignee, would be held liable. (D) The tenant, because his assignment to the attorney constituted a novation, thereby extinguishing his obligation to pay rent.

(A) The landlord, because the tenant's contractual obligation under the lease survived the assignments. Where a tenant-assignor transfers all of his leasehold interest to an assignee, such assignment does not release the original tenant from his contract obligations to the landlord under the terms of the lease. This is true even when the assignee thereafter, assigns/transfers his leasehold interest to a sub-assignee. Thus, this is the best answer.

A state resident could purchase a license solely for hunting elk for $9.00. A nonresident, in order to hunt elk, was required to purchase a combination license at a cost of $225.00; this entitled him to take one elk, one deer, and one black bear. A nonresident, however, could obtain a license restricted to deer for $51.00. A resident was not required to buy any combination of licenses, but if he did, the cost to him of all the privileges granted by the nonresident combination license was $30.00. Due to its successful management program for elk, the state has not been compelled to limit the overall number of hunters by means of drawings or lotteries, as have other states. Elk are not hunted commercially in the state. Nonresident hunters seek the animal for its trophy value; the trophy is the distinctive set of antlers. Whereas the interest of resident hunters more often may be in the meat, among nonresident hunters, big-game hunting is clearly a sport in the state. Two residents of another state bring suit against the state. They assert in their complaint that the disparities between residents and nonresidents in the state hunting license system is unconstitutional. How should the court rule on the state's hunting license system? (A) The license system is constitutional because it is within the police power of a state to regulate a recreational, noncommercial activity. (B) The license system is constitutional because there is a compelling state interest. (C) The license system is unconstitutional because it violates the Privileges and Immunities Clause of Article IV, Section 2. (D) The license system is unconstitutional because it violates the Equal Protection Clause of the Fourteenth Amendment.

(A) The license system is constitutional, because it is within the police power of a state to regulate a recreational, noncommercial activity. In Baldwin v. Montana Fish and Game Commission [436 U.S. 371 (1977)], the U.S. Supreme Court held that the Montana elk-hunting licensing scheme, as applied to nonresidents, was not a fundamental right under the Privileges and Immunities Clause of Article IV, Section 2. Furthermore, the Court majority found no discrimination in the distinctions drawn between residents and nonresidents under the Equal Protection Clause of the Fourteenth Amendment. The Supreme Court concluded that protection of the wildlife of a state is peculiarly within the police power of the state. The court also noted that the elk-hunting licensing scheme did not violate petitioners' "privileges and immunities" because hunting (on the part of non-Montana residents) was primarily a recreational endeavor. Choice (B) is incorrect. The reference to "compelling state interest" suggests that the court will apply strict scrutiny, but this is doubtful under Baldwin. Choice (C) is incorrect for the reasons stated in the discussion of choice (A), above. Finally, choice (D) is incorrect. The Equal Protection Clause provides heightened protections for groups whose legal classifications are suspect (e.g., classifications based on race, ethnicity, nationality, and religion) and for groups whose legal classifications are quasi-suspect (e.g., gender and illegitimacy). The residents may have been the subject of discrimination by the state, but they are not members of a suspect or quasi-suspect class. Only rational review will be applied, which the state probably will pass with ease.

A state-owned vast tracts of woodlands containing highly desirable lumber, and provided licenses to businesses to exploit these resources, with various regulations in place to ensure that a certain percentage of them would be preserved and the woodlands would not be completely depleted. State building codes required that a minimum of 10% of the lumber used by construction companies be the highly desirable, locally sourced lumber, and in-state commercial enterprises could obtain a license from the state to cut timber up to a set amount for an annual fee of $7,500. Out-of-state commercial enterprises, which made up only about a small fraction of the lumber industry in the state, could obtain a license for an annual fee of $20,000 but were permitted to remove 10% more lumber than in-state businesses, a difference which enabled those nonresident enterprises to make a profit greater than the difference in license costs. The schemes were adopted after research showed that they would keep the level of exploitation at or below the rate of regrowth without requiring the state to resort to a lottery system for available licenses, which would have been administratively costly and burdensome. A nonresident commercial enterprise sued the state claiming that the disparities in the licensing and regulatory scheme were unconstitutional. How should the court rule on the state's resource management system? (A) The license system is constitutional because it is within the police power of a state to regulate and preserve state-owned natural resources. (B) The license system is constitutional because there is a compelling state interest. (C) The license system is unconstitutional because it violates the Equal Protection Clause of the Fourteenth Amendment. (D) The license system is unconstitutional because it violates the Privileges and Immunities Clause of Article IV, Section 2.

(A) The license system is constitutional, because it is within the police power of a state to regulate and preserve state-owned natural resources. In Baldwin v. Montana Fish and Game Commission [436 U.S. 371 (1977)], the U.S. Supreme Court held that the protection of the wildlife of a state is peculiarly within the police power of the state. Furthermore, the Court later held that discrimination against nonresidents will be given special consideration if its purpose is the preservation of natural, state-owned resources [Sporhase v. Nebraska, 458 U.S. 941 (1982)]. Because the purpose of the disparate licensing schemes was to allow the limited exploitation of natural resources, these schemes will likely be found constitutional.

A man who was legally blind applied for admission to a state medical school. The medical school is a state institution and receives financial grants from the state government. After receiving notice of his rejection, the man contacted the director of admissions, who admitted that the man was denied admission solely due to the fact that he was blind. The man has since initiated an appropriate action against the medical school asserting that his equal protection rights have been violated. Which of the following correctly states the applicable burden of persuasion? (A) The man bears the burden to show the medical school's actions are not rationally related to a legitimate state interest. (B) The man bears the burden to show the medical school's actions do not further a compelling state interest. (C) The medical school has the burden to show that the man's denial of admission furthers an important state interest. (D) The medical school has the burden to show that the man's denial of admission is rationally related to a legitimate state interest.

(A) The man bears the burden to show the medical school's actions are not rationally related to a legitimate state interest. Discrimination based on a physical disability such as blindness implicates neither a fundamental right nor a suspect class. As such, rational basis scrutiny would be applied. The burden of persuasion would be on the plaintiff, the man, to show that the medical school's actions are not rationally related to a legitimate state interest. Students should note, however, that not all rational basis challenges fail. In Romer v. Evans [517 U.S. 620 (1996)], a Colorado law that prohibited any state entity from enacting measures that protected gays and lesbians from discrimination "flunked" the rational scrutiny test on two grounds. First, no legitimate interest was served by singling out a politically unpopular group and forbidding them the safeguards other persons enjoy. Second, the means used were not rationally related to the end result of protecting other citizens' freedom of association. In general, where discrimination motivated by an animus or hostility towards a particular group occurs, such legislation will be struck down. In this question, the man would have to show such invidious discrimination. Notwithstanding certain protections created by state governments and Congress (notably, the ADA) to prevent discrimination against persons with disabilities, the fact that the man was rejected based on his blindness per se does not warrant the use of heightened equal protection scrutiny.

A man offered to sell his barbecue to his neighbor for $100. After receiving the man's offer, the neighbor responded, "Let me think it over." The man then said, "If you say so." The next day, the man sold the barbecue to his brother for $100. Thereafter, the neighbor decided to accept the man's offer, but learned from a reliable source that the barbecue had been sold to the brother. If the neighbor sues the man for breach of contract, judgment should be for whom? (A) The man, because the offer to the neighbor terminated when the neighbor learned of the sale to the brother. (B) The man, because there was no consideration to keep the offer open for an extended period of time. (C) The neighbor, because the offer became irrevocable for a reasonable time when the man allowed the neighbor to "think it over." (D) The neighbor, because he is a merchant.

(A) The man, because the offer to the neighbor terminated when the neighbor learned of the sale to the brother. An offeree's power of acceptance is terminated when the offeror takes definite action inconsistent with an intention to enter into the proposed contract and the offeree acquires reliable information to that effect. Thus, the man's offer was effectively revoked when the neighbor learned from a reliable source that the barbecue had been sold to the man's brother.

A man decided to take his own life and went to the roof to jump to the street below. On his way up the stairs, he told his neighbor what he was planning to do. The neighbor had a strong belief that anyone who committed suicide would not enter Heaven. Therefore, he snuck up behind the man as he was readying to jump off the rooftop. After the man jumped, the neighbor shot him once in the back before the man struck the sidewalk below. At trial, the medical examiner testified that the man died of a gunshot wound, and that in all likelihood, the fall would not have killed the man. The jury also found that the neighbor's religious belief was honestly held. If the neighbor is prosecuted for murder, what should the result be at trial? (A) The neighbor should be convicted, regardless of the man's intent or the medical examiner's testimony. (B) The neighbor should be convicted, because the medical examiner testified that the fall would not have killed the man. (C) The neighbor should be acquitted, because the man intended to kill himself. (D) The neighbor should be found guilty of a lesser charge, because the jury believed that his religious belief was honestly held.

(A) The neighbor should be convicted, regardless of the man's intent or the medical examiner's testimony. Murder requires malice. Malice can be express or implied. Here, there is express malice, because the defendant intentionally took the life of another. Regardless of whether the man would have died or not otherwise is irrelevant, because it was the neighbor's act that caused the man's death. Moreover, the fact that the neighbor was acting in what he thought was the best interest of the victim is also irrelevant.

A police officer has long been suspicious of one of his neighbors. Every time he encounters her, she behaves furtively and seems nervous. Convinced that there is a reason for the neighbor's edginess, the officer decides to investigate further. He knows that the neighbor lives alone, so when she leaves the house with friends one evening, he walks into the open garage and tries the door to the house. Finding it unlocked, he enters the house. In plain view on the coffee table is a large quantity of cocaine and a clear plastic bag containing cash. Looking further, the officer finds records tying the neighbor's sister to the drug operation. What is the most likely outcome of the neighbor's and her sister's motions to suppress the evidence? (A) The neighbor's motion will be granted, but the sister's will not. (B) The sister's motion will be granted, but the neighbor's will not. (C) Both motions will be granted as to the records, but not as to the items that were in plain view. (D) Neither motion will be granted.

(A) The neighbor's motion will be granted, but the sister's will not. The search was clearly illegal in that the officer did not obtain a warrant and there were no exigent circumstances or other applicable exceptions to the warrant requirement. Thus, the neighbor's motion to suppress all of the evidence obtained during the search of her house will be granted. The neighbor's sister, however, has no standing to challenge the search of the neighbor's house, and, therefore, the items will not be excluded as against he

While on routine patrol, police officers observed a driver making an illegal U-turn. After stopping the driver's car, they saw him reach under the driver's seat. They ordered him out of the car and saw a cellophane package protruding from under the seat. The package contained a white powdery substance, which the officers suspected to be cocaine. The officers placed the driver under arrest and put him in the rear of their patrol car. They then proceeded to search the rest of the driver's car. In the trunk they found an assault rifle that was later determined to be the weapon used in a liquor store robbery. Charged with that robbery, the driver moved to suppress the assault rifle as evidence on the grounds that the police did not have a warrant to search the trunk. What is the best theory that the prosecution can use in support of the admissibility of the assault rifle as evidence? (A) The police conducted an automobile search. (B) The police conducted an inventory search. (C) The search was incident to a lawful arrest. (D) The search was made under exigent circumstances.

(A) The police conducted an automobile search. This question deals with the highly tested area of warrantless searches. A warrantless search and seizure of items from an automobile may be permitted where there is probable cause to believe the vehicle contains contraband (or where the vehicle could be moved before there is time to obtain a warrant). In accordance with the holding in United States v. Ross [456 U.S. 798 (1982)], once probable cause to search exists, the police can search the entire vehicle, including closed containers. Thus, this is the most helpful theory to the admission of the rifle.

A developer owned a strip mall in a busy area of the city, and leased the storefronts to various businesses pursuant to year-to-year periodic lease agreements. Deciding to retire, the developer sold his interests to a business investor and assigned all of his lease agreements to the investor. None of the written lease agreements contained clauses addressing assignments. Which of the following is true? (A) The developer remains the landlord under the terms of the leases, because the assignments are invalid. (B) The investor is now the landlord under the terms of the leases, because a landlord may assign his lease interest. (C) The investor is the landlord under the terms of the leases, because the leases were commercial. (D) The leases terminated upon the developer's attempted assignment.

(B) The investor is now the landlord under the terms of the leases, because a landlord may assign his lease interest. Only an at-will tenancy terminates when one of the parties attempts to assign his or her interest. By contrast, periodic leases, such as the year-to-year leases in this case, do not terminate upon a party's assignment. As such, absent a lease term to the contrary, it was permissible for the developer, as landlord, to assign his lease interests to the business investor, who then became the new landlord under the lease agreements.

A supermarket had just reopened after a six-month renovation period. The renovations included the repair, replastering, and repainting of the entire ceiling and walls by the painting company. The day following the reopening of the supermarket, the store's manager noticed small fragments of plaster on the floor, which appeared to have fallen from a part of the ceiling about 10-square-feet in area. The manager immediately posted signs in that area that read: "Caution: Falling Plaster." In addition, the manager promptly called the painting company and requested a repairman to attend to the problem. That afternoon, a shopper was shopping in the supermarket and wanted to purchase some canned goods in the posted area. Moving quickly, the shopper reached for the desired items. At that very moment, a section of the ceiling fell and struck the shopper, injuring her very seriously. If the shopper asserts a claim against the supermarket, what is the likely outcome? (A) The shopper will prevail, because the supermarket failed to take adequate precaution to ensure the safety of its patrons against such an unreasonable risk of harm. (B) The shopper will prevail, because the supermarket is vicariously liable for the negligence of its independent contractor in the performance of a nondelegable duty. (C) The shopper will not prevail, because the injury was the fault of the painting company. (D) The shopper will not prevail, because the supermarket posted signs warning customers of the risk of harm.

(A) The shopper will prevail, because the supermarket failed to take adequate precaution to ensure the safety of its patrons against such an unreasonable risk of harm. A possessor of land is required to exercise reasonable care to protect invitees, such as customers in a store, from harm. This is a nondelegable duty, meaning that the possessor will be obligated to exercise reasonable care, even where the danger is created by the acts of an independent contractor doing work on the property. Although giving a warning about a danger may be sufficient in some circumstances, reasonable care may require the possessor of the land to do more than just post a warning sign. Here, although the manager posted a warning sign in the potentially dangerous area of the supermarket, more adequate precautions should have been taken under the circumstances. Consequently, the shopper would be entitled to recover damages in negligence against the supermarket.

The state legislature enacted a statute requiring permanent removal from parental custody of any child born to a mother currently receiving state welfare payments. A young mother recently lost her job and had been receiving state welfare assistance. She now has given birth to her second child and the state brought an action under the statute to terminate her custody rights solely because she was receiving the state welfare payments. Her defense is based on the ground that the statute is unconstitutional as applied. Which is the most probable burden of persuasion on this constitutional issue? (A) The state has the burden of persuading the court that the application of the statute to the mother is necessary to vindicate a compelling government interest. (B) The state has the burden to persuade the court that the application of the statute to the mother is rationally related to a legitimate government interest. (C) The mother has the burden of persuading the court that the application of the statute to her conduct is not necessary to vindicate an important state interest. (D) The mother has the burden of persuading the court that the application of the statute to her conduct is not rationally related to a legitimate state interest.

(A) The state has the burden of persuading the court that the application of the statute to the mother is necessary to vindicate a compelling government interest. When a fundamental right is infringed, the government has the burden of showing that the infringement is necessary to advance a compelling government interest. Here, the mother is threatened with the denial of custody of her child because of her receipt of welfare payments. The Supreme Court has recognized a fundamental right to live with the members of your own family [Moore v. City of East Cleveland, 431 U.S. 494 (1977)]. When the government infringes on a fundamental right, it has the burden to show that its action is necessary to achieve a compelling government interest. This choice correctly states this rule.

Concerned by the number of large commercial properties being purchased by foreign nationals, the legislature of State Red enacted a statute prohibiting the sale of commercial properties in excess of 10,000 square feet to aliens. A resident alien who has lived in the State Red for several years enters into a contract with an investor to buy a commercial printing business in the industrial zone of State Red's capital city. The square footage of the physical plant is 23,000. The resident and the investor decide to test the constitutionality of the State statute, and to that end, they seek a declaratory judgment in federal court. Which of the following statements regarding the declaratory judgment action is most accurate? (A) The state has the burden of showing a compelling state interest in carrying out the statute's purposes regarding large commercial properties. (B) The resident has the burden of showing that the state has no compelling interest in carrying out the statute's purposes regarding large commercial properties. (C) The investor has the burden of showing that there is no rational basis for the statute's purposes regarding large commercial properties. (D) The court will find that the resident, but not the investor, has standing.

(A) The state has the burden of showing a compelling state interest in carrying out the statute's purposes regarding large commercial properties. When a state statute discriminates against a suspect class such as aliens, the burden is on the state to show that its actions are necessary to achieve a compelling state interest. In this case, the State Red statute may violate the Equal Protection Clause by prohibiting aliens from acquiring large commercial properties. The federal court hearing the resident and the investor's case will apply the strict scrutiny test to the statute to determine whether the law violates the constitutional guarantee of equal protection.

A resident alien was recently certified to be a teacher by a state regulatory agency. She applied to several schools that were hiring in her district, but all of them refused to hire her, citing a state law prohibiting non-citizens from teaching in a public school. The woman contacted the schools and was told that, if not for the state law, they would have hired her. The woman sued the state to challenge the constitutionality of the law prohibiting non-citizens from public school teaching. What is the most likely outcome? (A) The statute is constitutional, because public school education involves the functioning of government. (B) The statute is constitutional, because the exclusion is rationally related to legitimate state purposes. (C) The statute is unconstitutional, because the action of the state violates the Privileges and Immunities Clause of Article IV of the Constitution. (D) The statute is unconstitutional, because there is not sufficient justification to discriminate against aliens as a class.

(A) The statute is constitutional, because public school education involves the functioning of government. The state may discriminate against aliens only in specific matters relating to the governmental process, such as the denial of a right to run for elective office/to hold a position as a police officer/probation officer/a public school teacher. In all other cases, a state's attempt to discriminate against persons based on alienage will be subject to the strict scrutiny test, because alienage is a suspect classification.

A landlord and a tenant lived together in an apartment as roommates. The lease agreement between the two parties stated that the tenant would have access to all of the apartment's common areas, including the living room, bathroom, and kitchen. Believing that the tenant had damaged the television in the living room, the landlord removed all of the tenant's personal property from the living room, locked the door to the living room, and told the tenant that the tenant would no longer have access to that room. Although the tenant objected, the tenant continued to reside in the other rooms of the apartment. To what relief is the tenant entitled under the law of most jurisdictions? (A) The tenant is relieved of all liability for rent, because he has been actually evicted. (B) The tenant is relieved of all liability for rent, because he has been constructively evicted. (C) The tenant is only entitled to a rent abatement, because he has been partially evicted. (D) The tenant is only entitled to a rent abatement, because he has been constructively evicted.

(A) The tenant is relieved of all liability for rent, because he has been actually evicted. In the scenario presented, the tenant has been actually evicted from a part of the premises but continues to occupy the rest of the premises. Under the majority rule, the tenant is relieved of all liability for rent under these circumstances. Note that the Second Restatement of Property rejects this rule and calls for a rent abatement only.

A tenant leased two floors of an office building from a landlord for a 10-year period, with rent due at the start of each month. According to the terms of the lease, the landlord agreed to perform routine maintenance, including but not limited to repairing and painting the walls, replacing carpeting, and maintaining the air conditioning and electrical systems, at the start of each year, after the tenant submitted an itemized list of requests. At the start of the third year of the lease, the tenant submitted the list to the landlord. However, the landlord, after seeing the costs incurred in doing the repairs in the second year, refused to do them. What are the tenant's remedies? (A) The tenant may make the repairs himself and offset his subsequent rental payments by the costs incurred. (B) The tenant may not make the repairs, because doing so would constitute an ameliorative waste. (C) The tenant may refuse to pay rent due to the landlord's breach of covenant. (D) The tenant may treat the lease as terminated if he vacates the premises.

(A) The tenant may make the repairs himself and offset his subsequent rental payments by the costs incurred. A tenant may seek money damages for a landlord's breach of a term or covenant in the lease. Traditionally, a tenant may vacate the premises and terminate the lease if the tenant has been evicted by the landlord, actually or constructively. Today, statutes in many jurisdictions provide the tenant with the following statutory remedies: (1) withholding rent; or (2) repairing the premises and deducting the cost of repair from subsequent rent payments. Therefore, under the circumstances, this is the best answer choice.

A landlord entered into a lease agreement with a tenant. The jurisdiction in which the leased premises were located had enacted a statute requiring landlords to maintain a temperature of at least 65 degrees Fahrenheit in leased apartments during the winter months. The lease agreement itself contained no such requirement. In January of the first year of the lease, the temperature in the apartment was no higher than 58 degrees. The tenant vacated the apartment and sued the landlord for violating the lease's covenant of quiet enjoyment. What remedies are available to the tenant in most jurisdictions? (A) The tenant may treat the lease as terminated and withhold rent, because he has been constructively evicted. (B) The tenant is entitled only to a rent abatement, because he has been constructively evicted. (C) The only remedies available to the tenant are those provided for in the statute, because the landlord has not violated the lease. (D) The only remedies available to the tenant are those provided for in the statute, because the tenant vacated the apartment.

(A) The tenant may treat the lease as terminated and withhold rent, because he has been constructively evicted. A constructive eviction exists when a landlord acts to substantially and permanently interfere with the tenant's use and enjoyment of the premises and the tenant moves out. Withholding something required by statute constitutes constructive eviction. Withholding something essential to the full enjoyment of the lease that is included within the terms of the lease also constitutes constructive eviction. In this example, while a requirement to provide sufficient heat was not included within the terms of the lease itself, it was required by statute. Therefore, the landlord's failure to properly supply heat constitutes a constructive eviction. In most jurisdictions, a tenant who has been constructively evicted may treat the lease as cancelled and withhold the entire rent.

A university leased a two-story house near the campus to a newly hired professor for a term of five years. According to the terms of the lease, assignments and subleases were prohibited, and the professor agreed to pay $500 per month in rent. The professor moved in with his family and lived there for two years. After two years, the professor had a new child and decided that he needed more space. The professor moved out and assigned the lease to a student. The student agreed to pay the university the $500 monthly rent. The university did not object to the assignment. One year later, the student subleased the top floor of the house to a friend for $250 per month, which the friend paid to the student. The friend paid rent for the first six months, but then stopped. The next month, the student paid the university $250 and notified them that they should collect the remainder of the rent from the friend, who refused to vacate the house. The university instead sued the student to collect the unpaid $250 rent. What is the most probable outcome? (A) The university will recover because the student remains liable. (B) The university will recover because they did not object to the assignment or sublease. (C) The university will not recover, because the friend is the party that is liable. (D) The university will not recover, because the professor remains liable.

(A) The university will recover, because the student remains liable. When a tenant subleases the premises, the sublessee does not come into privity of contract or privity of estate with the landlord. The tenant remains in privity with the landlord, and the sublessee is only in privity with the tenant/sublessor. Therefore, the student remains liable to the university for the unpaid rent, because she is in privity with the university.

A retired policeman was hired as a night watchman by a manufacturer of large construction machinery. During the watchman's orientation, the manager of the factory emphasized that in the event of any problem, the watchman was to keep himself out of danger and call the police immediately. The night watchman also was warned never to go outside the fence while on duty. One night, a drug addict, whom the watchman recognized from his days on the police force, was walking past the yard. In a rare moment of lucidity, the addict also recognized the former police officer. The addict jeered, "Hey! You a rent-a-cop now?" The watchman ignored the addict for a while, but the addict continued to rant loudly and derisively about the watchman's new career. After several minutes of this, the watchman flew into a rage. He went outside the gate and chased the addict down. The watchman punched the addict several times, giving him a concussion. The addict subsequently filed a lawsuit against the watchman and the manufacturer. Which of the following is the strongest argument in the manufacturer's defense? (A) The watchman's act was outside the scope of his employment. (B) The watchman was defending the manufacturer's property. (C) The watchman was acting in self-defense. (D) The watchman was contributorily negligent.

(A) The watchman's act was outside the scope of his employment. Under the doctrine of respondeat superior, an employer is liable for injuries caused by the negligence or strict liability of an employee, if the tortious act occurred within the scope of the employment. Acts are within the scope of employment if they are so closely connected with what the employee was hired to do and so fairly and reasonably incidental to it that they may be regarded as methods, even though improper, of carrying out the objectives of the employment. If an employee uses force, even misguidedly, wholly or partly to further the employer's purpose, such use of force may fall within the scope of employment, resulting in vicarious liability for the employer. However, employers are generally not liable for the intentional torts of their employees. Here, although the watchman was on duty at the time of his assault on the addict, the addict was not threatening the employer or the employer's property. The watchman used force in response to the addict's verbal taunts and was not furthering the employer's purpose in any way when he did so, because the taunts were directed at the watchman personally and were not directed at the manufacturer. Therefore, it is likely that the watchman's act was outside the scope of his employment, and the manufacturer should not be held vicariously liable for the addict's injuries.

While on vacation in Vermont, an Ohio citizen visited an antiques store and saw what was labeled as a packet of Civil War coins. Desperate for a sale, the owner of the store told the Ohio citizen that the coins were rare and worth much more than the $100,000 he was asking for them. However, the woman thought the price was too high. When the woman returned home to Ohio, she received a call from the store owner reducing the price by 20%, prompting the woman to buy the coins. Once she sent the money to the owner, and after receiving the coins, she discovered that the coins were actually worth only a few hundred dollars. The Ohio woman wished to sue the owner for fraud in Ohio. The store owner argued that the court in Ohio lacked jurisdiction. Which of the following is true? (D) The woman cannot sue in Ohio, because the initial conversation took place in Vermont. (A) The woman can sue in Ohio, because the owner's call provided sufficient minimum contacts to support Ohio's exercise of jurisdiction over him. (C) The woman cannot sue in Ohio, because the man is not subject to personal jurisdiction in Ohio based on a single phone call. (B) The woman can sue in Ohio, because the amount in controversy is more than $75,000.

(A) The woman can sue in Ohio because the owner's call provided sufficient minimum contacts to support Ohio's exercise of jurisdiction over him. Although the owner did not physically enter the state, he would have sufficient minimum contacts via his directly reaching into the state to make contact with the woman in order to conduct business. Had the sale occurred entirely at the shop, and the owner merely followed up with a call, or the woman had a change of mind and called the owner, those situations would not have provided sufficient minimum contacts. However, the call here was made directly to the woman for the purpose of getting her to make a purchase. That is purposeful availment sufficient to establish personal jurisdiction.

A man negligently hit a woman with his car. The woman was rushed to the hospital and sent into surgery. During the course of the operation, the doctor was careless and caused the woman more injuries. If the woman sued the man, which of the following statements is most accurate? (A) The woman should recover for all of her injuries, because they were foreseeably caused by the man. (B) The woman should recover for all of her injuries, because the thin-skulled plaintiff rule provides that a defendant takes their victim as they find them. (C) The woman should only recover for those injuries sustained in the accident, and she must sue doctor as a separate tortfeasor for the damages sustained during surgery. (D) The woman should only recover for those injuries sustained in the accident, because it is not foreseeable that a doctor would be negligent.

(A) The woman should recover for all of her injuries, because they were foreseeably caused by the man. In situations like this, the doctor's carelessness represents an intervening act. However, it is considered foreseeable that a plaintiff would need medical care as a result of a defendant's negligence. It is also considered foreseeable that the medical care may be administered carelessly. Therefore, the defendant would be liable for both the harm he inflicted on the plaintiff himself and for the harm the plaintiff suffered in the hospital.

A State A plaintiff sued two State B defendants in State A. The plaintiff's claim against each defendant exceeded $100,000. Ten days after being served, the defendants were served notice that the plaintiff had added a third defendant, who was from State A. All three defendants agreed to remove the case and did so within the next 10 days. The plaintiff successfully challenged the removal in federal court and the case was remanded back to State A court. Forty days later, the plaintiff settled her case against the State A defendant. At this point, what recourse do the remaining defendants have? (D) They cannot remove, because although they are not time-barred, removal can only occur once. (A) They can remove the case immediately, because they have 30 days from when the grounds for removal become apparent. (B) They can remove the case immediately because this is a diversity case and the date for removal was within one year of when the suit was filed. (C) They cannot remove, because they are time-barred.

(A) They can remove the case immediately, because they have 30 days from when the grounds for removal become apparent. The defendants can remove after the third defendant has been dropped from the suit. A party can remove within 30 days of when the grounds for removal become apparent, and in diversity cases, removal must always be within one year of the date when the suit was filed. Once the plaintiff settled with the nondiverse defendant, the remaining defendants now had grounds for removal, and had 30 days in which to do so.y cannot remove, because they are time-barred.

A pedestrian, who was walking along Chestnut Street at 10:20 p.m. on the night of December 3, urgently needed to find a restroom. Just ahead, the pedestrian noticed a private dinner club. As the pedestrian approached the club, he noticed a sign over the front door that read: "ADMITTANCE TO MEMBERS ONLY." Although the pedestrian was not a member of the exclusive club, he entered the dimly lit club, found a door marked "Gentlemen," and entered the restroom. Which of the following would best describe the pedestrian's legal status when he was in the restroom? (A) Trespasser. (B) Guest. (C) Licensee. (D) Invitee.

(A) Trespasser. A trespasser is someone who enters land without the owner's express or implied permission. In the present case, the pedestrian would be classified as a trespasser because he entered the premises without the club's permission.

Investigating a 35-year-old suspect believed to be trafficking in child pornography, police detectives arrived at the suspect's house, hoping to interview her. An old man answered the door. He identified himself as the suspect's father and told them that the suspect was not at home. The police asked if they could come in and speak with the father, who let them in. While in the home, the police asked to search for evidence of child pornography. The horrified father again agreed and took the police to the suspect's locked upstairs bedroom. He told them that he never went inside the room because only the suspect had the key. With the father's consent, the police broke down the door to the bedroom. The police searched the bedroom and found hundreds of images of child pornography inside. While the police were searching, the suspect returned home and was arrested. Have the police violated the suspect's Fourth Amendment rights? (A) Yes, because the father could not consent to the search of the bedroom. (B) Yes, because the images were not in plain view. (C) No, because the images were part of a search incident to an arrest. (D) No, because the father gave valid consent to the search of the shared premises.

(A) Yes, because the father could not consent to the search of the bedroom. The general rule for consent to shared premises is that any person who has joint control of the shared premises may consent to a valid search, and any evidence obtained may be used against the other occupants. However, the power to consent applies to common areas only and not to private areas where the defendant has exclusive control. In this case, the father had the power to consent to the common areas of the house. He did not have the power to consent to a search of the locked bedroom, over which the suspect had exclusive control. The warrantless search was thus unreasonable under the Fourth Amendment and violated the suspect's Fourth Amendment rights.

A dog breeder lost her prized golden retriever puppy during a walk in a nature preserve. To enlist help in finding her dog, the breeder posted a notice on the internet and offered to pay $1,000 for the return of her puppy. A hiker saw the notice, equipped herself, drove to the nature preserve, and subsequently found the puppy. However, before the hiker could return the puppy, the breeder heard that the dog had been found and posted another notice on the internet withdrawing the reward offer. The hiker brought the breeder the dog and demanded the reward. The breeder refused. Is the breeder liable to pay the hiker the reward? (A) Yes, because the hiker had undertaken substantial steps toward acceptance of the breeder's offer before it was revoked. (B) Yes, because the hiker did not actually see the breeder's purported revocation of her reward offer. (C) No, because the hiker did not communicate her acceptance of the breeder's offer. (D) No, because the breeder revoked the offer before completion of the hiker's performance.

(A) Yes, because the hiker had undertaken substantial steps toward acceptance of the breeder's offer before it was revoked. A unilateral contract is created when an offer seeks performance rather than a promise in return. An offer in a unilateral contract cannot be revoked once performance has begun. This answer is correct because the hiker undertook substantial efforts towards acceptance by performance of the breeder's offer to enter into a unilateral contract. All that remained was delivery of the puppy to the breeder. Therefore, the breeder cannot revoke her offer to enter into a unilateral contract once the hiker began performance.

A tenant leased an apartment from a landlord pursuant to a five-year lease. One afternoon, a group of boys including the landlord's son were playing in the street outside the apartment. One of the boys kicked a soccer ball into the front screen door. As a result, the screen door was partially crushed and had a small hole in the area where the ball had hit it. The tenant requested that the landlord repair the broken door, but the landlord refused. The tenant then replaced the door for $150 and deducted the $150 from the next month's rental payment. The lease agreement did not address which party would be responsible for making repairs to the apartment. Under the common law rule, will the landlord prevail in an action to recover the $150 against the tenant? (A) Yes, because the lease did not specify that the landlord had the burden of making repairs. (B) Yes, because although responsibility for repairing the door lies with the landlord, the tenant is not entitled to reimbursement if the tenant chooses to repair the door himself. (C) No, because the broken door constitutes a partial, constructive eviction. (D) No, because deduction of the cost of repairs was the appropriate remedy for breach of the warranty of habitability.

(A) Yes, because the lease did not specify that the landlord had the burden of making repairs. Under the common law rule, in the absence of contractual terms to the contrary, a tenant has a duty to make minor repairs to keep the leased premises wind and water tight. This responsibility arises from the tenant's duty not to commit waste. In this case, given that the lease did not state which party would be responsible for making repairs, the default common law rule applies, and the tenant is responsible for repairing the door. Since liability for the broken door lies with the tenant, the tenant may not deduct the cost of making the repair from the rent payment. Of course, the tenant may recover from the boy who actually damaged the door but fault is a red herring in this question.

The United States Post Office received an anonymous tip that heroin would be found inside a large black hat box sent to a local address from Panama. Pursuant to this tip, United States postal employees opened the hat box bound for the local address and found heroin inside. They then resealed the package and left the heroin inside. The local police were notified and watched as the package was delivered to the address. The police then secured a warrant to search the house for the package. About two hours after the package was delivered, the warrant was executed. The man who opened the door was arrested, and the officers found the package unopened, sitting on the kitchen counter. After seizing the package, the officers looked through the rest of the house. In an upstairs cedar chest, they found a similar hat box that also contained heroin. At trial, the man brings a motion to suppress the heroin found in the cedar chest. Should the court grant the motion? (A) Yes, because the search exceeded the authority granted by the warrant. (B) Yes, because the initial search by the United States Post Office was without probable cause. (C) No, because, having found the package, the officers had probable cause to believe more narcotics could be located in the house. (D) No, because the heroin was found pursuant to a valid protective sweep.

(A) Yes, because the search exceeded the authority granted by the warrant. The warrant was valid, but its validity was triggered by and limited to the delivered package [United States v. Grubbs, 547 U.S. 90 (2006)]. Accordingly, once the only object of that search was discovered, the warrant did not authorize a further exploratory search of the house [Horton v. California, 496 U.S. 128 (1990)].

The police had, over time, accumulated reliable information that the defendant operated a large cocaine-distribution network, that he and his accomplices often resorted to violence, and that they kept a small arsenal of weapons in his home. One day, the police received reliable information that a large, brown suitcase with leather straps containing a supply of cocaine had been delivered to the defendant's home, and that it would be moved to a distribution point the next morning. The police obtained a valid warrant to search for and seize the brown suitcase and the cocaine, and went to the defendant's house to execute it. Upon executing the warrant, the police encountered the defendant in the kitchen holding the suitcase. They seized the suitcase and put handcuffs on the defendant. A search of his person revealed a switchblade knife and a .45-caliber pistol. The police then searched the rest of the house, but found no other people. However, one officer found an Uzi automatic weapon in a box on a closet shelf in the defendant's bedroom. The defendant was subsequently charged with various crimes. The defendant filed a pretrial motion to suppress the use of the Uzi as evidence. Should the defendant's motion be granted? (A) Yes, because the search exceeded the scope needed to find out if other persons were present. (B) Yes, because once the object of the warrant had been found and seized, no further search of the house was permitted. (C) No, because the police were lawfully in the bedroom and the weapon was immediately identifiable as being subject to seizure. (D) No, because the police were lawfully in the house and had probable cause to believe that weapons were present.

(A) Yes, because the search exceeded the scope needed to find out if other persons were present. If a search of a home is made pursuant to a valid warrant, then the police may make a protective sweep, which is limited to a cursory search of the premises to assure that they do not contain people who might endanger the officers' safety. Officers performing the sweep must actually believe that dangerous people are on the premises, based on specific and articulable facts. Police must have objective evidence that the house contains someone who poses a danger to those present during the arrest of the defendant. The evidence will be suppressed in this instance because the Uzi was found in a box in the closet, and the police exceeded their permitted scope of a protective sweep by looking there.

A bookie was arrested and charged with the murder of a gambler. The bookie lived in New York City. The gambler, who resided in Philadelphia, was found shot to death in New York City. During the bookie's murder trial, the prosecution called the gambler's sister to testify. She proposed to testify that the day before the gambler was killed, he told her, "I intend to take the train to New York City tomorrow and pay my bookie the money I owe him." Is the sister's testimony admissible? (A) Yes, to show the gambler's intent. (B) Yes, as a verbal act. (C) No, because the gambler is not subject to cross-examination. (D) No, as hearsay not within any recognized exception.

(A) Yes, to show the gambler's intent. Federal Rule of Evidence 803(3) provides a hearsay exception, without regard to the availability of the declarant, for a statement of the declarant's then-existing state of mind, emotion, sensation, or physical condition, including intent, plan, motive, design, or mental feeling. Such statements are admissible substantively to prove the declarant's state of mind when that state of mind is at issue. The gambler's statement will be admissible to show his state of mind the day before he was killed. Therefore, this answer choice is correct.

Following a three-car accident, a woman sued a man for negligence, claiming that she suffered damages of $100,000. The driver of the third car was uninsured and the woman chose not to join him as a party. At trial, the jury found that the woman was 30% at fault, the man was 40% at fault, and the third driver was 30% at fault. The jury agreed that the woman's damages were worth $100,000. How much will the woman be able to recover from the man? (A) $100,000, because pure comparative negligence allows for recovery regardless of the plaintiff's negligence, and the man can sue the third driver. (B) $70,000, because a negligent plaintiff's recovery should be reduced by her percentage of fault. (C) $40,000, because that is the man's percentage of fault. (D) Nothing, because her negligence was equal to the third driver's negligence, precluding her recovery.

(B) $70,000, because a negligent plaintiff's recovery should be reduced by her percentage of fault. The woman would only be able to recover $70,000 (or $100,000 reduced by the 30% of fault attributed to the woman's own negligence), because this is a pure comparative fault jurisdiction. Because the two other drivers would be jointly and severally liable, the woman can recover the full amount of her damages from the man, who would then sue the third driver for contribution for the amount that he paid over his own personal percentage of fault.

While on vacation, two friends went scuba diving in the ocean. Each of them was equipped with a spear gun, which they planned to use to kill tuna. As they were swimming, both men saw what appeared to be a large fin entangled in a nearby coral reef. Simultaneously, they shot their spear guns at the projecting wing-like figure. Unbeknownst to either of the friends, it was not a fish but rather another scuba diver. The victim was struck and slightly wounded by one of the spears. The victim asserts a claim for damages against the two friends. At trial, how should the judge instruct the jury to rule? (A) In favor of both defendants, if no evidence is presented showing who actually shot the spear that injured the victim. (B) Against both defendants jointly, unless one of them proves that he did not shoot the spear that struck the victim. (C) Against each defendant for one-half of the amount of damages, because they both acted independently and not jointly. (D) Against each defendant for one-half of the amount of damages, because they both shot their spear guns simultaneously.

(B) Against both defendants jointly, unless one of them proves that he did not shoot the spear that struck the victim. A plaintiff is generally required to identify the person who was the cause of the plaintiff's injury. However, the doctrine of alternative liability or alternative cause liability can assist a plaintiff unable to prove which one out of a group of defendants caused the plaintiff's injury. This doctrine applies where the plaintiff sues a group of defendants and shows that every one of the defendants engaged in the same tortious conduct, one of the defendants caused the plaintiff's injury, and the plaintiff is unable to determine which one was the cause. The burden of proof on the issue of who actually caused the harm will be shifted to the defendants. If a defendant can prove that it was not the cause, that defendant will avoid liability. However, defendants who cannot exculpate themselves (i.e., cannot prove they were not the cause) will be held jointly and severally liable for the plaintiff's injury. This is the rule enunciated in the case of Summers v. Tice [199 P.2d 1 (Cal. 1948)], where two hunters negligently fired shotguns in a plaintiff's direction. One of the hunters hit the plaintiff in the eye, but the plaintiff could not identify which hunter fired the shot that hit his eye. The plaintiff sued both hunters, and the court held that the hunters would be jointly and severally liable unless they could prove who actually caused the plaintiff's injury. This choice is, therefore, correct.

A man and a woman were involved in a car accident that occurred when the man abruptly switched lanes in front of the woman without signaling. Unable to stop, the woman hit the man's car into a median. Immediately after the accident, as the two drivers emerged from their vehicles, a pedestrian, obviously emotional and upset from having just observed an accident, ran up to the man and shouted, "You careless driver, you! Didn't you ever learn that you're supposed to use a turn signal when you switch lanes?" The pedestrian was never identified and is not present at trial. How should the court rule on the pedestrian's statement? (A) Admissible, as non-hearsay. (B) Admissible, even though it is hearsay. (C) Admissible, under the excited utterance exception. (D) Inadmissible, because the bystander cannot be identified and is not present at trial.

(C) Admissible, under the excited utterance exception. Under FRE 803(2), a statement relating to a startling event or condition made while the declarant was under the stress of excitement caused by the event or condition is not excluded by the hearsay rule. The theory behind the excited utterance exception is simply that circumstances may produce a condition of excitement that temporarily stills the capacity of reflection and, thus, produces utterances free of conscious fabrication. Spontaneity is the key factor in determining whether the statement was, indeed, made without any conscious reflection.

A landlord and a business executive entered into a written lease agreement for a penthouse apartment for a period of five years, with rent to be paid monthly. Two years later, the executive was reassigned to an office in another state for one year. The executive leased the apartment to a coworker for the duration of his reassignment. The coworker promptly moved in, but never paid any rent. After nine months, the coworker was fired, and he moved out of the apartment and back in with his parents. Upon returning home from his reassignment, the executive had barely unpacked before he was promptly served with a lawsuit filed by the landlord against both the executive and the coworker for the previous year's unpaid rent. Assuming the landlord is successful, how will the court rule regarding damages? (A) Against the coworker individually for the full year's rent, and no judgment against the executive. (B) Against the executive for the full year's rent, and no judgment against the coworker. (C) Against the executive individually for three months' rent, and against the coworker individually for nine months' rent. (D) Against the executive and coworker for the full year's rent, because they are jointly and severally liable.

(B) Against the executive for the full year's rent, and no judgment against the coworker. A sublease occurs when a tenant transfers to a third person (the subtenant, or sublessee) less than all of his rights, title, and interest in the leased premises. A subtenant does not come into privity of estate with the original landlord. Therefore, absent an express assumption of the duty to pay rent by the subtenant, the landlord may not sue the subtenant directly for unpaid rent. Here, the coworker was a sublessee, because his lease was for only one of the three years remaining on the lease term. As such, the landlord has no cause of action against the coworker, and must instead recover all of the unpaid rent from the original tenant, the business executive.

After driving his car up a curb and hitting a streetlight, an accident for which he was found to have been at fault after a trial, a 70-year-old retiree had his driver's license automatically revoked pursuant to a state law requiring all drivers over the age of 65 who are involved in a serious accident to have their licenses revoked until they complete a safety course and pass another driving test. The retiree was a handyman who now volunteered by going to the homes of people who could not afford to hire someone to make various repairs. The man was now unable to help these indigent people due to the revocation of his license. He brought suit against the state challenging the automatic revocation of his license without his first having an opportunity to present evidence on the accident to the state's department of motor vehicles. How should the court rule? (A) Against the retiree, because he is now retired. (B) Against the retiree, because he received adequate due process. (C) For the retiree, because a pre-termination hearing was required before his license could be revoked. (D) For the retiree, because his Fourteenth Amendment rights were violated.

(B) Against the retiree, because he received adequate due process. The deprivation of a driver's license is a property interest protected by the Fourteenth Amendment [Bell v. Burson, 402 U.S. 535 (1971)]. However, the Supreme Court has found that there is a substantial public interest in administrative efficiency which would be impeded by the availability of a pre-termination hearing in every case. Especially when there is an opportunity for the validity of the plaintiff's fault to be determined in a prior setting, the automatic suspension or revocation of a driver's license does not violate a plaintiff's procedural due process rights when weighed against the important public interest in safety on the roads and highways, and the prompt removal of a safety hazard [Dixon v. Love, 431 U.S. 105 (1977)]. As such, the man's procedural due process rights were not violated in this case.

On December 1, Arnold, who had purchased two tickets for a holiday magic show, sent the following email to his three friends, Baker, Charlie, and Dora: "Dear Baker, Charlie, and Dora: I have two tickets for the upcoming holiday magic show. If any of you are interested in buying my tickets, I will let you have them for a reasonable price, but I must have your reply by December 20. s/Arnold" The tickets, which cost Arnold $10, were now selling for between $50 and $70. On December 19, Arnold received an email from Baker that stated, "I accept your offer and will pay you $50 for the two tickets." Arnold did not immediately respond to Baker's email. On December 22, Arnold received the following text message from Charlie: "I am dying to go to the concert...will pay $70 for your seats." The next day, Arnold sent a reply text to Charlie which read: "The tickets are yours. You may take delivery upon payment of the $70." Dora did not respond to Arnold's letter. Baker tendered the $50 to Arnold within a reasonable time, but Arnold refused to sell the tickets to him. In an action by Baker against Arnold for breach of contract, judgment should be for whom? (A) Arnold, because his email of December 1 and Baker's reply were too indefinite to constitute an offer and acceptance. (B) Arnold, because his email was not an offer, but rather, an invitation to bargain, and he was free to accept either Baker or Charlie's offer. (C) Baker, because he accepted Arnold's offer first. (D) Baker, because Arnold's offer lapsed on December 20, and therefore, Charlie's attempted acceptance was invalid.

(B) Arnold, because his email was not an offer, but rather, an invitation to bargain, and he was free to accept either Baker or Charlie's offer. An offer is the manifestation of a willingness to enter into a bargain, so made as to justify another person in understanding that his assent to that bargain is invited and will conclude it. Arnold's email to three people does not qualify as an offer. An offer must evidence a commitment by the offeror that he reasonably intends to be bound upon acceptance by the offeree, rather than a mere statement of present intention or preliminary negotiations. Arnold's email stated, "I will let you have [the tickets] for a reasonable price." This language evidences an intent to invite a reasonable offer by the message's recipients. Therefore, Arnold's email was not an offer. On the other hand, Baker's email was an offer, which Arnold chose not to accept. Charlie's text was also a valid offer and, in this case, Arnold accepted it upon dispatch of his email on December 23. Therefore, Baker will not prevail against Arnold.

HoagieWorld franchised thousands of sandwich shops nationwide. Ten years ago, a franchisee signed a franchise agreement with HoagieWorld granting him the right to operate five franchises in State A. Believing that the franchisee breached his original agreement requiring him to purchase all of his meat, condiments, and bread from approved vendors, HoagieWorld filed a request for a permanent injunction in federal court. In its complaint, HoagieWorld correctly identified the franchisee as a citizen of State A and itself as a citizen of State B, where it was incorporated, and State C, where it had its primary place of business. In determining the cost of the injunction, HoagieWorld focused on the alleged improper use of the franchise and reputational damage suffered by the brand and valued the stopping of that improper use at $90,000. The franchisee calculated the injunction's cost to be $5,000. He derived this figure from the increased yearly costs of using approved vendors, rather than local, less expensive vendors. The franchisee filed a motion to dismiss the case, claiming that the federal court lacked subject-matter jurisdiction to hear the case. How should the court rule on the motion? (A) Deny the motion, because federal courts have exclusive jurisdiction to hear cases involving copyright violations. (B) Deny the motion, because there is a sufficient amount in controversy. (C) Grant the motion, because the case deals with breach of contract, not a federal question. (D) Grant the motion, because HoagieWorld's $90,000 figure associated with reputational damage was too speculative.

(B) Deny the motion, because there is a sufficient amount in controversy. For diversity jurisdiction, the amount in controversy must be more than $75,000 [28 U.S.C. Sec. 1332]. To determine the value of an injunction for amount-in-controversy purposes, courts look at the larger of two figures: (1) the injunction's worth to the plaintiff; or (2) its cost to the defendant [JTH Tax, Inc. v. Frashier, 624 F.3d 635 (4th Cir. 2010)]. Here, HoagieWorld determined that the value of the injunction was $90,000, which meets the $75,000 requirement.

A defendant was walking through a park when he decided to rob someone. He hid behind a tree, lying in wait for a victim to approach. Shortly thereafter, a girl was strolling in the park when the defendant suddenly jumped from his hiding place and accosted her. Although the defendant intended only to rob his victim, he punched her in the mouth and she fell to the ground. The defendant then grabbed her purse and fled. Unknown to the defendant, the girl suffered a fractured skull when her head struck the pavement. She subsequently died from her head injuries. Which of the following is the most serious crime for which the defendant can be found guilty? (A) Murder. (B) Felony murder. (C) Involuntary manslaughter. (D) Voluntary manslaughter.

(B) Felony murder. As a general rule, one whose conduct brings about a death in the commission or attempted commission of a felony is guilty of felony murder. In many states, the felony-murder rule is limited in its application to serious felonies that must be dangerous to life. Exam tip: These felonies can be remembered by the mnemonic BARRK--burglary, arson, rape, robbery, and kidnapping. Because the girl's death occurred during the commission of a robbery, the defendant would be guilty of felony murder.

A woman from State A had worked for a law school in neighboring State B for 30 years. Each week, she would deposit her check into the savings account she maintained at a State B bank. Upon her retirement, the woman moved to State C, where she died five years later. Her daughter, named as executrix of her estate, discovered that the account with the State B bank still existed. This was a surprise to the State B bank as well, because the account statements sent to the woman at her original State A address had come back as undeliverable for the past five years. The bank never learned that the woman moved to State C until the woman's daughter came forward to notify it of the woman's death. The daughter believes that the bank failed to pay proper interest on the account, and has since sued the bank in State C. The bank claims that it is not subject to personal jurisdiction in State C, as it has no branches there (its only branches are in States A and B) and was not aware that the woman had moved or died. If the bank's assertions are correct, how should the court rule? (B) For the bank, because its contracts with the state are insufficient to support personal jurisdiction. (D) Against the bank, because it is foreseeable that a customer would move to another jurisdiction. (A) For the bank, because it only has branches in State A and State B. (C) Against the bank, because the woman maintained her State B account after she moved to State C.

(B) For the bank, because its contracts with the state are insufficient to support personal jurisdiction. A state cannot exercise jurisdiction over a nonresident defendant if that party has only sporadic and inadvertent contacts with the state. Nonresidents must be protected from the burden of defending a suit in a distant and inconvenient court. In this case, the defendant bank had no office nor transacted business in State C. This cause of action did not arise from acts or transactions in State C. Choice (C) is incorrect as there was nothing the State B bank did to reach out to State C, or even the woman, as they were unaware of her having moved. Choice (D) is incorrect because more is required than just foreseeability of being sued, rather, it is about the foreseeability that the defendant would be haled into court in State C. Based on these facts, it was not foreseeable.

A detective received a tip from an informant, who had given reliable information many times in the past, that a man was a narcotics dealer. Specifically, the informant said that, two months before, he had visited the man's apartment with a friend and that on that occasion he saw the man sell his friend some heroin. The detective knew that the informant, the man, and the friend were acquaintances. Thereafter, the detective put all this information into affidavit form, appeared before a magistrate, and secured a search warrant for the man's apartment. The search turned up a supply of heroin. How will the court likely rule on the man's motion to suppress the introduction of the heroin into evidence? (A) Granted, because a search warrant cannot validly be issued solely on the basis of an informant's information. (B) Granted, because the information supplied to the detective concerned an occurrence too remote in time to justify a finding of probable cause at the time of the search. (C) Granted, because a search for mere evidence alone is improper and illegal. (D) Denied, because the informant had proven himself reliable in the past, and the information he gave turned out to be correct.

(B) Granted, because the information supplied to the detective concerned an occurrence too remote in time to justify a finding of probable cause at the time of the search. The 4th Amendment states, in part, "and no warrants shall issue but on probable cause supported by oath or affirmation." Searches conducted pursuant to a warrant must be based on an adequate and reasonable showing of probable cause when a police officer provides information to a neutral and detached magistrate by affidavit or by testimony under oath. However, when an informant uses an affidavit to provide information, the rigid Aguilar-Spinelli test has been used to establish: (1) probable cause for issuing the warrant; as well as (2) reliability of the informant. More recently, a "totality of the circumstances" approach has been adopted to determine whether there is a "fair probability" or "substantial basis" to conclude that contraband will be found at the particular time and place [Illinois v. Gates, 462 U.S. 213 (1983)]. The fact that the informant saw the man sell heroin two months before is critical because it is too remote in time to justify a present finding of probable cause, thus this answer choice is correct.

The police received a report that women dressed as superheroes had robbed a man in his house and stolen many of his valuables. The only way onto or off the island where he lived was a ferry. The last ferry was getting ready to leave for the night when the report came in to the police. Without attempting to get a warrant, the police stopped all the cars waiting for the ferry. In one vehicle, the police noticed three women fidgeting rather nervously. Upon searching their auto, the police found the victim's stolen property hidden under the front seat. The defendants were immediately placed under arrest. At their trial for armed robbery, the defendants move to suppress the use of the property confiscated by the police as evidence. What should the ruling be? (A) Granted, because the police did not have a warrant to search their car. (B) Granted, because the police did not have probable cause to suspect that their car contained evidence of the crime. (C) Denied, because the police had probable cause to search their car and, under the circumstances, no warrant was required. (D) Denied, because even though the detention was unlawful, the police had reasonable suspicion to believe that their car contained evidence of the crime since they observed the women fidgeting nervously.

(B) Granted, because the police did not have probable cause to suspect that their car contained evidence of the crime. Keep in mind that the warrant requirement is central to the 4th Amendment protection against unreasonable searches and seizures. As a basic rule, all warrantless searches are unconstitutional unless they fall into one of the following seven exceptions to the warrant requirement: (1) search incident to a lawful arrest; (2) the "automobile" exception; (3) plain view; (4) "stop and frisk"; (5) consent; (6) hot pursuit; and (7) other emergencies. A valid warrantless search must meet the requirements of at least one of the above exceptions. With respect to the "automobile" exception, the police must have probable cause to suspect or reasonably believe that the vehicle contains evidence of the crime. In the present example, the police did not have a reasonable or articulable suspicion to believe that the defendants' car contained evidence of the crime. As a result, choice (B) is correct because the police did not have probable cause to conduct a warrantless search of the auto. Choice (A) is wrong because, if the police had probable cause to suspect that the vehicle contained evidence of the crime, then no warrant would have been necessary. Choice (D) is incorrect because the mere fact that the police observed the women "fidgeting nervously" would not by itself give the officers probable cause to suspect that the defendants' car contained evidence of the crime. Choice (C) is incorrect because the police did not have probable cause to search the vehicle, for reasons stated above.

While driving to the grocery store, a 45-year-old man was hit from behind by a car that was being negligently driven by a woman. At the hospital, the doctor indicated that the accident itself should only have caused minor injuries. However, the doctor determined that due to a childhood condition, the man's bones did not fully form properly, and as a result, the injuries he suffered were more severe. In fact, it was determined that the man will have to be placed into traction for one year, and that he would never walk again. To what damages will the man be entitled? (A) He will be entitled to just those damages that can be attributed to the car accident. (B) He will be entitled to all damages he sustained as a result of the accident, because a tortfeasor is liable for the full consequences of the injury she causes. (C) He will be entitled to damages that a normal person would have sustained in a similar accident. (D) He will be entitled to damages for the traction, but not the paralysis, because of the doctrine of avoidable consequences.

(B) He will be entitled to all damages he sustained as a result of the accident, because a tortfeasor is liable for the full consequences of the injury she causes. Under the so-called thin-skulled or eggshell plaintiff rule, a defendant is liable for the full consequences of a plaintiff's injury, even though, due to the plaintiff's peculiar susceptibility to harm (of which the defendant was unaware), those consequences were more severe than they would have been in a normal person. Here, the woman was negligent, so the question then goes to damages. Applying the eggshell plaintiff rule, the defendant must take the plaintiff as she finds him. The man's childhood condition caused him to suffer more severe injuries than a normal person would have incurred. Even if the woman was unaware of the man's condition, she will be liable for the full extent of the injuries she caused. Therefore, this answer choice is correct.

A dentist entered into an agreement with a supply company pursuant to which the dentist promised to purchase all of her supplies from the company for two years. However, when the dentist decided to move into a new office, she ordered three new dental chairs from a different company. The cost of the chairs was $9,000, payable 60 days after delivery. The agreement with the original supplier was primarily oral; the only document that could prove the agreement was a letter, signed by the dentist but not by the company, which outlined the general nature of the agreement. It contained no specific terms such as item or quantity. Can the supply company enforce the agreement? (A) Yes, under the doctrine of promissory estoppel. (B) Yes, because the parties entered into a valid requirements contract. (C) No, because neither party agreed to a quantity term. (D) No, because the dentist's promise was illusory.

(B) Yes, because the parties entered into a valid requirements contract. Under the facts presented, the dentist and the supply company entered into a requirements contract. Under such a contract, the seller undertakes to supply all of the requirements needed by the buyer. Under the Uniform Commercial Code, contracts to supply a buyer's requirements are considered to provide sufficient legal detriment and can be enforced. It is implied that both buyer and seller will act in good faith in making their determination of the quantities needed. Therefore, the supply company can enforce the agreement with the dentist.

A man lived in a duplex and worked at night as a security guard. The man soon became annoyed at his upstairs neighbor, who kept a large number of dogs inside his home during the day while he was at work. The noise from the dogs was so loud that the man could not fall asleep. On several occasions, the man asked the neighbor to do something about the dogs, but the neighbor simply replied, "Get a day job!" The following day, the man was coming home just as the neighbor was leaving for work and again pleaded with him to keep the dogs quiet, because he had been awake for 24 hours straight at that point, and desperately needed sleep. The neighbor simply smiled and got into his car, but said nothing to the man. A short time later, the dogs began to bark, and the man fired three shots through the ceiling of his apartment, as at that point he merely wanted to put an end to the noise. One of the bullets struck the neighbor, who had returned home because he had forgotten his laptop. The neighbor died a week later from the gunshot wound. For what crime should the man be found guilty? (A) Murder, because the man intentionally shot into a dwelling. (B) Involuntary manslaughter, because while the defendant was criminally negligent, he did not act with malice. (C) Voluntary manslaughter, because a jury could conclude the man was adequately provoked. (D) Felony murder, because the man intended for the bullet to enter the apartment to cause either damage to the building or injury to the dogs.

(B) Involuntary manslaughter, because while the defendant was criminally negligent, he did not act with malice. The man lacked malice, because he was unaware that anyone was upstairs at the time. While it is not reasonable to shoot a gun into a dwelling, occupied or not, the defendant was under the impression that the victim had left the premise. Therefore, while his action was criminally negligent and led to the death of the neighbor, the man lacked malice, and should be convicted of involuntary manslaughter.

A victim was leaving his favorite local watering hole and contemplating a late-night cheeseburger when he tripped over his own feet and fell on the sidewalk. A few moments later, the victim got up in a daze and stepped into the street, where he was struck by a car driven by the defendant, who was driving 35 miles per hour and could not break in time to avoid the victim. The posted speed limit was 25 miles per hour. The victim died in the hospital two hours later. What is the most serious crime for which the defendant can be found guilty? (A) Felony murder. (B) Involuntary manslaughter. (C) Voluntary manslaughter. (D) Depraved-heart murder.

(B) Involuntary manslaughter. Involuntary manslaughter consists of two types: (1) criminal negligence manslaughter, which requires conduct creating an unreasonable and high degree of risk of death or serious bodily injury (i.e., more than ordinary tort negligence); and (2) unlawful act manslaughter, where the death-causing conduct occurs during the commission or attempted commission of an unlawful act (generally a malum in se misdemeanor) involving a danger of death or serious bodily injury. Malum in se crimes generally include morality offenses and serious traffic offenses, as well as criminal assault or intentional battery. By exceeding the speed limit, the defendant committed a traffic violation (i.e., a misdemeanor) that resulted in the victim's death. The defendant may be found guilty of involuntary manslaughter based on the fact that his unlawful act directly and proximately caused the victim's death. Therefore, this choice is correct.

A man makes the following statement to a woman: "I hereby offer to sell you Blackacre for $10,000. This offer expires in 30 days." Which of the following is most accurate regarding the man's offer? (A) It is a firm offer that is irrevocable before the 30-day period has passed. (B) It is neither a firm offer nor an option contract, and is revocable at any time before acceptance has been made. (C) Because the man is not a merchant, the woman has a reasonable time to accept. (D) Regardless of the man's status, the woman must accept within 30 days and before receiving notice of the man's direct revocation.

(B) It is neither a firm offer nor an option contract, and is revocable at any time before acceptance has been made. An offeree's power of acceptance terminates at the time stated in the offer. There are two ways for contracting parties to keep an offer open and irrevocable: a common law option contract, and a firm offer under the UCC. Under the UCC, only a merchant can make a firm (irrevocable) offer to either buy or sell goods without consideration, so long as the offer: (1) is made by a merchant; (2) is made in a writing signed by the merchant; and (3) expressly states by its terms that it will be held open. At common law, three elements are required for an enforceable option contract: (i) an offer; (ii) a subsidiary promise to keep the offer open; and (iii) some valid mechanism for securing enforcement of the subsidiary promise (consideration). Here, assuming that Blackacre is real property, the UCC will not apply. Under the rule of free revocability in American contract law, the offeror is free to revoke an outstanding offer, terminating the offeree's power of acceptance, at any time and for any reason, so long as the revocation occurs prior to acceptance and is effectively communicated. Because there was no consideration exchanged here, no option contract was created. The man's promise to keep the offer open is therefore simply an offer with an express termination date. As such, the offer was neither a firm offer nor an option contract, and is revocable at any time prior to acceptance.

Former employees of a local college planned to file an action in state court against the college's board of trustees. On April 15, they met with the university president, who was a member of the board, and told him that they were planning to sue. They claimed that they had been unlawfully discharged due to their political beliefs. The plaintiffs filed their complaint on April 25. Three of the 12 defendants were served with the process on the same day. The nine others were served on May 19. The first three who were served and eight of the nine others then petitioned for removal to federal court on May 25. The one defendant who had been served on May 19 but who did not join in the petition for removal on May 25 was the university president. He had been traveling out of state. The president joined in the petition for removal on June 18. April has 30 days, and May has 31 days. This court follows the majority rule regarding such petitions and did not make any adjustments for weekends. For the case to be properly removable to federal court, when must the president have joined the petition for removal? (C) May 15, 30 days from the time the president knew about the lawsuit. (B) June 18, 30 days from the time the president was served. (D) May 29, 15 days from the time the president was served because he was served in the second round of service. (A) May 25, 30 days from the time the first defendant was served.

(B) June 18, 30 days from the time the president was served. In an action based on federal-question jurisdiction, the action is removable without regard to the citizenship or residence of the parties. However, all defendants must consent or join in removal for removal to be proper [28 U.S.C. Sec. 1446(b)]. There is a split of authority amongst federal courts as to when the time for removal begins to run and when such time expires in multiple defendant cases. A slight majority follows the intermediate, or McKinney, rule that, in multiple defendant cases, all defendants served within 30 days of the first defendant being served must join in an existing removal action or file their own. However, individual defendants have 30 days from the time they are served with process or with a complaint to join in an otherwise valid removal petition [McKinney v. Bd. of Trustees of Md. Cmty. Coll., 955 F.2d 924 (4th Cir. 1992)]. Here, because the president was served on May 19, he would have had until June 18 to join the petition.

In the latest of a long series of government corruption scandals, a state employee was convicted in federal court of taking kickbacks in exchange for influencing government contract decisions. The employee was sentenced to time served plus probation, fined heavily, and given a significant amount of community service. The employee resigned from his position in the government and began to collect his pension, for which he had fully qualified after spending several decades in government service. The media began to lash out against the state government for what it viewed as lax punishment by a corrupt government. In response, the state legislature passed, and the governor signed into law, a bill which stripped former government employees of any retirement or other benefits or payments of any kind from the state if the employee was convicted of bribery or corruption, declaring that such a conviction amounted to breach of the government official's employment contract. As a result of the new law, the state employee was notified that the state was immediately discontinuing pension benefit payments to him on account of his conviction. The state employee then properly filed a lawsuit challenging the termination of his retirement benefits on the grounds that the new law was unconstitutional. Which of the following is the best argument the state could make in favor of the law's constitutionality? (A) The notification that the state employee received was sufficient notice to satisfy due process for the discontinuation of pension benefits. (B) Deprivation of pension benefits is not cruel and unusual punishment. (C) It is implicit that one of the conditions of the state's contract of employment with a government official is that he shall not take kickbacks or otherwise engage in bribery. (D) The state employee was afforded an opportunity to express his views about the new legislation at public hearings prior to the enactment of the statute.

(C) It is implicit that one of the conditions of the state's contract of employment with a government official is that he shall not take kickbacks or otherwise engage in bribery. This question requires a 2-step analysis to select the best alternative. First, you must determine the constitutional issue involved. 2nd, you must apply the appropriate constitutional principle to the question asked--i.e., the state's best argument against a constitutional challenge to the forfeiture statute. This answer choice is correct because the argument concerning a condition of employment contract with a government official affects the Contract Clause of the Constitution. The state employee could validly challenge the constitutionality of the statute, alleging unconstitutional impairment of the obligation to contract. The pension forfeiture statute would be violative of the Contract Clause because under the circumstances, the state employee has satisfied the conditions of retirement eligibility (he was fully qualified for his pension, as stated in the facts). His retirement pay has ripened into a full contractual obligation and has become a vested right. Therefore, the pension forfeiture statute would be an unconstitutional impairment of his vested right to receive retirement benefits (his pension).

A private investigator who does a great deal of divorce and pre-divorce work recently obtained evidence that a wealthy and volatile husband had been cheating on his wife. When the husband learned of the investigator's discovery of the evidence, he came to the investigator's office and initiated a series of screaming matches with him. The investigator always believed the husband to be an idiot because, even though he had several inconspicuous cars at his disposal, he always drove an absolutely unique classic car when he went to meet his lover. One night, when the investigator was crossing the street, he saw the husband's car coming directly at him. When the car came within 20 feet of him without slowing down or swerving, the investigator pulled out his gun and shot at the husband. The husband died from the gunshot wound the next day. Prior to his death, the husband was heard to repeatedly say, "Why did that man shoot me? He could have gotten out of the way. My brakes failed, and I couldn't stop." The investigator, in turn, told the police that he thought that the husband was trying to hit him with the car to get revenge. Of what crime, if any, is the investigator likely to be convicted with regard to the husband's death? (A) Murder, because the investigator intended to cause the husband's death. (B) Manslaughter, because the investigator thought that the husband was trying to kill him. C) Murder, because the investigator acted with wanton and willful disregard for human life. (D) No crime.

(B) Manslaughter, because the investigator thought that the husband was trying to kill him. While self-defense is a full defense to murder, the force used in self-defense must be appropriate to the threat. Here, the facts indicate that the investigator did genuinely believe that his life was in danger from the husband. However, the facts also indicate that this belief was unreasonable and that the force used by the investigator to protect himself--shooting the husband with a gun--was unreasonable, given that merely moving out of the car's path would have sufficed to save him from harm. As such, the investigator's unreasonable belief that his life was being threatened by the husband constitutes an imperfect claim of self-defense, which would not operate to completely negate the investigator's liability but would serve to mitigate his murder conviction to manslaughter. As such, this is the best response.

A man was recently hired as a professor at a state university. The university had a policy of offering tenure to its professors after three years. Prior to that, the professor could be let go for any reason, though the university typically offered notice and conducted a hearing prior to terminating any professor's employment. 2.5 years after he was hired, the university administration underwent a change of personnel, and during this period, the professor was told that he was being let go. The professor was never given notice or a hearing prior to his termination. If the professor sued challenging the constitutionality of the state's action, will the professor succeed? (A) No, because a state's procedure with respect to state employees is a matter reserved to the state under the 10th Amendment. (B) No, because he does not have a right to notice and a hearing protected by the Due Process Clause of the 14th Amendment. (C) Yes, because he was denied a notice & hearing. (D) Yes, because the state's action is an act of attainder.

(B) No, because he does not have a right to notice and a hearing protected by the Due Process Clause of the 14th Amendment. An employee who has no expectation of continued employment (i.e., one who has been hired on an at-will basis) has no due process rights regarding termination. Here, because professors have no expectation of a permanent job until they are offered tenure after three years, they have no property right, and therefore, no entitlement to a hearing. As such, the state's discharge procedure need not be consistent.

Two police officers, a sergeant and a lieutenant, were charged with robbery and felony murder. The sergeant's lawyer moved to have the two officers tried separately, and the court granted the motion. The sergeant's trial was held first, and he was acquitted. At the lieutenant's trial, his lawyer called the sergeant's wife as a witness. The sergeant's wife testified that after the sergeant was acquitted, the sergeant told her that he had committed the robbery because he wanted to buy her a new ring and that he felt bad because the lieutenant was not involved. Should the wife's testimony be admitted? (A) No, because the sergeant is estopped from contradicting the jury's acquittal verdict. (B) No, because it is hearsay not within any exception. (C) Yes, because it is a statement by a co-conspirator. (D) Yes, because it is a statement against a penal interest.

(B) No, because it is hearsay not within any exception. Pursuant to Federal Rule of Evidence 801(c), hearsay is an out-of-court statement offered to prove the truth of the matter asserted therein. Some out-of-court statements are admissible under exceptions to the rule excluding hearsay evidence. Furthermore, a statement is not hearsay if it is offered to prove something other than the truth of the matter asserted in the statement. Here, the wife's testimony is being used to prove the truth of the matter asserted therein--that is, the lieutenant did not participate in the crime. Therefore, the testimony is hearsay. Because the statement does not fall within any of the exceptions to

In the wake of a recent presidential election mishap that garnered national media coverage, a state revamped its voting procedures in an attempt to streamline the process. Among the many changes the state instituted were vote-by-mail, same-day voter registration, and the requirement that voters check in (or register) at the table labeled with their party affiliation. One couple in the state, who were very active within their local community, were registered Republicans in a heavily Democratic district. The couple did not wish for their party affiliation to be made public, fearing repercussions. The couple challenged the constitutionality of the rule requiring voters to check in or register according to party affiliation. Assuming proper standing, should the court invalidate the rule? (A) No, because it passes strict scrutiny. (B) No, because it passes the rational basis test. (C) Yes, because it violates their procedural due process rights. (D) Yes, because it violates their right to privacy.

(B) No, because it passes the rational basis test. The court is likely to uphold this rule, because the state will only need to show a rational relationship between the rule and the objective to streamline voting procedures. Strict scrutiny applies when the government restricts the right to vote, a right that has been held to be fundamental under the Constitution. However, rational basis scrutiny applies to reasonable restrictions based on things like age or duration of residency. Moreover, voter registration requirements and regulation of the time, place, and manner of casting ballots are valid so long as they do not impose an undue burden on the right to vote. Here, there is no restriction at all on the right to vote, other than the concern that others will know your party affiliation. Because the state made vote-by-mail an option for everyone and because voter registrations are public records anyway, it seems very unlikely that this rule will be held to violate the Constitution. There is no constitutionally protected privacy right as to party affiliation.

A state enacted a statute which required that 60% of the employees of any business that was located in the state and that employed more than 10 people be residents of the state. A woman worked at a telemarketing firm close to the border in the state and commuted from the neighboring state. The telemarketing firm had 20 employees, 15 of whom lived outside of the state and either commuted into the office or worked remotely. The woman was informed by her manager that she would be one of the seven nonresident employees whose employment was being terminated pursuant to the new state statute. The woman properly filed an action challenging the state statute on constitutional grounds. Is the state statute constitutional? (A) No, because it authorizes employers to arbitrarily dismiss some nonresidents while retaining others. (B) No, because it unfairly discriminates in favor of residents of the state as to the basic right to employment. (C) Yes, because a state is permitted to treat nonresidents differently from residents whenever it sees fit. (D) Yes, because the woman was discharged in compliance with the ordinance and was not otherwise discriminated against by her employer.

(B) No, because it unfairly discriminates in favor of residents of the state as to the basic right to employment. The state is discriminating against nonresidents with respect to one of the privileges and immunities of citizenship protected by Article IV, Section 2--the right to pursue a livelihood. This clause protects nonresidents from being discriminated against by state or local governments with respect to certain basic rights or essential activities of citizenship. Among these are the right to own property, equal access to medical care and the courts, and the right to earn a living. Here, the state statute discriminates against nonresidents. When discrimination against the employment rights of nonresidents is found to exist, the law will be struck down unless the responsible state or local government can show such discrimination is proper because the nonresidents are a peculiar source of the evil the law was designed to redress. For instance, if the state were experiencing high unemployment because nonresidents were taking all the jobs, a court could find that the ordinance was constitutional. However, no such showing has been made here.

In the wake of the federal government suspending welfare benefits and subsidized medical care, states began to see a significant influx in citizens claiming such benefits under state programs. One state in particular, which provided better coverage than most others, soon found that many people were moving into the state for the sole purpose of claiming these benefits. In response to the suddenly increased drain on the state budget, the legislature passed a bill requiring that anyone filing a claim for welfare or state-subsidized medical care show proof of having resided within the state for at least one year prior in order to be eligible for such benefits. A woman moved to the state six months ago for a job, but was currently unemployed after being fired due to downsizing. The woman applied for state welfare benefits, but her application was denied because she had not been in the state for a full year. The woman filed a proper suit challenging the constitutionality of the statute. Is the statute constitutional? (A) No, because it discriminates against the poor in violation of the Equal Protection Clause. (B) No, because it violates the Privileges and Immunities Clause of Article IV, Section 2. (C) Yes, because it is rationally related to a legitimate government interest. (D) Yes, because it violates the Due Process Clause of the Fourteenth Amendment.

(B) No, because it violates the Privileges and Immunities Clause of Article IV, Section 2.The Privileges and Immunities Clause of Article IV, Section 2 protects the fundamental right of citizens to travel freely from state to state. Pursuant to this, durational residency requirements for dispensing government benefits are subject to strict scrutiny. While a reasonable residency requirement would likely be found valid, a one-year waiting period before being able to receive welfare benefits or state-subsidized medical care has been found invalid.

A man from State B brought suit against his former employer in an appropriate U.S. District Court, claiming $100,000 in damages as a consequence of the employee's termination in violation of the Civil Rights Act. The man, who is African American, claimed that he was terminated after he brought several safety violations to the attention of state inspectors when the employer's facilities were being investigated for state health-code violations. The employer operates in States A, C, and D and is incorporated in State B. The man's salary was $37,000 annually, and he had two years remaining on his employment contract. If the employer moves to dismiss the case for lack of subject-matter jurisdiction, should the court grant the motion? (A) No, because a federal court can hear violations of state employment contract disputes. (B) No, because subject-matter jurisdiction is satisfied. (C) Yes, because the federal court lacks diversity jurisdiction. (D) Yes, because the man's claim amounts to a breach of contract for termination stemming from whistleblowing to state inspectors.

(B) No, because subject-matter jurisdiction is satisfied. Because the man is asserting a federal claim against the employer, subject-matter jurisdiction exists, even though the amount in controversy does not exceed $75,000 and without regard to diversity.

A prominent attorney was suspected of growing marijuana in his home. Based on an anonymous tip, the police went to the lawyer's neighborhood just before dawn. The police could not see into the lawyer's house from the street due to lack of overhead street lights. Seeing that a neighbor's yard looked directly into the lawyer's home, the officers climbed over a fence and stood in the neighbors' yard and looked through the man's living room window. The officers observed what appeared to be marijuana plants being cultivated under grow lights in the kitchen. Based on the tip and their observations, the officers then obtained a search warrant which turned up 40 large marijuana plants. Following the lawyer's arrest, he brings a motion to suppress claiming that the evidence supporting the warrant was obtained through a search that violated the Fourth Amendment. Will the lawyer be successful in his motion to suppress? (A) No, because the house was searched pursuant to a valid warrant. (B) No, because the Fourth Amendment does not protect unlawful activities conducted in plain view. (C) Yes, because the officers' clandestine observation of the plants violated the lawyer's reasonable expectation of privacy concerning activities occurring in his home. (D) Yes, because no unlawful activities could be observed by the officers from any public vantage point.

(B) No, because the Fourth Amendment does not protect unlawful activities conducted in plain view. The marijuana plants were in plain view of the neighbors, and the lawyer has no standing to complain of any police trespass on the neighbors' property [Horton v. California, 496 U.S. 128 (1990)].

On Monday, a chicken farmer emailed a restaurant owner promising to sell him 500 chickens for $2,000. On Tuesday, the owner replied to the farmer that he was rejecting the offer because the price was too high. The farmer quickly replied, "Why don't you wait and think it over?" The following morning, the owner learned that due to a salmonella outbreak, there was a nationwide chicken shortage. He immediately telephoned the farmer to accept. The farmer knew it was the owner, and so he answered the phone by saying, "Good morning, I hereby revoke my offer." The owner replied, "You told me yesterday to think it over, and I have, I accept." The farmer then replied, "You had your chance and you blew it." After the farmer refused to sell the chickens to him, the owner sued for breach of contract. Is there an enforceable contract between the parties? (A) No, because the offer terminated due to a lapse of time. (B) No, because the farmer revoked the offer before the owner accepted it. (C) Yes, because the farmer's statement, "Why don't you think it over until tomorrow," revived the offer, which the owner subsequently accepted in a timely fashion. (D) Yes, because the farmer made the owner a firm offer, which was irrevocable.

(B) No, because the farmer revoked the offer before the owner accepted it. This is a contract for the sale of goods, and so the UCC will govern. There was a valid offer made by the farmer, which was rejected via email by the restaurant owner. There was then a second offer made, or a revival of the original offer, when the farmer told the restaurant owner to wait and think over the decision. Like the first offer, the revived offer was revocable, because it was not a firm offer. Because the farmer revoked the offer before the owner accepted, there was no longer an offer for the owner to accept.

A patient sued his doctor for malpractice. The patient called an expert witness to testify that the drugs prescribed by the doctor were so experimental that it constituted negligence under accepted practices in the medical community. On cross-examination, the expert stated that Pharmacology: A Guide for the Practitioner was a reliable authority in her area of specialty. At trial, it was also stipulated that the treatise in question was first published at the beginning of the last century and had been reprinted every year since with updates. The doctor's attorney then proposed to enter the treatise as an exhibit, highlighting a passage which stated that the drugs prescribed by the doctor are widely used by other physicians in treating patients. The patient's attorney objected. Will the court admit the treatise? (A) No, because the text of the treatise constitutes inadmissible hearsay. (B) No, because the learned treatise exception requires the relevant sections be read to the jury, but not received as evidence itself. (C) Yes, as hearsay falling under the learned treatise exception. (D) Yes, because the jury determines the weight of the evidence.

(B) No, because the learned treatise exception requires the relevant sections be read to the jury, but not received as evidence itself. FRE 803(18) provides that statements contained in treatises may be admitted into evidence during direct or cross-examination of an expert witness if: (1) the treatise is established as a reliable authority; and (2) the treatise is called to the attention of the expert witness during cross-examination or is relied upon by the expert in direct testimony. As an exception to the hearsay rule, statements in treatises admitted pursuant to FRE 803(18) may be used as substantive evidence. They may be read to the jury, but cannot be provided directly to the jury. Therefore, the treatise itself will not be admissible.

A landlord and a soft drink manufacturing company entered into a two-year commercial lease agreement for the lease of a factory owned by the landlord. When the soft drink manufacturing company took possession of the factory at the beginning of the lease term, its engineers discovered that there was a high concentration of toxic chemicals in the factory's air due to the manufacturing activities of the previous tenant. The landlord, who was not aware of these toxic chemicals until notified by the soft drink manufacturing company, refused to pay to have the chemicals removed from the air. The soft drink manufacturing company's lawyer wrote a threatening letter to the landlord asserting that it had breached the lease's implied warranty of habitability. In most jurisdictions, would the lawyer's contention be successful in a court of law? (A) No, because the landlord did not know about the chemicals at the time it entered into the lease with the soft drink manufacturing company. (B) No, because the lease is a commercial lease. (C) Yes, because the tenant notified the landlord of the toxic chemicals within a reasonable time. (D) Yes, because the lease is not a long-term lease

(B) No, because the lease is a commercial lease. The lease in this example is a commercial lease, and commercial leases do not have implied warranties of habitability. Therefore, the lawyer's contention that the landlord violated the lease's implied warranty of habitability is without merit.

A plaintiff and defendant were playing basketball. The plaintiff knew that the defendant had a reputation for being an extremely competitive player. During the game, the plaintiff's eye was severely injured when the defendant poked the plaintiff in her face while trying to get a rebound. If the plaintiff brings a cause of action against the defendant, will the plaintiff prevail? (A) No, because the defendant did not intend to injure the plaintiff. (B) No, because the plaintiff impliedly consented to this kind of injury. (C) Yes, because she suffered a harmful or offensive contact. (D) Yes, because the defendant's actions fell below the standard of care.

(B) No, because the plaintiff impliedly consented to this kind of injury. A plaintiff cannot recover in situations where an injury is caused by a defendant's negligence if the plaintiff expressly or impliedly consents to undergo the risk created by the defendant's conduct. The plaintiff will be barred from recovery, or his recovery will be reduced if the defendant establishes that the plaintiff had knowledge of and appreciated the nature of the danger involved. In some circumstances, a court will determine that a defendant has no obligation to be non-negligent toward the plaintiff because of the nature of the activity in which they are engaged. The most common context is sports. Here, the plaintiff knew that the defendant has a reputation for being an extremely competitive player, and knew or should have known that eye-poke injuries are common in basketball. In fact, some NBA players wear protective goggles to prevent eye injuries. The injury occurred in the regular course of play--that is, the plaintiff was injured while the defendant was trying to get a rebound. Thus, even if the defendant was negligent, the plaintiff will likely not prevail.

Following a judicial determination that it had engaged in past discrimination, a state university agreed to hold 15% of the admissions spots in each department for minority students. A couple years later, a white female student applied to the state university's comparative poetry program and was rejected. The student's aptitude test scores and grade point average were above more than half of the minority students admitted into the program. The student subsequently brought suit against the university claiming that she was unlawfully discriminated against because of her race. Should the court rule in favor of the student? (A) No, because a state may consider race as a factor in admissions when it is attempting to aid disadvantaged minorities and penalizes no particular group. (B) No, because the racial classification is designed to remedy past unlawful discrimination. (C) Yes, because a classification solely by race, even to achieve a worthy purpose, is not necessary to satisfy a compelling state need, and therefore violates the Equal Protection Clause. (D) Yes, because a state university may not use race as a criterion in making admissions decisions.

(B) No, because the racial classification is designed to remedy past unlawful discrimination. While race/ethnic origin may be considered in admissions programs, minority set-asides are not constitutional when they are instituted in an attempt to remedy general societal injustice. However, racial quotas may be used to remedy past unlawful racial discrimination [University of California Regents v. Bakke, 438 U.S. 265 (1978); Grutter v. Bollinger, 539 U.S. 306 (2003)]. Here, the state university has been found guilty of past discrimination, and instituted the affirmative action program under court order. As such, the set-aside here is legal, and the court should rule in favor of the university.

A publishing company offered to buy a manuscript for $100,000. The writer, as part of an ill-advised negotiating strategy, rejected the offer. The next day, the publishing company filed for bankruptcy. When the writer learned of the bankruptcy, she quickly realized that the original offer was the best she would receive and called the publishing company, telling the receptionist, "I accept your offer." Will the writer be successful in her lawsuit to recover $100,000 from the publishing company's bankruptcy estate? (A) No, because the publishing company's bankruptcy action negated its contractual liability. (B) No, because the writer had already rejected the offer. (C) Yes, because the bankruptcy action would not affect the valid offer. (D) Yes, because $100,000 is sufficient consideration to support the contract.

(B) No, because the writer had already rejected the offer. Once an offeree refuses an offer, the offer is closed. Here, because the writer initially refused the publishing company's offer, the offer was no longer open when she changed course and attempted to accept it. Had she desired more time to consider the offer, she should have sought a written extension of the offer. Under the circumstances, however, the writer cannot recover.

A landlord entered into a residential lease agreement with a tenant. Five months after the commencement of the lease, the leased premises were destroyed by a fire caused by a lightning strike. The lease agreement did not contain any provision addressing natural disasters or acts of God. Is the tenant required to rebuild the premises? (A) No, because under the modern contract theory, neither the landlord nor the tenant is required to rebuild the premises in the event of an act of God. (B) No, because under the common law estate theory, the tenant is required only to make minor repairs. (C) Yes, because under the common law estate theory, the tenant has a duty to repair without exception. (D) Yes, because the lease does not address acts of God.

(B) No, because under the common law estate theory, the tenant is required only to make minor repairs. Under the common law estate theory, a tenant has the duty not to commit waste, which includes a duty to repair the premises. While this duty to repair the premises covers minor repairs to keep the building wind and watertight, the tenant is not required to rebuild the premises if destroyed by an act of God (absent an agreement to the contrary).

The defendant wanted to steal some documents from his wife's divorce attorney's computer, and so he arranged to have a late-night meeting at the attorney's office. He took with him a small vial filled with a drug used to sedate animals for surgery. The drug was odorless and colorless. When the attorney left the room to take a phone call, the defendant put a small dose of the drug into his drinking cup. After he returned, the attorney took a drink and soon passed out in his chair. The defendant then accessed several documents on the attorney's computer and left. The defendant was unaware that the attorney had severe allergies which were exacerbated by the drug, causing him to suffer a heart attack and die. This jurisdiction defines first-degree murder as the intentional killing of another human being with premeditation, and all other murders are classified as second-degree murder. Manslaughter is defined as at common law. This jurisdiction limits felony murder to inherently dangerous felonies and defines those as at common law. What is the most serious offense of which the defendant can be convicted? (A) First-degree murder. (B) Second-degree murder. (C) Voluntary manslaughter. (D) Involuntary manslaughter.

(B) Second-degree murder. Because the defendant did not have any intent to kill the attorney, the issue becomes one of malice. Malice can be express or implied. Implied malice is measured by the circumstances surrounding the defendant's actions. Here, one could argue that a person lacking sufficient medical training could be malicious in their use of a narcotic. Moreover, while there was no burglary here because this was not a dwelling house, there is a robbery, as the defendant used the narcotic to make the attorney fall unconscious in order to take something from him. The use of fear, force, or intimidation is needed for robbery, and the use of a narcotic would be sufficient to satisfy the force element required. Therefore, the defendant would be guilty of felony murder, which in this jurisdiction would fall within the definition of second-degree murder.

The defendant considers himself the greatest and most inventive practical joker on earth. He plans a dinner party incorporating an elaborate joke to spook his guests. The planned joke involves a timed fire near the dining room door, set to go off as the defendant's guests are eating dinner. The defendant has treated the walls, door, ceiling, and floor of the room with a special fireproof finish to prevent them from igniting, and the fire is set to burn for only 45 seconds. On the night of the dinner party, the defendant is electric with anticipation; he cannot wait to see the terrified looks on his friends' faces. However, when the fire goes off as planned, a woman, one of the guests, panics, and runs from the room through the only doorway, which is precisely where the flames are. As the woman runs through the flames, her clothes catch fire, and she dies from her injuries. The defendant should be convicted of which of the following crimes? (A) First-degree murder. (B) Second-degree murder. (C) Voluntary manslaughter. (D) No crime.

(B) Second-degree murder. In most states, murder is divided into two degrees for the purpose of imposing a more severe penalty for some murders than for others. First-degree murder includes intent-to-kill murder accompanied by premeditation, deliberation, and murder in the commission of any of several named serious or inherently dangerous felonies ("BARRK"--burglary, arson, rape, robbery, and kidnapping). Murder not falling within any of the first-degree murder categories is considered second-degree murder. Here, the defendant's act is best described as depraved-heart murder, which falls into the category of second-degree murder. Depraved-heart murder is defined as an unintentional killing resulting from conduct involving a wanton indifference to human life and a conscious disregard of an unreasonable risk of death or serious bodily injury, absent any defense negating the defendant's awareness of the risk. Unlike involuntary manslaughter, depraved-heart murder involves extremely negligent conduct (or recklessness) that is of a higher degree than gross or criminal negligence (the standard for involuntary manslaughter). Here, there is little question that starting a fire in a dining room filled with guests created a great risk of death or serious bodily injury. As such, although the defendant did not intend to hurt anyone, he clearly committed an act that was extremely dangerous and exhibited a wanton and willful disregard for human life. Therefore, the defendant is guilty of murder despite the unintentional nature of the homicide. The question is only one of degree.

Railroad Corporation has brought an action in the U.S. District Court for the Eastern District of Ohio against the United Railroad Firemen's Union ("Union") and against the Union's officers. Railroad is a Pennsylvania corporation that has its principal place of business in New York. The Union is an unincorporated association that has its headquarters in Ohio, which is also the place where the major portion of the Union's administrative work is performed. All of the officers of the Union are citizens of Pennsylvania. The membership of the Union includes railroad employees who reside in Michigan, Pennsylvania, and Ohio. The action asserts that members of the Union destroyed railroad property in Pennsylvania during a strike. Which of the following is most accurate? (C) The action is outside the subject-matter jurisdiction of federal courts, because it is an action involving an unincorporated labor organization. (A) Venue is proper only in Ohio, because that is where the Union is doing business, and would therefore be considered its residence. (D) Venue is proper only in Pennsylvania, because all of the officers of the Union are citizens of Pennsylvania. (B) The action cannot be brought in a U.S. District Court unless it is based upon a federal claim.

(B) The action cannot be brought in a U.S. District Court unless it is based upon a federal claim. The citizenship of an unincorporated association is the citizenship of all the members of the association. As such, the Union's citizenship would be Michigan, Pennsylvania, and Ohio. Because the plaintiff and the defendant are citizens of the same state (Pennsylvania), diversity does not exist. Thus, the plaintiff's action would have to be predicated upon a federal claim to avoid dismissal. If, however, the plaintiff's claim were based upon a federal statute, no diversity requirements would exist.

A carpet cleaner sent the following terms in a writing to a homeowner. "I will clean your carpets up to 1,000 square feet for a price of $100 per room. This offer is non-cancellable for 60 days from today." The carpet cleaner dated the letter and signed it at the bottom. The homeowner received the letter and immediately wrote a letter accepting the carpet cleaner's terms, but did not immediately send the letter. A week after sending his letter to the homeowner, the carpet cleaner received an offer from a hotel owner requesting his services for the next month at several of his hotels. The carpet cleaner wanted to accept the hotel owner's offer, but was concerned about the outstanding letter to the homeowner. Which of the following is most accurate? (A) The carpet cleaner can revoke his offer to the homeowner, because the mailbox rule does not apply to option contracts. (B) The carpet cleaner can revoke his offer to the homeowner, because there was no consideration provided that would make the offer irrevocable. (C) The homeowner has manifested his intent to accept, and if he accepts within 60 days of the date of the letter, there is a contract. (D) The homeowner has three months from the date of the letter to accept, because offers from merchants are irrevocable.

(B) The carpet cleaner can revoke his offer to the homeowner, because there was no consideration provided that would make the offer irrevocable. This is an offer for services, rather than goods, and therefore it is governed by the common law. As such, the offer would need to be an option contract, rather than a firm offer, to remain irrevocable. An irrevocable offer could be made if there was consideration offered, but here there was not. Therefore, revocation is effective if it comes before acceptance. If the carpet cleaner revokes his offer, no contract exists with the homeowner.

A man from State B was injured while operating a machine that was manufactured by a State B corporation whose principal place of business was in State A. The man found his home state to be more favorable to his case, and so brought a state-law products liability action against the corporation in State B, claiming damages of $95,000. One week after being served, the corporation removed the case to federal court. The man now seeks to have the case remanded back to state court. How should the court rule? (D) The case should remain in federal court, because the defendant is permitted to remove a case that could have originally been brought in federal court. (C) The case should remain in federal court, because the corporation removed within 30 days of being served. (A) The case should be remanded, because the amount in controversy in a strict liability case in federal court must exceed $100,000. (B) The case should be remanded, because a federal court lacks subject-matter jurisdiction to hear the matter.

(B) The case should be remanded, because a federal court lacks subject-matter jurisdiction to hear the matter. The defendant cannot remove the action for two reasons. First, the suit could not have been filed in federal court originally, because there is no federal question and the parties are not completely diverse. Second, even if the parties were diverse, the defendant cannot remove a diversity action to federal court if it is a citizen of the state where the suit was filed. That is the case here, because the corporation is a citizen of both State A and State B. Choice (A) is incorrect, as there is no such rule. Choices (C) and (D) are incorrect because the court would not have subject-matter jurisdiction over the case.

A chemist owned a house that he leased to a tenant pursuant to a residential month-to-month lease agreement. Five months after executing the lease agreement with the tenant, the chemist assigned his interest in the lease to his mother. The written lease agreement did not address assignments. What is the status of the lease? (A) The chemist is the landlord under the terms of the lease, because the assignment is invalid. (B) The chemist's mother is the landlord under the terms of the lease, because a landlord under either a residential or a commercial periodic lease may assign his lease interest. (C) The chemist's mother is the landlord under the terms of the lease, because the lease was a residential lease. (D) Neither the chemist nor his mother is the landlord under the terms of the lease, because the lease terminated upon the chemist's attempted assignment.

(B) The chemist's mother is the landlord under the terms of the lease, because a landlord under either a residential or a commercial periodic lease may assign his lease interest. There are four types of leases: term of years, periodic, at-will, and tenancy at sufferance. Of these, only an at-will tenancy terminates when the tenant and/or the landlord attempts to assign his or her interest. By contrast, a periodic lease, such as the month-to-month lease in this case, does not terminate upon a party's assignment of a lease interest. Therefore, under the facts presented, it was permissible for the chemist, as landlord, to assign his lease interest to his mother, who then became the new landlord under the lease agreement.

A plaintiff brought suit against a car owner for personal injuries sustained in an automobile accident. The car was driven by a neighbor, who was angry that the plaintiff would not allow him to date the plaintiff's daughter. Liability is based on a statute that assesses liability on the automobile owner for the negligent actions of those driving with the owner's consent. The plaintiff offers the testimony of his chiropractor's receptionist that the plaintiff told her he needed an appointment for the neck and back pain he has had for several days. The car owner's lawyer objects. Which of the following is most accurate? (A) The court should overrule the objection because the evidence is admissible as a present sense impression. (B) The court should overrule the objection because the evidence is admissible as a statement for purposes of medical treatment or diagnosis. (C) The court should sustain the objection because a statement of personal history is not appropriate if the declarant is available. (D) The court should sustain the objection because the receptionist is not a medical professional.

(B) The court should overrule the objection, because the evidence is admissible as a statement for purposes of medical treatment or diagnosis. The statement appears to be hearsay--an out-of-court statement offered for the truth of the matter asserted. However, it should be admissible under the exception for statements made for the purpose of medical diagnosis or treatment. Note that this exception does not apply only to physicians or other licensed practitioners; it also applies to their agents.

A defendant was a firm believer that psychics have the power to see into the future. She consulted with a practicing clairvoyant before making any major decisions. The clairvoyant told the defendant that she sensed an "evil presence" in the defendant's life that planned to murder the defendant in the near future. The clairvoyant described the evil presence as someone with red hair who was close to the defendant and whom the defendant would never suspect. The only person with red hair the defendant knew was her sister. The following day, the sister visited the defendant on her way home from work. Believing her sister had come to kill her, the defendant struck her sister in the temple with a fireplace poker, killing her instantly. The defendant was subsequently charged with murder. At her murder trial, the defendant claimed that she killed her sister in self-defense because she believed that her life was threatened. If the jury finds that the defendant honestly believed that she was acting in self-defense, which of the following is true? (A) The defendant will not be convicted of a crime, because she had a good-faith belief that her life was in danger. (B) The defendant will be convicted of voluntary manslaughter because the defendant's belief that her life was threatened was unreasonable. (C) The defendant will be convicted of involuntary manslaughter because the defendant honestly believed that her life was in danger. (D) The defendant will be convicted of murder because the defendant's life was not actually threatened.

(B) The defendant will be convicted of voluntary manslaughter, because the defendant's belief that her life was threatened was unreasonable. This fact pattern presents an example of imperfect self-defense. Imperfect self-defense may mitigate murder to voluntary manslaughter where a defendant was either at fault in starting an altercation, or unreasonably, but honestly, believed that harm was imminent or that deadly force was necessary. In this case, the defendant honestly believed that her sister was going to kill her. However, her belief was unreasonable and thus, her crime will likely be mitigated to voluntary manslaughter.

Liz, a citizen of California, filed an action in the U.S District Court in Los Angeles against Watchco, a Texas corporation, and Monitinc, a New York corporation, alleging that they were engaged in a conspiracy to fix prices in violation of the Sherman Act and that her actual damages from the conduct of the defendants were $73,000. Liz further alleged in the second cause of action that Watchco was wrongfully withholding $1,000 that Liz paid to them as a deposit for a computer that was not delivered. Which of the following statements is true? (A) The action should be dismissed by the district court because the value of the aggregated claims does not meet the amount-in-controversy requirement. (B) The district court may exercise supplemental jurisdiction over the second cause of action, but only if it arises from a common nucleus of operative facts with the first cause of action. (C) The federal court has subject-matter jurisdiction because there is complete diversity of citizenship between all plaintiffs and defendants. (D) The federal court lacks subject-matter jurisdiction because Sherman Act violations must be tried in the state court where such violations allegedly occurred.

(B) The district court may exercise supplemental jurisdiction over the second cause of action, but only if it arises from a common nucleus of operative facts with the first cause of action. Under the doctrine of supplemental jurisdiction, a U.S. District Court may, if a federal claim exists, permit the plaintiff to append any state claims arising from a "common nucleus of operative facts" with the federal claim [United Mineworkers v. Gibbs, 383 U.S. 715 (1966)].

An emergency room doctor was camping with his family in a local state park. As the doctor was walking to get water, he heard a woman screaming that her husband was choking. The doctor rushed to help, but was unable to dislodge whatever was causing the man to choke, and the man died. The doctor now seeks your advice as to whether he can charge the man's estate for his services. This jurisdiction does not have a Good Samaritan statute, and the hospital that the doctor works at treats all doctors as independent contractors. What advice would be most accurate? (A) The doctor can recover, because he is not a full-time employee at the hospital. (B) The doctor can recover, based on quasi-contract. (C) The doctor cannot recover, because allowing a doctor to benefit in such a situation would be against public policy. (D) The doctor cannot not recover, because the man died.

(B) The doctor can recover, based on quasi-contract. Although there is no formal contract entered into between the man and the doctor, the doctor can recover in quasi-contract for two reasons. First, courts will find an implied-in-law contractual obligation where there is an equitable imposition of a would-be contract. Second, it will prevent unjust enrichment, where one party has bestowed a benefit on the other. Emergency services are a typical situation where courts will find that a quasi-contract exists.

To investigate a suspected large-scale marijuana farming operation, police officers used a helicopter to fly over an isolated and unoccupied large garden shed. The police used a high-resolution heat-sensing camera that is not available for sale to the public to take thermal photographs of the outside of the shed that suggested the presence of heat lamps associated with marijuana cultivation. The police did not enter the shed, but at trial sought to introduce the thermal pictures as part of their evidence of defendant's drug production. Defendant seeks to have the evidence excluded. Should the court admit the evidence? (A) Yes, because of the plain view doctrine. (B) Yes, because the shed was not within the curtilage of a dwelling. (C) No, because the camera was not generally available to the public. (D) No, because the defendant had a reasonable expectation of privacy in the shed.

(B) Yes, because the shed was not within the curtilage of a dwelling. The Fourth Amendment gives a reasonable expectation of privacy to a dwelling, as well as outbuildings within the curtilage of the dwelling, the living space immediately outside the dwelling. However, under the open fields doctrine, areas beyond the curtilage do not receive Fourth Amendment protection. Because the shed was isolated and far away from the dwelling, the police thermal scanning does not violate the Fourth Amendment, so the exclusionary rule does not apply.

An absentee landlord owned a run-down apartment building. He made as few repairs as possible. One day, a man visited the landlord's building to see whether he wanted to rent an apartment there. The landlord showed the man the top-floor apartment that was available. The landlord quickly steered the man into the apartment, hoping that the man wouldn't notice that the front door was about to come loose and fall off. The man did not in fact notice the broken door. The man nevertheless decided to rent the apartment, and moved in that Sunday night. The next morning, the tenant in the apartment below the man's heard a loud crash from upstairs. The tenant raced upstairs to find the man on the floor, unconscious. The broken door was lying on the floor near the man. The man sued the landlord for negligence. If the man prevails, which of the following provides the best reason? (A) The door represented a patent defect. A patent defect is one that is obvious, and for which the landlord would not needed to have provided a warning. (B) The door represented a latent defect. (C) The contract did not contain any language that stated the man was renting the property "as is." (D) The landlord breached the implied warranty of habitability.

(B) The door represented a latent defect. A landlord has a duty to warn a tenant of latent defects or repair them. Generally, a landlord is under no duty to warn of or repair obvious conditions. However, a landlord is responsible for dangerous natural or artificial conditions of which: (1) the tenant is unaware and the condition is not reasonably apparent; and (2) the landlord is or should reasonably be aware. Here, the landlord was aware of the broken door, and did nothing to fix.

A man on his bike was run off the road when the driver of a car, who was not paying attention to the road, drifted into the bicycle lane. The man ran into the guardrail, pinning his leg against his bike, but did not fall. After the incident, the man chose not to go to a doctor, as he was a little shaken and sore, but otherwise did not appear injured. The next week, as the man was delivering a package as part of his job, his legs suddenly gave way, causing him to tumble down several flights of stairs. He was then rushed to the hospital, where he was treated for two broken legs and a shattered hip. If the man now sues the driver of the car, which of the following statements is the most accurate? (A) The driver is responsible for all injuries sustained by the man because the driver must take the victim as he finds him. (B) The driver is responsible for all injuries sustained by the man because the man's conduct after the accident was reasonable. (C) The driver is not responsible for all injuries sustained by the man, because the man failed to go to the doctor, who may have warned him to stay off of his feet. (D) The driver is not responsible for all injuries sustained by the man, under the doctrine of avoidable consequences.

(B) The driver is responsible for all injuries sustained by the man, because the man's conduct after the accident was reasonable. The avoidable consequences doctrine is a legal principle that places the responsibility of minimizing damages upon the person who has been injured. The plaintiff, after an injury or breach of contract, should make reasonable efforts to mitigate the effects of the injury or breach. If the defendant can show that the plaintiff failed to mitigate damages, the plaintiff's recovery can be barred or reduced. The major function of the doctrine is to reduce the damages brought about by the defendant's misconduct. The plaintiff need only make reasonable efforts to avoid the consequences of the misconduct. Whether the plaintiff took reasonable efforts to alleviate the effects of the injury or breach depends on the circumstances of the particular case, and is subject to the rules of common sense and fair dealing. A party who suffers a personal injury should exercise ordinary care and perseverance to find a cure, and reduce the damages to the most practicable extent. Because the man's conduct was reasonable in light of the fact that he did not appear to be injured, he will be able to recover for the full extent of his injuries. Had the man gone to a doctor, or chosen not to when a reasonable person would have, then the doctrine of avoidable consequences would operate to reduce his recovery against the defendant.

A landlady believed that her tenants were like her family, and for this reason, all of her leases contained a prohibition on assignments. An executive signed a lease on Apartment 3 for five years at $3,000 per month rent. The executive paid her rent on time for four years. She then agreed to spend most of the next year setting up her company's latest headquarters in London. The executive entered into an agreement with a dentist for the dentist to move into the executive's apartment and pay the rent for 10 of the remaining 12 months on the lease. The executive intended to return to and resume living in the apartment for the last two months. The dentist moved into the apartment but never paid any rent to the landlady. When the executive returned home, she discovered a note from the landlady demanding the back rent. Who is liable to the landlady for the rent owed? (A) The executive, because assignments were prohibited. (B) The executive, because the dentist was a subtenant. (C) The executive and the dentist are jointly and severally liable. (D) The dentist, because he agreed to pay the rent.

(B) The executive, because the dentist was a subtenant. Absent an express provision in the lease prohibiting or restricting transfers, a tenant may freely transfer his leasehold interest in whole or part. If he makes a complete transfer of his entire remaining estate, he has made an assignment; where the tenant retains any part of the leasehold interest, the transfer is a sublease. However, a sublease creates no legal relationship between the landlord and the subtenant. As such, the sublessee is not liable to the landlord to pay rent. Thus, here, the executive remains liable to the landlady for the rent owed. Here, the facts indicate that the lease in question prohibited assignments. When such a restriction is present, the restriction will be strictly construed. Therefore, the executive will be prohibited from assigning, but she will be free to sublet her interest. The executive signed a five-year lease and, after four years, signed an agreement with the dentist, whereby the dentist agreed to move into the apartment and pay the rent for 10 of the remaining 12 months on the lease. The executive intended to return and resume living in the apartment for the last two months. As the executive transferred less than her entire estate, the transfer will be a sublease. As discussed above, this sublease will be valid. The tenant-sublessor remains both in privity of estate and in privity of contract with the landlord, and thus continues to be obligated to pay the rent. A sublease creates no legal relationship between the landlord and the subtenant, so the sublessee is not liable to the landlord. Therefore, the executive, and not the dentist, is liable to the landlady.

A hobbyist went out fishing on a lake at dawn. Few other people were on the lake so early in the morning, and the hobbyist enjoyed the peace and quiet. A short while later, the hobbyist noticed a canoe with one occupant heading in his direction. The canoe stopped about 50 yards away. The hobbyist was glad that the canoe didn't come any closer, because he didn't want the movement of the canoe to disturb the fish. The hobbyist then watched as the canoeist stood up in the canoe and began moving around, apparently attempting to organize his fishing tackle. Suddenly, the canoeist's canoe overturned. The canoeist, who was not wearing a life preserver, was thrown into the water. At that same moment, the hobbyist felt a strong tug on his fishing line. He didn't want to lose the fish, so he decided to reel in the fish before going to see if the canoeist needed any assistance. The hobbyist figured that, at worst, the canoeist would have to tread water for a few minutes until the hobbyist got to him. Unfortunately, canoeist did not know how to swim, and by the time the hobbyist made his way over to the canoeist and pulled him out of the water, the canoeist was no longer breathing. Although the hobbyist promptly administered CPR and the canoeist was eventually revived by paramedics, he suffered severe brain damage. If the canoeist's parents sue the hobbyist for the canoeist's injuries, what is the hobbyist's best argument in support of a motion to dismiss their claim? (A) The hobbyist's delay in assisting the canoeist was reasonable. (B) The hobbyist had no duty to help the canoeist. (C) The canoeist was contributorily negligent. (D) The hobbyist properly administered CPR to the canoeist so that the canoeist could be revived by the paramedics.

(B) The hobbyist had no duty to help the canoeist. The hobbyist's best argument in support of a motion to dismiss is that he had no duty to help the canoeist. Barring certain limited exceptions, one has no affirmative duty to assist a person in danger, even if one could do so easily and without endangering herself. If the hobbyist owed no duty to the canoeist, then the canoeist's parents have no claim against the hobbyist.

A landlord and tenant entered into a residential lease agreement. According to the terms of the agreement, the lease would continue until the landlord, in his sole discretion, terminates it by providing 30 days' notice. Two years later, the tenant received a new job and moved out of state, assigning the lease to a new tenant and notifying the landlord. The landlord then filed an action to evict the new tenant. What is the most likely outcome? (A) The landlord will prevail, because the landlord had the discretion to terminate the lease at any time. (B) The landlord will prevail, because the lease was terminated when the tenant assigned it. (C) The new tenant will prevail, because the lease did not prohibit assignments. (D) The new tenant will prevail, because the landlord did not provide the required notice.

(B) The landlord will prevail, because the lease was terminated when the tenant assigned it. A tenancy that has no fixed duration and lasts only as long as the landlord or tenant desire is an at-will tenancy. In this case, the lease agreement had no fixed duration and provided that only the landlord would have the power to terminate the lease. However, most courts will interpret this language as creating an at-will tenancy that can be terminated by either party. An at-will tenancy is terminated automatically if: (1) either party dies; (2) the tenant commits waste; (3) the tenant attempts to assign his interest; (4) the landlord transfers his interest; or (5) the landlord transfers the premises to a third party for a term of years. Because the tenant transferred his interest to a new tenant, the lease was terminated at that time.

A man offered to buy a woman's farm for $100,000. The woman did not wish to sell her farm, but she jokingly accepted the offer because she did not believe that the man has $100,000. The man and the woman worked out the terms of the contract and the woman, still joking, wrote out the contract on a sheet of paper which both parties signed. The man took the writing and subsequently tried to enforce it, arriving with a check for $100,000. In a suit by the man against the woman, what is the likely outcome? (A) The man will win because they signed the agreement. (B) The man will win, because the contract is binding even if the woman did not intend to sell her farm, as the man actually believed this to be a serious transaction and his belief was reasonable. (C) The woman will win because there was no good faith on her part to enter into a contract. (D) The woman will win because there was no true meeting of the minds.

(B) The man will win, because the contract is binding even if the woman did not intend to sell her farm, as the man actually believed this to be a serious transaction and his belief was reasonable. At the heart of any contract issue is the question of whether or not a contract was actually formed. This usually requires a determination of whether there was valid consideration, and whether there was a valid offer and acceptance. In order for a contract to be formed, there must be mutual assent, which is simply the agreement by both parties to enter into a contract. In deciding whether or not there is mutual assent, courts use an objective "reasonable person" test, in which the court examines the exchange between the parties that led to the establishment of the contract and then determines what a reasonable person in the place of the parties would have understood the exchange to mean. Please note that the court is not interested in what the parties actually thought. It is only interested in what a reasonable person in the same circumstances would have thought. Therefore, even if the woman was joking, if the man actually believed this to be a serious transaction and his belief was reasonable, the court will find that an enforceable contract was validly executed.

On Monday, a man told a gardener, "I am having a party on Sunday and I want my house to look good. If you will agree to mow my lawn by Saturday, I will pay you $50. Think about it and let me know." The man did not hear from the gardener all week. Therefore, the man decided to mow the grass himself. On Saturday, the gardener arrived with his lawn mower, and saw that the grass had been freshly mowed. The man informed the gardener that he had just finished cutting the grass himself. The gardener then brought suit against the man for breach of contract, because he argued that he was within the window of time to accept and that he had given up another job in reliance on mowing the man's grass. Who is likely to prevail? (A) The man, because the gardener never accepted the offer. (B) The man, because the offer to the gardener was indirectly revoked. (C) The gardener, because the man did not specify that acceptance had to be by return promise. (D) The gardener, because he gave up another job in order to accept the man's offer.

(B) The man, because the offer to the gardener was indirectly revoked. Unless specified otherwise, an offer can be accepted by any means reasonable. Here, a return promise was requested, but not received. When the gardener arrived at the house, he became aware that the man had already mowed the grass. At this point, the offer was revoked, and the gardener could no longer accept the offer.

On Monday, a man offered to sell his lawn mower to his neighbor for $100. After receiving the man's offer, the neighbor responded, "Let me think it over." The man then said, "If you say so, but I need to know in a day or two, tops." On Friday, the man sold the lawn mower to his brother. Thereafter, the neighbor decided to accept the man's offer and walked to his house, only to find the man's brother loading it onto his truck. If the neighbor sued the man for breach of contract, judgment should be for whom? (A) The man, because the offer to the neighbor terminated when the neighbor saw the brother loading the lawn mower onto his truck. (B) The man, because the offer was terminated due to the lapse of time. (C) The neighbor, because the offer became irrevocable for a reasonable time when the neighbor asked to "think it over." (D) The neighbor, because the sale of a lawn mower is governed by the UCC.

(B) The man, because the offer was terminated due to the lapse of time. The man agreed to hold the offer open for the neighbor to give the neighbor time to think it over, but the man stated that the offer would only be open for, at most, two days. Thus, when the neighbor failed to accept the offer by the end of Wednesday, two days after the offer was made, the offer lapsed due to time.

A man was charged with domestic abuse of his wife after police responded to a call from his house. When they arrived, the man's wife told police that her husband had beaten her. While the woman did not testify at the man's trial, the police officer did testify about what she had told him. The man's attorney objected to the admission of the wife's statement to the police. How should the trial court rule? (A) The statement is inadmissible, because in criminal cases, all evidence presented against the defendant must be subject to cross-examination in front of the trier of fact. (B) The statement is inadmissible, because it was testimonial in nature. (C) The statement is admissible, because it was made while under the stress of a startling event. (D) The statement is admissible, because it was not testimonial in nature and qualifies as an excited utterance.

(B) The statement is inadmissible, because it was testimonial in nature. The Confrontation Clause of the Sixth Amendment, as interpreted in Crawford v. Washington [541 U.S. 36 (2004)], does not apply to nontestimonial statements not intended to be preserved as evidence at trial. In Hammon v. Indiana [547 U.S. 813 (2006)], the Supreme Court ruled that the victim's statements to the police were testimonial. At the time of her questioning, she faced "no emergency in progress" and "no immediate threat to her person." Instead, the relative safety of the conversation between the declarant and the officer made it "formal enough" to qualify as a "testimonial" statement intended as evidence of the past crime.

A pilot decided to fly his small airplane to another city for a weekend visit. He invited one of his best friends to join him as a passenger on the trip. Near the end of the flight, the pilot wanted to show off for his friend. The pilot saw a cloud of fog ahead and decided to navigate through it. However, the fog was much thicker than he anticipated and he got lost, crashing the plane into the side of a mountain. The pilot and his passenger were both killed instantly in the crash. If the passenger's family brings a wrongful death action against the pilot's estate, what is the most likely outcome? (A) The plaintiffs will recover under strict liability, because the pilot engaged in an abnormally dangerous activity. (B) The plaintiffs will recover under negligence, because the pilot failed to exercise reasonable care in the operation of his airplane. (C) The plaintiffs will not recover, because the passenger did not pay to go on the flight. (D) The plaintiffs will not recover, because the passenger unreasonably exposed himself to a known danger.

(B) The plaintiffs will recover under negligence, because the pilot failed to exercise reasonable care in the operation of his airplane. In modern day society, flying an airplane is so commonplace as not to be regarded as an abnormally dangerous activity. Historically, flying was, of course, regarded at its outset as a questionable and highly dangerous enterprise. This view was encouraged by the fact that the first cases arose in New York, where there was strict liability for any physical invasion of land. In fact, the First Restatement of Torts in 1939 took the position that aviation had not reached such a stage of safety as to justify treating it by analogy to the railroads, and classified it as an ultrahazardous activity upon which strict liability for ground damage was imposed. However, with the further development of the industry, later years witnessed a definite reversal of this trend. Therefore, this is the correct answer because the plaintiffs will recover for negligence rather than strict liability.

A chef was a buyer for a gourmet grocery store. He contracted in writing with a seafood supplier for the purchase of 5,000 frozen swordfish steaks at $2 each, for a total contract price of $10,000. The manager of the gourmet grocery store was very pleased with the chef's purchase. The purchase price of the swordfish steaks was only half of the usual price. The gourmet grocery store would be able to sell each steak for $5 each, reaping a substantial profit. The manager approached the chef one afternoon and told him that, because of his good work, he would be receiving a raise in salary of $50 per week beginning the next pay period. Three days later, the manager of the seafood supplier called the chef and informed him that the price per swordfish steak was actually $4, for a total contract price of $20,000. Further, given the size of the error, the seafood supplier would not be able to deliver the swordfish unless the grocery store agreed to pay the additional $10,000. The chef advised the manager of the problem and the manager approved payment of the entire $20,000. Which of the following accurately states the legal effect of the manager's promise to give the chef a raise in pay? (A) The promise is unenforceable because it was illusory. (B) The promise is unenforceable because it was not supported by consideration. (C) The promise is enforceable because the manager is morally obligated to perform. (D) The promise is enforceable, assuming the chef conferred a benefit on the manager.

(B) The promise is unenforceable, because it was not supported by consideration. To be enforceable, a promise must be supported by consideration. Consideration is defined as a bargained-for exchange or, alternatively, as a bargained-for legal detriment. A legal detriment does not arise unless a promisee does something that he is not otherwise legally obligated to do or refrains from doing something that he is legally entitled to do. Under the facts presented, there has been no bargained-for legal detriment. The manager has not bargained for a legal detriment on the chef's part; the chef had already performed the work when the promise was made. These facts provide a classic example of past consideration, which, in fact, is not consideration at all. Thus, the manager's promise to give the chef a raise is not enforceable because it was not supported by consideration.

A woman-owned a used bookstore. One day, a customer came into the store drinking a large soda. He carelessly dropped it on the floor, and the bookstore owner failed to clean it up. About four hours later, a second customer came into the bookstore, hoping to find the first edition of a book. The second customer, trying to break in a new pair of five-inch heels that were a size too small, could barely walk. As she was teetering up and down the aisles browsing the selection, she slipped on the wet floor and broke her ankle. The second customer sued the bookstore owner, and after the trial, the jury found the second customer 20% at fault, the bookstore owner 40% at fault, and the first customer 40% at fault. This jurisdiction has a modified comparative negligence statute. Which of the following is most accurate? (A) The second customer can recover because she was not more negligent than the owner or the first customer. (B) The second customer can recover because she was not more negligent than the owner and the first customer. (C) The second customer can recover as long as she did not have the last clear chance to avoid the injury. (D) The second customer cannot recover, because she has contributed to her own negligence.

(B) The second customer can recover, because she was not more negligent than the owner and the first customer. This jurisdiction has a modified comparative negligence statute, which means that if the plaintiff is more responsible for her injuries than the defendants, she will be barred from recovery. Here, because the second customer's degree of fault is less than the combined total of the defendant's fault, she will be able to recover.

A landlord owned several apartment buildings surrounding a college campus, which he rented to students. After several bad experiences, the landlord now includes in all leases a prohibition against assignments. A sophomore rented one of the apartments for a year, beginning July 1, for $1,200 per month. Unless proper notice was given, the lease would renew automatically each year. During the fall semester, the sophomore was accepted into a program allowing her to study abroad in the spring. Not wanting to give up her apartment due to a housing shortage, the sophomore agreed to let her friend take over the lease for the months of January through May. The friend moved into the apartment, but never paid any rent to the landlord. When the sophomore returned in June, she was greeted by a very angry landlord, who demanded the previous five months' unpaid rent. From whom can the landlord recover the rent owed? (A) The sophomore, because assignments were prohibited under the terms of the lease. (B) The sophomore, because the friend was a subtenant. (C) The sophomore and the friend, because they are jointly and severally liable. (D) The friend, because he agreed to take over the lease.

(B) The sophomore, because the friend was a subtenant. Absent an express provision in the lease prohibiting or restricting transfers, a tenant may freely transfer his leasehold interest in whole or part. If he makes a complete transfer of his entire remaining estate, he has made an assignment. Where the tenant retains any part of the leasehold interest, the transfer is a sublease. The tenant-sublessor remains both in privity of estate and in privity of contract with the landlord, and thus continues to be obligated to pay the rent. A sublease creates no legal relationship between the landlord and the subtenant, so the sublessee is not liable to the landlord. As such, the sublessee is not liable to the landlord to pay rent. Here, the sophomore only conveyed her lease interest to her friend for five months. As such, this was a sublease, not an assignment. Though the lease prohibited assignments, it did not prohibit subleases, making the sublease valid. Accordingly, the sophomore is liable to the landlord for the unpaid rent; the landlord could not collect directly from the friend.

A wife and her husband were having dinner at a restaurant when the wife excused herself to go to the bathroom. The restaurant was owned and operated by a chef. As the wife was walking past a table where another customer was seated, she slipped and fell on an egg roll that had been lying on the floor for quite some time, although the chef was unaware that it had fallen onto the floor. When she fell, her head struck a serving tray that was located in the aisle. The fall caused the wife to suffer a severe concussion. The customer knew that the egg roll was on the floor, and although he could have done so, he did not warn the wife. If the wife asserts a claim against the chef for the injuries she suffered from the fall, what is the most likely outcome? (A) The wife will recover, because the egg roll on the floor constituted an unsafe condition of the premises. (B) The wife will recover, because the egg roll was on the floor for a substantial period of time before the accident. (C) The wife will not recover, because the chef did not know that the egg roll was on the floor. (D) The wife will not recover, because the customer could have prevented the injury by warning the wife of the presences of the egg roll.

(B) The wife will recover, because the egg roll was on the floor for a substantial period of time before the accident. A person who possesses land generally owes a duty to exercise reasonable care for the safety of invitees. An invitee is someone who enters land that is held open to the general public or who enters the land in connection with some potential business dealing that would benefit the person who possesses the land. The obligation to exercise reasonable care extends to everything that threatens the invitee with an unreasonable risk of harm--care against negligent activities, warning of known latent dangers, precautions against foreseeable dangers, and inspection of the premises to discover possible dangerous conditions that are not known. As a patron in a restaurant, the wife should be classified as an invitee. The chef thus owes a duty to exercise reasonable care for her safety. In this very tricky Torts question, students must choose the answer choice that is the most precisely correct and is neither over-inclusive nor under-inclusive. This answer choice is correct because the chef has a duty to act reasonably in inspecting the premises, and that includes inspecting to discover an unsafe condition within a reasonable period of time. Therefore, because the egg roll had been on the floor for a substantial period of time, the chef would be liable.

A woman had known a hairstylist for many years. One day, the hairstylist was walking through the mall when he ran into the woman. The hairstylist said to the woman, "I will straighten your hair tomorrow for $20. Come by the shop." The woman replied, "That's a great price." Thereupon the woman's husband approached and the three then started gossiping about a man who walked by wearing some out-of-fashion shoes. The next day, the woman arrived at the hairstylist's shop and said, "I accept your offer." The hairstylist replied, "I can't straighten your hair because after we spoke yesterday, I contracted with several other people to do their hair, and now I'm all booked up." If the woman sues the hairstylist for breach of contract, who will likely prevail? (A) The woman, because she relied on the hairstylist's promise by coming to the shop. (B) The woman because the hairstylist made a unilateral offer, and the woman arrived at the shop ready to pay $20 for the haircut. (C) The hairstylist, because he contracted with other people to do their hair. (D) The hairstylist, because the woman did not accept during their conversation.

(B) The woman because the hairstylist made a unilateral offer, and the woman arrived at the shop ready to pay $20 for the haircut. The hairstylist made an offer. While it is true that the woman did not accept the offer during the conversation, the offer was for a unilateral agreement, which the woman accepted by coming by the shop the next day.

A man negligently set fire to an apartment building. The fire caused panic and residents rushed to try to escape the burning building. In the rush to get out, a woman was knocked to the ground and trampled by the other residents. The woman later sued the man for her injuries. Which of the following statements is most accurate? (A) The woman will recover because the man is responsible for all injuries sustained by the woman. (B) The woman will recover because the man is responsible for the foreseeable injuries sustained by the woman. (C) The woman will not recover, because her injuries were sustained in an unforeseeable manner. (D) The woman will not recover, because her injuries were sustained by an unforeseeable force.

(B) The woman will recover because the man is responsible for the foreseeable injuries sustained by the woman. It is foreseeable that individuals threatened with harm will try to escape. It is also foreseeable that those efforts to escape may endanger others. Therefore, the man will be held liable for the woman's injuries, even though they were caused by the escaping crowd and not the fire directly.

Two nine-year-old boys, Pete and Victor, were sitting across from each other in their classroom at school. Pete kicked Victor on his shin so lightly that Victor did not immediately feel it. However, minutes later, Victor was crying out in pain due to the injury. Over the next week, the wound swelled and surgery was performed on Victor's shin. The surgeons found that Victor's shin bone was damaged from an earlier sledding accident and the kick that Pete delivered exacerbated the injury, causing bone degradation and irreparable harm to the leg. Due to these events, Victor would never be able to use the leg again. Victor's parents, on his behalf, subsequently brought suit against Pete for the damages that Victor incurred. What is the most likely outcome? (A) Victor should recover for all of his injuries, because he was not at fault. (B) Victor should recover for all of his injuries, because the eggshell plaintiff rule provides that a defendant takes their victim as they find them. (C) Victor should only recover for those injuries sustained by the initial kick, but not for those attributable to the earlier sledding accident. (D) Victor should not recover, because a nine-year-old boy cannot be liable for either negligence or intentional torts.

(B) Victor should recover for all of his injuries, because the eggshell plaintiff rule provides that a defendant takes their victim as they find them. The eggshell plaintiff rule holds that a tortfeasor must take his victim as he finds him. Damages are not mitigated because the victim happens to be more susceptible to injury than an average person. Therefore, the court should rule against Pete for the full amount of damages incurred by Victor, even though the kick would not have normally caused the extent of injury he sustained.

A man's car broke down, and since he was only a mile from the repair shop, he decided to walk. As he approached the repair shop, he tripped on the cracked front cement step. The man fell and fractured his wrist. A woman who was jogging by rushed up to him and asked, "Are you hurt?" The man said, "Yes." The jogger then exclaimed, "It's horrible that a business would allow its front step to deteriorate to this state!" A few months later, the man filed an action against the repair shop to recover from his injuries. Is the jogger's statement admissible at trial? (A) Yes, as a statement by an opposing party. (B) Yes, as an excited utterance. (C) No, as hearsay not within any recognized exception. (D) No, as irrelevant nonhearsay.

(B) Yes, as an excited utterance. Under the Federal Rules of Evidence, an excited utterance is admissible as an exception to the general rule excluding hearsay evidence. An excited utterance is defined as a statement relating to a startling event or condition, made while the declarant was under the stress or excitement caused by the event or condition. A statement made by a witness to an accident will generally qualify as an excited utterance. In this case, the jogger rushed to help and expressed her concern, and was probably still under the stress of the event. Therefore, her statement is admissible pursuant to this exception to the hearsay rule.

A department store had experienced a growing incidence of shoplifting. At the store's request, the police concealed a woman, who was a detective, at a vantage point above the women's apparel fitting rooms, where she could see into these rooms as customers tried on clothes. The detective saw the defendant enter a fitting room, stuff a dress into her pocketbook, leave the fitting room, and start for the street door. By prearranged signal, the detective notified another police officer near the door, who detained the defendant as she started to go out into the street. The defendant was placed under arrest and the dress was retrieved from her purse. The defendant was subsequently charged with shoplifting. At trial, the defendant moved to prevent the introduction of the dress into evidence. Will her motion be granted? (A) Yes, because the police should have secured a search warrant to search her bag. (B) Yes, because a customer has a reasonable expectation of privacy while using a department store fitting room. (C) No, because the search and seizure were made incident to a valid arrest based on probable cause. (D) No, because the detective could see into the room, and thus, the defendant's activities were legitimately in plain view.

(B) Yes, because a customer has a reasonable expectation of privacy while using a department store fitting room. The 4th Amendment prohibits unreasonable searches and seizures by the government without probable cause in places where an individual would have a reasonable expectation of privacy. The police detective did not have the right to look into the dressing room as the defendant was changing solely on the basis that there had been an increase in shoplifting at the store. The fact that the store invited the police to look in the dressing room does not excuse the police of the requirements of the 4th Amendment.

Police detectives looking for a gun used in the murder of a federal judge received a tip that the gun had been hidden in a storage unit from the manager of the storage unit who had seen the suspect placing a gun in the unit. The suspect was subsequently arrested and jailed on an unrelated charge. While the suspect was in jail, the detectives applied for a warrant to search the storage unit for the gun. Before they could execute the warrant, the storage unit manager told a policeman who was on patrol in the neighborhood about the gun. The patrolling officer broke into the storage unit and discovered the murder weapon. Just after the patrolling officer left the storage unit, the detectives arrived at the storage unit with their valid warrant. Subsequent ballistics testing confirmed that the gun was in fact the murder weapon, and the suspect was charged with murder. At trial, the defendant seeks to have the gun excluded from evidence. Should the court admit the gun into evidence? (A) Yes, because the patrolling officer's search was reasonable due to exigent circumstances. (B) Yes, because of the inevitable discovery rule. (C) No, because there were no exigent circumstances justifying the patrolling officer's search. (D) No, because the patrolling officer's search was unreasonable.

(B) Yes, because of the inevitable discovery rule. Under the inevitable discovery rule, a court should not exclude the fruits of an unreasonable search when the police would have obtained the evidence eventually without an unreasonable search. Here, the patrolling officer's search was warrantless and unreasonable, but the detectives would have obtained the evidence shortly after the patrolman did anyway. Because the detectives had a valid warrant, the gun is admissible.

The plaintiff, a man from State A, was injured when the defendant, a citizen of State B, ran a red light in State A, causing $40,000 in damages. During an informal interview before any suit was filed, the defendant told a newspaper reporter that the plaintiff was "a drunk who should not be allowed to drive a car, let alone practice medicine." The plaintiff believed that he suffered $50,000 in damages to his reputation as a result of the newspaper article. The plaintiff filed suit against the defendant in a State A federal court. The defendant objects, claiming that the federal court does not have jurisdiction to hear the case. May the federal court hear the case? (C) No, because even though the plaintiff's damages arose out of the same case or controversy, both claims arose from violations of state law. (A) Yes, because defamation actions are properly heard in federal court. (B) Yes, because the elements for diversity jurisdiction are met. (D) No, because the plaintiff cannot aggregate unrelated claims.

(B) Yes, because the elements for diversity jurisdiction are met. The plaintiff can bring his case in federal court if there is a federal question or diversity of citizenship. Here, there is no federal question involved. However, the plaintiff and the defendant are from different states, and the amount in controversy exceeds $75,000. The amount in controversy can be aggregated even if the claims are unrelated, as long as they are asserted by the same plaintiff against the same defendant. Here, both claims are brought by the same plaintiff against the same defendant. Therefore, aggregation is proper.

The plaintiff, a citizen of State A, went on a business trip to State B and stayed in a hotel. While there, he was injured while using an exercise machine in the hotel's gym. He filed a tort suit in State A state court against the hotel and the technician (an independent contractor) who serviced the gym equipment. The man alleged a violation of State B law and sought $100,000 in damages. The hotel and the technician are citizens of State B for the purposes of diversity jurisdiction. One week after the hotel and technician were served with the process, the hotel's lawyer sought to remove the case to federal court, but the technician did not wish to remove the action. Two months later, the plaintiff joined the manufacturer of the exercise machine as a defendant. Several weeks after being served, the manufacturer, a resident of State C, wished to remove the case to federal court and persuaded the technician and hotel to join in a notice of removal. The manufacturer filed a notice of removal exactly four weeks after being served, and the hotel and technician filed a notice that same day joining in the removal. May the suit be removed under these circumstances? (A) Yes, because the defendants have the right to remove the action at any time prior to trial. (C) No, because the manufacturer waited too long to initiate its removal. (B) Yes, because the manufacturer's removal was timely and the hotel and technician were free to join in the manufacturer's removal. (D) No, because the hotel and technician waited too long to initiate its removal.

(B) Yes, because the manufacturer's removal was timely and the hotel and technician were free to join in manufacturer's removal. Notice of removal must be filed within 30 days of when the grounds for removal become apparent. Normally, this is when process is served, but it could be later, such as when an amendment is made. If a defendant declines to remove within this 30-day window, he may not initiate a removal at a later time. However, he may join in a removal initiated by a defendant that is served at a later time. Here, removal did not occur within the 30-day window applicable to the hotel or technician based on their service. (While the hotel desired to remove, all defendants must agree to removal for removal to occur.) However, when the manufacturer was served and elected to remove within the 30-day window applicable to it, the hotel and technician were free to join in the removal within that same 30-day window. Thus, removal is permissible here.

Tax Inc. franchised thousands of tax preparation offices nationwide. Ten years ago, a franchisee signed a franchise agreement with Tax Inc. granting him the right to operate Tax Inc. franchises in Charleston, West Virginia. Believing that the franchisee breached his original agreement and was using his franchise to further a rival tax preparation business, Tax Inc. filed a request for a permanent injunction in federal court. In its complaint, Tax Inc. correctly identified the franchisee as a citizen of West Virginia and itself as a citizen of Delaware, where it was incorporated, and New York, where it had its primary place of business. In determining the cost of the injunction, Tax Inc. focused on the alleged improper use of the franchise, and valued the stopping of that improper use at $80,000. The franchisee calculated the injunction's cost to be $25,000. He derived this figure from the remaining five years on his franchise agreement, arguing that he had made $5,000 per year in profit for the previous 10 years of the agreement and was likely to continue to do so for the remaining five years. The federal district court then dismissed Tax Inc.'s complaint for lack of subject-matter jurisdiction, finding that it failed to meet the $75,000 amount-in-controversy requirement for diversity jurisdiction. Tax Inc. appealed. Did Tax Inc.'s complaint meet the necessary amount in controversy to be heard in federal court? (D) No, because Tax Inc.'s $80,000 figure was too speculative. (A) Yes, because the value of an injunction is determined by the plaintiff. (C) No, because lost profits is the correct way to determine the value of an injunction. (B) Yes, because the value of an injunction is determined by the larger of the injunction's worth to the plaintiff or its cost to the defendant.

(B) Yes, because the value of an injunction is determined by the larger of the injunction's worth to the plaintiff or its cost to the defendant. For diversity jurisdiction, the amount in controversy must be more than $75,000 [28 U.S.C. Sec. 1332]. To determine the value of an injunction for amount in controversy purposes, courts look at the larger of two figures: (1) the injunction's worth to the plaintiff; or (2) its cost to the defendant [JTH Tax, Inc. v. Frashier, 624 F.3d 635 (4th Cir. 2010)]. Here, Tax Inc. determined that the value of shutting down the franchisee's office was $80,000, which meets the $75,000 requirement.

A particular state has funded abortions for low-income women as part of its state welfare program for the past 15 years. After a recent election cycle resulted in a shift in the makeup of the state's legislature, the state removed the provision allowing for the public funding for abortions. The removal is slated to become effective one year from the date of the enactment of the provision's repeal. Is the removal of the provision constitutional? (A) Yes, because the constitutional right to an abortion does not apply to the states, only to the federal government. (B) Yes, because there is no constitutional right for low-income women to receive state-funded abortions. (C) No, because the statute discriminates against low-income women on the basis of their poverty, in violation of the Equal Protection Clause of the Fourteenth Amendment. (D) No, because the statute places an undue burden on a woman's right to have an abortion.

(B) Yes, because there is no constitutional right for low-income women to receive state-funded abortions. This is actually a correct statement of the law, even though at first glance it might seem too stark of a rejection of the right to an abortion as recognized in Roe v. Wade [410 U.S. 113 (1973)] and reaffirmed in Planned Parenthood of Southeastern Pennsylvania v. Casey [505 U.S. 833 (1992)]. Current law prohibits the government from placing an undue burden on a woman's access to an abortion, but this rule does not require the government to pay for a woman's abortion, even if she is unable to pay for it herself without aid from the state [Maher v. Roe, 432 U.S. 464 (1977)].

A state requires aspiring applicants who wish to become officers in the state National Guard to take a "leadership exam" that consists of 100 multiple-choice questions. The questions measure literacy, knowledge of National Guard procedures, and judgment of possible courses of action in hypothetical combat and noncombat situations. The racial composition of the applicant pool for officer candidates is 54% white and 46% racial minorities. Historically, 60% of white applicants pass the exam, while only 25% of racial minority applicants do so. As a result of the exam requirement, the officer corps of the state National Guard is disproportionately composed of white officers compared to both the proportion of minority applicants and the proportion of racial minorities in the population of the state as a whole. Is the state's leadership exam requirement constitutional? (A) Yes, because a state's police powers includes the absolute right to prescribe qualifications for state officers, including members of its National Guard. (B) Yes, because there is no evidence of intentional race discrimination in the administration of the test. (C) No, because the test has a disparate impact on the success of racial minority candidates. (D) No, because tests that disadvantage racial minorities receive strict scrutiny and are presumptively unconstitutional.

(B) Yes, because there is no evidence of intentional race discrimination in the administration of the test. In Washington v. Davis [426 U.S. 229 (1976)], the Supreme Court held that race-neutral qualifications for state offices did not trigger heightened (i.e., intermediate or strict) scrutiny in the absence of evidence of a discriminatory purpose. In Davis and other cases, the Court has made clear that a disproportionate impact on distinct racial groups, standing alone, is insufficient to show intentional discrimination and trigger heightened scrutiny. Here, there is no evidence of intentional race discrimination other than the disproportionate impact, and so the exam requirement is constitutional.

A state university offers a merit scholarship to those high school students who scored above a certain score on their SATs. Resident aliens are not eligible for this scholarship. A student is a legal resident of the state, but is not a citizen of the United States. He has been denied this scholarship solely because of his alien status. If the student should challenge the constitutionality of his exclusion from the program, will he win? (A) Yes, because the action of the state violates the Privileges and Immunities Clause of Article IV of the Constitution. (B) Yes, because there is not sufficient justification to discriminate against aliens as a class. (C) No, because aliens are not per se a discreet and insular minority; therefore, a classification excluding them is valid. (D) No, because the exclusion is rationally related to legitimate state purposes.

(B) Yes, because there is not sufficient justification to discriminate against aliens as a class. The state may discriminate against aliens only in specific matters relating to the governmental process, such as the denial of a right to run for elective office or to hold a position as a police officer or probation officer or a public school teacher. In all other cases, a state's attempt to discriminate against persons based on alienage will be subject to the strict scrutiny test because alienage is a suspect classification. There is no compelling justification for denying the student the same educational opportunities offered to other residents of the state.

A traffic accident occurred at a road intersection. A motorcycle, a car, and a truck were involved. The motorcyclist was injured and brought suit against the driver of the car and the driver of the truck. The jury returned a verdict finding that the motorcyclist's injuries were caused by negligence on the part of all three of the parties. The jury assigned 55% of the fault for the motorcyclist's injuries to the motorcyclist, 25% to the driver of the car, and 20% to the driver of the truck. The jury found that the number of the motorcyclist's injuries was $100,000. The motorcyclist enforced the judgment against the driver of the car and collects $45,000 from him. The driver of the car then brought an action against the driver of the truck for contribution. What should the driver of the car recover? (A) Nothing, because he was more at fault than the driver of the truck was. (B) $27,000. (C) $20,000. (D) $15,000.

(C) $20,000. On the Multistate Bar Exam, you must assume that pure comparative fault and joint and several liability apply unless the question states otherwise. Under pure comparative fault, the plaintiff can recover damages representing the shares of fault assigned to the defendants, even if the plaintiff's fault exceeded the fault of the defendants. Under joint and several liability, each defendant can be held liable for the full amount of damages owed to the plaintiff, and then the defendant who pays the judgment can use contribution claims to obtain reimbursement from the other defendants for their shares of the judgment. Here, under comparative fault, the plaintiff is able to recover 45% of the damages, or $45,000. If the driver of the car is forced to pay the $45,000 under joint and several liability, the driver of the car then can seek reimbursement from the other defendant for its share of the judgment. Choice (A) is incorrect because a contribution claim would be allowed to distribute the liability among the defendants according to their shares of the fault, even if the defendant seeking contribution had the higher share of the fault. Choices (B) and (D) are incorrect because the amount that could be recovered through a contribution claim would be $20,000, which is the truck driver's share of the judgment.

A man invited several friends to come over to his house to watch a movie on television. While they were watching the movie, freezing rain fell outside, coating everything with a thin layer of very slippery ice. When the movie ended, one of the man's guests slipped on the ice as soon as he took a step out of the door and onto the front stoop of the man's house. Which of the following would best describe the duty of care owed by the man to his guest? (A) No duty of care. (B) A duty to inspect the premises for unknown dangers and disclose their existence to others. (C) A duty to warn of any known dangerous condition on the premises. (D) An absolute duty of care.

(C) A duty to warn of any known dangerous condition on the premises. First, the guest is a licensee, who, in the broadest sense, includes anyone who comes upon the land with a privilege arising from the consent of the possessor. Next, what duty of care does a possessor owe to a licensee? As to passive conditions on the land, it is still the settled rule that the possessor is under no obligation to the licensee with respect to anything that the possessor does not know. The duty is not to maintain the land in a safe condition, but to exercise reasonable care so that the licensee is aware of the danger known to the possessor.

A landlord agreed to lease his townhouse to a tenant for a period of one year. The written lease agreement stated that the lease would automatically renew for successive one-year periods unless either the landlord or the tenant gave notice of termination at least one month (and no more than two months) in advance of the end of the period. One year passed after the landlord and tenant executed the lease agreement, and neither gave notice of termination. After the one-year anniversary of the execution of the lease agreement, what type of tenancy do the landlord and tenant have? (A) An implied tenancy. (B) A term of years. (C) A periodic tenancy. (D) An at-will tenancy.

(C) A periodic tenancy. A periodic tenancy continues from period to period without a set termination date until proper notice is given. The lease in this example is periodic because it renews for successive one-year periods until proper notice of termination is given by either party.

A landlord agreed to lease his townhouse to a tenant for a period of one year. The written lease agreement stated that the lease would automatically renew for successive one-year periods unless either the landlord or the tenant gave notice of termination at least one month (and no more than two months) in advance of the end of the period. One year passed after the landlord and tenant executed the lease agreement, and neither gave notice of termination. After the one-year anniversary of the execution of the lease agreement, what type of tenancy do the landlord and tenant have? (A) An implied tenancy. (B) A term of years. (C) A periodic tenancy. A periodic tenancy continues from period to period without a set termination date until proper notice is given. The lease in this example is periodic, because it renews for successive one-year periods until proper notice of termination is given by either party. (D) An at-will tenancy.

(C) A periodic tenancy. A periodic tenancy continues from period to period without a set termination date until proper notice is given. The lease in this example is periodic, because it renews for successive one-year periods until proper notice of termination is given by either party.

While driving her company vehicle near a pedestrian mall, a woman came upon the scene of a three-car accident. She was so busy gawking at the damaged vehicles that she failed to see one of the victims lying on the road in front of her car. She hit and ran over the victim, who survived and sued the woman's company. The victim offers the testimony of a witness to the incident. Referring to the woman, the witness stated, "The driver of that car ran over the victim as he was lying on the ground awaiting an ambulance, and said 'It is all my fault, I should have been paying more attention to my driving.'" Assume for this question that the woman is available to testify. How should the trial judge rule on the admission of the testimony? (A) Admissible, as a statement against interest. (B) Admissible, as a present sense impression. (C) Admissible, as a statement by a party-opponent. (D) Inadmissible, as hearsay not within any recognized exception.

(C) Admissible, as a statement by a party-opponent. A statement will not be barred from admission by the general rule against hearsay where the statement is offered against an opposing party and was made by the party's agent or employee on a matter within the scope of that relationship and while it existed. Indeed, there is a substantial trend favoring the admission of statements relating to matters within the scope of the agency or employment. Therefore, this answer is correct, because the statement will be admitted as a vicarious statement by a party-opponent.

The defendant was driving down the highway, which had a posted speed limit of 65 miles per hour, at 90 miles per hour when a police officer pulled him over for speeding. As per protocol, the officer called for back-up. The closest officer, who responded to the scene, had a drug-sniffing dog in his car. When the back-up officer arrived, they asked the defendant to step out of the car. While the first officer was questioning the defendant, the second officer allowed his dog to sniff the defendant's car. The dog quickly alerted the officer that he smelled drugs in the trunk of the defendant's car. Which of the following is true? (A) Allowing the dog to sniff the exterior of the car constitutes a search of the vehicle, subject to Fourth Amendment protections. (B) Police may only use a drug-sniffing dog when they already have probable cause to search the defendant's vehicle. (C) Allowing the dog to sniff the exterior of the car does not violate the privacy rights of the suspect. (D) Allowing the dog to sniff the exterior of the car in this situation violates the privacy rights of the suspect.

(C) Allowing the dog to sniff the exterior of the car does not violate the privacy rights of the suspect. The automobile exception is based on the idea that a lesser expectation of privacy exists in an automobile, boat, or airplane than in one's home or personal property. Moreover, the use of drug-sniffing dogs trained to detect contraband but not other items does not invade the privacy rights of the suspect. Therefore, in this case, the defendant's privacy rights in his vehicle were not violated when the police allowed the drug-sniffing dog to sniff his car. Thus, this is the correct answer choice.

A construction company was replacing large portions of a sidewalk in the downtown section of a city. The construction crew started tearing out the old sidewalk and left for the day without putting up warning signs or barriers around the work area. A few hours later, a jogger came around the corner and fell in a hole at the spot where the construction crew had torn up the sidewalk. As the jogger was attempting to get out of the hole, she began screaming for help. Her screams attracted the attention of a passerby, who immediately hurried to assist the jogger. As the passerby was leaning over the edge of the work area, he too fell into the hole. Both the jogger and the passerby suffered serious bodily injuries before they were pulled out of the hole by police. This jurisdiction has not adopted a "Good Samaritan" statute. In a negligence action by the passerby and the jogger to recover for their personal injuries, what is the most likely outcome? (A) The jogger will prevail but not the passerby because there was no "Good Samaritan" statute requiring him to come to the jogger's aid. (B) The passerby will prevail because danger invites rescue, but the jogger will not recover because she created her own peril. (C) Both will recover because the jogger was injured by the negligence of the construction company and the passerby acted reasonably given the situation. (D) Neither will recover because the jogger was running at night and the passerby had no relationship to her.

(C) Both will recover, because the jogger was injured by the negligence of the construction company and the passerby acted reasonably given the situation. The rescue doctrine recognizes that if a defendant's negligence puts someone in danger, it is foreseeable that another person will come to the rescue of the person in peril. If the rescuer is injured, the rescuer can hold the defendant whose negligence created the need for the rescue attempt liable. Thus, the passerby will be able to recover for his injuries. And because the jogger did not create her own peril, she will be able to recover as well.

A chef negotiated with a contractor to remodel his home kitchen to look like the kitchen in his restaurant. In order to do this, the contractor had to order top-of-the-line appliances. He also needed to remove some sections of a wall and install new electrical sockets to accommodate the appliances. The cost of the remodel was $50,000, with two-thirds of the price coming from the purchase of the appliances. A dispute arose and the parties both suspended their obligations. The contractor refused to further perform and the chef refused to pay the contractor. What law should be used to analyze the dispute? (A) The UCC applies to the purchase of the appliances and common law applies to the delivery, the wall removal, and the electrical socket installation. (B) The UCC should be applied to the entire transaction, because the primary purpose of the contract was the appliances, which account for the overwhelming majority of the contract price. (C) Common law should be applied to the entire transaction, because the primary purpose was to provide a service, regardless of the cost of the appliances. (D) When a contract is for both goods and services, in the absence of a choice-of-law selection clause, common law is the default law to be applied.

(C) Common law should be applied to the entire transaction, because the primary purpose was to provide a service, regardless of the cost of the appliances. Following the predominant purpose test, a court will view the predominant purpose of the contract to be for services--specifically, a kitchen remodel. Where a contract has a mix of goods and services, relevant criteria for determining whether the UCC will control will include: (1) the contract language; (2) the nature of the seller's business; (3) the reason for entering the contract; and (4) the amounts charged under the contract for the goods and services. Here, although most of the contract price was for the purchase of the appliances, the purchase was not made directly to an appliance store, where delivery was the only service. Rather, the chef went to a contractor, presumably for his skill and experience in remodeling. The end result that the chef was contracting for was not the appliances, but a kitchen that looked like the one he used in his restaurant. As such, this is primarily a services contract, with the goods being incidental to the agreement.

A landlord and a tenant entered into a written residential lease agreement. The lease included a provision requiring the landlord to make necessary repairs to the leased house within a reasonable time, except for damage caused by ordinary wear and tear. During the lease term, the living room window was broken during an attempted burglary. The tenant informed the landlord of the need to replace the broken window, but the landlord did nothing. A week after the attempted burglary, the tenant severely injured his hand on the jagged glass from the broken window. On what basis may damages be awarded to the tenant? (A) Damages may be awarded on the basis of breach of the lease, but not on the basis of the tenant's injuries. (B) Damages may be awarded on the basis of the tenant's injuries, but not on the basis of breach of the lease. (C) Damages may be awarded on the basis of breach of the lease as well as on the basis of the tenant's injuries. (D) Damages may be awarded neither on the basis of breach of the lease nor on the basis of the tenant's injuries.

(C) Damages may be awarded on the basis of breach of the lease as well as on the basis of the tenant's injuries. A tenant may seek money damages for a landlord's breach of the lease. In this case, by failing to repair the broken window, the landlord breached the provision in the lease agreement requiring the landlord to make necessary repairs, and the tenant may therefore recover damages based on this breach of the lease agreement. In addition, although the general rule is that a landlord is not responsible for the injuries of a tenant or the tenant's guests caused by conditions of the premises, there exists an exception to this general rule where the landlord has covenanted to repair. Therefore, given that the landlord covenanted to repair, he is also responsible for the tenant's injuries resulting from his (the landlord's) breach of this covenant to repair.

Sally sold heroin to John. John was later stopped by police for speeding. The police searched John's car and found the heroin concealed under the rear seat. Sally was charged with illegally selling heroin. How will the court likely rule on Sally's motion to prevent introduction of the heroin into evidence? (A) Granted, because the heroin was not in plain view. (B) Granted, because the scope of the search was excessive. (C) Denied, because Sally has no standing to object to the search. (D) Denied, because the search was proper as incident to a valid full custodial arrest.

(C) Denied, because Sally has no standing to object to the search. A commonly tested area on the MBE is that of standing, both in Constitutional Law and in Criminal Procedure. In Rakas v. Illinois [439 U.S. 128 (1978)], a passenger who had no property interest in an automobile was held not to have standing to challenge a search of the vehicle as to items seized from it. This decision is founded on the principle of no reasonable expectation of privacy. To have Fourth Amendment standing, a person must show that his own rights were violated. Standing is proper if a person owns or has a right to possession of the place or thing searched, or if the place searched is the person's home. Since John, not Sally, owned the car, Sally has no standing to object to the search. This answer choice is, therefore, correct. Note further that Rakas held that being "legitimately on the premises" is insufficient grounds to assert standing by itself, without proof of some possessory interest. Also, testimony given by the defendant to assert standing may not be admitted substantively against him at trial [Simmons v. United States, 390 U.S. 377 (1968)].

Late one evening, a police department received a telephone call from an unidentified woman who reported hearing loud shrieks and screams from a neighboring home. The caller furnished the police with the address where the disturbance was taking place. When the police arrived at the home, they discovered the homeowner, bludgeoned to death. The murder weapon was a bloodstained baseball bat found near the victim's body. A subsequent investigation revealed that the homeowner had recently been separated from her husband who had since moved to another city. After questioning several of the victim's friends and relatives, the police concluded that the husband was a prime suspect in the murder. Thus, two detectives went to question the husband about the killing. When they arrived at his apartment, the detectives persuaded the landlord to let them into his apartment. Finding no one inside, they searched the apartment. Before leaving, the detectives took a box of cereal, which they planned to use for fingerprint comparison. The prints from the cereal box matched those found on the baseball bat. The police presented the fingerprint comparison to the grand jury investigating the murder. The husband is to testify before the grand jury. Before his appearance, the husband files a motion to evidence of the fingerprint comparison, contending that the evidence was illegally obtained. How should the court rule on his motion? (A) Granted, because the warrantless search of the husband's apartment was unconstitutional. (B) Granted, because the grand jury is barred from considering illegally obtained evidence. (C) Denied, because the exclusionary rule has not been extended to grand jury hearings. (D) Denied, because the landlord had the apparent authority to authorize the search of the husband's apartment.

(C) Denied, because the exclusionary rule has not been extended to grand jury hearings. In United States v. Calandra [414 U.S. 338 (1974)], the Court refused to extend the exclusionary rule to grand jury proceedings. A divided Supreme Court (6-3) noted that, "in deciding whether to extend the exclusionary rule to grand jury proceedings, we must weigh the potential injury to the historic role and functions of the grand jury against the potential benefits of the rule as applied in this context. It is evident that this extension of the exclusionary rule would seriously impede the grand jury."

A wife was divorced from her husband. The wife and the husband had a daughter who was 12 years of age. The daughter lived with the wife, but the husband frequently visited his daughter at her house. One morning, the husband came to his ex-wife's home to see his daughter. He and his daughter were outside on the patio chatting when the wife noticed her ex-husband's briefcase lying on the kitchen floor. The husband had entered the house carrying his briefcase, which he placed down before going out to see his daughter on the patio. Seeing the briefcase, the wife decided to open it. Inside she found a large envelope. She decided to keep it, thinking it contained money. Later that night after her ex-husband left, she opened the envelope and found a large amount of cocaine. The wife called the police who came and confiscated the drugs. The husband was subsequently charged with possession of cocaine. The husband has filed a motion to exclude the cocaine from being introduced into evidence. How should the court rule on his motion? (A) Granted, because the seizure violated the husband's right of privacy. (B) Granted, because the police failed to secure a search warrant before the seizure of the cocaine. (C) Denied, because the initial seizure was made by a private individual. (D) Denied, because the husband left the briefcase in the kitchen in plain view.

(C) Denied, because the initial seizure was made by a private individual. The Fourth Amendment generally protects only against governmental conduct, and not against searches by private persons. Searches by private individuals who are not acting as agents of government officials do not implicate the Fourth Amendment. An individual can claim Fourth Amendment protection only when governmental conduct infringes upon society's reasonable expectations of privacy, and further infringes upon the individual's legitimate expectations of privacy. Here, the cocaine was initially seized by the wife, who was not a government agent. She, in turn, gave the cocaine to the police. In all likelihood, since the police received the cocaine from the wife, they would not be required to secure a warrant.

The police received a tip that a man was selling drugs on a street corner. Two officers parked their unmarked car on the opposite side of the street and began to observe the man. He was approached several times by individuals and, after receiving something from them, would reach into his pocket and hand something back, at which point they would walk away. As a marked police car came down the block, the man dashed into his pickup truck to escape. Unable to start the truck, he fled across a vacant parking lot towards a small shopping mall. An officer tackled him just as he entered the parking lot. As the man attempted to get up, the officer noticed that a bag containing cocaine was lying on the ground underneath the man. The man was arrested and charged. At trial, the man moved to suppress the cocaine. How should the court rule? (A) Grant the motion, because the police had not obtained a warrant. (B) Grant the motion, because the police lacked probable cause. (C) Deny the motion, because the police had probable cause to arrest the man. (D) Deny the motion, because the drugs were in plain view.

(C) Deny the motion, because the police had probable cause to arrest the man. An individual has the right to be free of unreasonable searches and seizures under the Fourth Amendment. However, the police may acquire probable cause from any evidence that they observe. Here, based on the totality of the circumstances, the police had probable cause to arrest the man. Note that answer choice (D) is incorrect because it does not account for how the police were at a lawful vantage point when they observed the bag containing the white powder.

On Monday, Jim decided to sell his vintage car. Bill had always admired the car and inquired as soon as he saw the for-sale sign. Jim said he was firm at $5,000. Bill thought this was a good price, and told Jim he wanted it, but needed to check with his wife before spending such a large amount of money. In the meantime, Bill pulled a $100 bill from his pocket and gave it to Jim, saying, "Here's a down payment. See you tomorrow!" Jim replied, "Okay, it will be here." On Tuesday morning, Bill approached Jim with a check written out for $4,900. Jim said, "Sorry, but you're too late. I accepted an offer last night for $10,000." In an action by Bill against Jim for breach of contract, how should the court rule? (A) For Jim, because $100 is insufficient to create an option contract for a $5,000 car. (B) For Jim, because Bill's language was insufficient to create a valid option contract. (C) For Bill, because Jim created an option contract, which is irrevocable at least until the following day. (D) For Bill, because the UCC precludes Jim from selling the car to anyone other than Bill before Wednesday.

(C) For Bill, because Jim created an option contract, which is irrevocable at least until the following day. An option contract is a promise to keep an offer open for a period of time and which is supported by consideration. With an option contact, the offeror is not permitted to revoke the offer during the stated period, because with the payment, he is bargaining away his right to revoke the offer. Because Bill paid Jim $100 to keep the offer open until the next day, the offer was still open when Bill then attempted to accept it.

On January 1, a State A plaintiff sued a State B defendant and a State A defendant in a State A state court. The plaintiff's claim against each defendant exceeded $100,000. The case went through a series of pretrial conferences, and after a long, drawn-out discovery process, the plaintiff settled her case against the State A defendant on January 20 of the following year. The case has now been certified to proceed to trial. At this point, what recourse does the State B defendant have? (A) He could remove the case to federal court, because he has 30 days from when the grounds for removal become apparent. (B) He can remove because the plaintiff, by settling with the State A defendant, provided complete diversity. (C) He cannot remove at this point, because he is time barred. (D) He cannot remove the case, because removal can only occur before the close of discovery.

(C) He cannot remove at this point, because he is time barred. It is true that a defendant can remove within 30 days of when the grounds for removal become apparent. However, in diversity cases, removal cannot occur more than one year after the date when the suit was filed. Here, more than one year has passed. Therefore, the defendant can no longer remove the case to federal court.

A senior associate in a law firm was conducting spot checks of other associates as they left the office to determine who had taken some documents that were missing from the firm's library. A young associate was leaving his office for the day when he was stopped by the defendant, who asked him to open his briefcase. The senior associate explained that some files from the law office had been stolen. Startled, the young associate refused to permit the senior associate to look in his briefcase. The young associate said that he had personal papers in his briefcase and he didn't want the senior associate to see them. Having his suspicions aroused, the senior associate raised his clenched fist and exclaimed, "If you don't hand over that briefcase, I'm going to punch you in the mouth." The young associate, who, unknown to the senior associate, suffered from a serious heart condition, had a seizure, and died as a result of fright produced by the threatened attack. Which of the following is the most serious crime for which the senior associate should be found guilty? (A) Assault. (B) Voluntary manslaughter. (C) Involuntary manslaughter. (D) Murder

(C) Involuntary manslaughter. As a general rule, whenever an intentional battery or assault results in an unintended death, the defendant is guilty of involuntary manslaughter. LaFave notes that a criminal assault, like a criminal battery, is an unlawful act malum in se. Therefore, if the defendant approaches close to another person intending to strike him but not to kill him, and the latter, who, unknown to the defendant, possesses a weak heart, has a heart seizure and dies as a result of fright produced by the threatened attack, the defendant is guilty of manslaughter, though he never touched the victim. See Regina v. Dugal [4 Que. L.R. 350 (Q.B. 1878)], referred to by LaFave in Criminal Law, pg. 601.

A defendant had an argument with his neighbor. As they were quarreling, the defendant pulled out his penknife intending only to frighten the neighbor. The defendant accidentally slightly nicked the neighbor's arm with the knife. Unknown to the defendant, his neighbor was a hemophiliac who then died from the cut. What is the most serious crime of which the defendant can be convicted? (A) Murder. (B) Voluntary manslaughter. (C) Involuntary manslaughter. (D) Battery.

(C) Involuntary manslaughter. Remember the distinction between voluntary and involuntary manslaughter. Voluntary manslaughter involves the intentional killing of another in the heat of passion, brought about by adequate provocation. Provocation is adequate if it would cause a reasonable person would to lose self-control. Except for the mitigation of this objectively reasonable emotional condition, the intentional killing would be murder. On the other hand, involuntary manslaughter is an unintended homicide where there is an unlawful killing in the commission of an unlawful act not amounting to a felony. Here we have a battery (a misdemeanor at common law) that results in an unintended death. The most serious crime that the defendant can be convicted of is involuntary manslaughter.

A baseball fanatic became enraged when a fan for the other team reached out into the field of play and touched a ball, causing the winning run to score. As the spectators were leaving the stadium, the fanatic saw the other fan, who had touched the ball, walking to his car. The fanatic walked up behind the other fan and punched him in the back of the head. The other fan, who had been drinking all day, was immediately knocked out and died as soon as his head hit the pavement. This jurisdiction defines assault as: (1) the causing of physical harm to another (categorized as felony assault); or (2) acting in a threatening manner to put another in fear of immediate harm (categorized as misdemeanor assault). The fanatic was arrested for the homicide of the other fan. Which of the following is the most serious crime for which the fanatic should be found guilty? (A) Assault. (B) Voluntary manslaughter. (C) Involuntary manslaughter (D) Felony murder.

(C) Involuntary manslaughter. As a general rule, whenever an intentional battery or assault results in an unintended death, the defendant is guilty of involuntary manslaughter. This is a good example of misdemeanor manslaughter.

An employee successfully negotiated a lucrative contract for her employer. As a result, her employer orally promised her a $10,000 bonus payable at the end of the year because of the employee's "good work." At the end of the year, the employer informed the employee that the company's profits were not as large as he expected, so the promised bonus would not be paid. Which of the following is the legal effect of the employer's promise to pay the bonus to the employee? (A) It is enforceable, because the employee conferred a material benefit on the employer by negotiating the lucrative contract. (B) It is enforceable, because the employer was morally obligated to pay the bonus. (C) It is unenforceable, because it was not supported by legally sufficient consideration. (D) It is unenforceable, because it was not in writing.

(C) It is unenforceable, because it was not supported by legally sufficient consideration. Donative promises generally are not enforceable unless supported by consideration or a consideration substitute (like promissory estoppel). Here, the employer's promise was made in exchange for work already performed by the employee. Past consideration is not considered to be good consideration, so the employer's promise to pay the bonus would be unenforceable.

In order to combat a rise in organized crime within the state, an anti-organized crime task force was created. After reviewing the previous attempts by various state and local agencies to infiltrate the criminal organizations, the state also adopted a policy that prohibited the assignment of female agents to the undercover infiltration division of the task force. Over the course of 10 separate attempts, seven agents were killed attempting to go undercover, including all five women that were sent in, none of whom managed to make even an initial penetration into the crime families. While undercover assignments were a very successful way to advance to a higher position, it was because of the state's concern with the safety and well-being of its female officers and agents and the demonstrated misogyny of the crime families under investigation that it adopted such a policy. One female agent, desiring to be a member of the undercover division in order to help her achieve her next promotion, submitted an application for the undercover division. After her application was rejected, the agent sued the state in federal court to enjoin enforcement of its stated policy on the grounds that it was unconstitutional. As a matter of constitutional law, which of the following results in this suit is most appropriate? (A) Judgment for the agent, because the particular classification contained in this policy is not necessary to further a compelling interest. (B) Judgment for the agent, because the terms and conditions of state government employees are privileges or immunities of state citizenship that may not be abridged by the state on the basis of gender. (C) Judgment for the state, because the policy is substantially related to the advancement of an important state interest. (D) Judgment for the state, because the state has articulated a rational basis for this classification and, therefore, a court may not substitute its judgment for that of responsible state officials.

(C) Judgment for the state, because the policy is substantially related to the advancement of an important state interest. Gender classifications are subject to intermediate scrutiny, which puts the burden of persuasion on the government to demonstrate that the classification (or discrimination) is substantially related to an important government interest. Here, there is a record showing that such an assignment carries a very high risk of death for female agents, due in large part to the nature of the groups being infiltrated. As such, this classification will likely be found to substantially relate to an important state interest.

A woman brought a negligence claim against her doctor for failing to warn her of the risks associated with an elective surgery. The surgery was a success, but there was a 50% chance that the woman would be unable to have children, something she had mentioned wanting to the doctor on several occasions. During discovery, several depositions were scheduled, including the plaintiff's, which was scheduled first. At her deposition, the woman recounted conversations she had with the doctor, her memories of the procedure, and admitted that she was currently pregnant. Immediately following the deposition, and before the remaining depositions, the doctor moved for summary judgment on the negligence claim. How should the court rule on the doctor's motion? (A) Motion denied, because the doctor's failure to warn caused the woman to consent to the surgery. (B) Motion denied, because the doctor breached his duty to the woman by failing to warn her of a material issue. (C) Motion granted, because the doctor's failure to warn did not result in sufficient injury to the woman. (D) Motion granted, because the surgery was elective.

(C) Motion granted, because the doctor's failure to warn did not result in sufficient injury to the woman. In order to recover, the woman needs to show damages. Here, the doctor may have breached a duty to warn of the risks associated with the surgery. However, the risk was that she may not be able to get pregnant. The fact that she is currently pregnant serves to negate the element of damages.

An ice cream manufacturer and a small restaurant entered into a contract whereby the restaurant would buy all the ice cream it needed from the ice cream manufacturer, and the ice cream manufacturer would sell as much ice cream as it wanted to the restaurant at their current market price. The restaurant sent its first order to the ice cream manufacturer. However, the ice cream manufacturer refused to deliver any ice cream to the restaurant, citing purchase orders it had just received from a large national chain of restaurants. If the restaurant sued the ice cream manufacturer, which of the following is most accurate? (A) A valid contract was formed, because there was valid consideration provided and the parties both acted in good faith. (B) A valid contract was formed, because there was mutual assent to clearly defined terms understood within the industry. (C) No contract exists, because the agreement was illusory. (D) No contract exists, because the parties failed to include a quantity term.

(C) No contract exists, because the agreement was illusory. This contract is illusory, because the restaurant is bound to buy all of their ice cream from the ice cream manufacturer, while the manufacturer is bound to nothing at all. In fact, if it so chooses, the manufacturer does not have to sell any ice cream to the restaurant, and can choose to sell instead to another vendor that offers more money. As such, this contract is not enforceable.

Acme became involved in a labor dispute with the Steamrollers' Union (the entity that ordinarily supplied workers for Acme's plant). Acme commenced an action against the Steamrollers' Union in the appropriate U.S. District Court, claiming $75,000 in damages as a consequence of the union's conduct in harassing and intimidating non-union workers in violation of the Federal National Labor Relations Act. Acme is an Indiana corporation, and the union (an unincorporated association) has members who are domiciled in every state except New York and New Jersey. The Steamrollers' Union answered by denying Acme's allegations and filing a $15,000 counterclaim, which asserted that Acme had deliberately made false accusations about the union to the local papers for the purpose of obtaining favorable press coverage. If the Steamrollers' Union moves to dismiss for lack of subject-matter jurisdiction, should the court grant the motion? (B) Yes, because there is no diversity. (A) Yes, because Acme has not claimed monetary damages in excess of $75,000. (C) No, because subject-matter jurisdiction is satisfied. (D) No, because a state claim has been asserted in a federal court.

(C) No, because subject-matter jurisdiction is satisfied. Because Acme is asserting a federal claim against the Steamrollers' Union, subject-matter jurisdiction exists, even though the amount in controversy does not exceed $75,000 and without regard to diversity.

A buyer ordered 1,000 widgets from a seller for immediate delivery. The seller responded by shipping 800 widgets, along with an accompanying notice to the buyer explaining that the seller did not have an adequate inventory to ship 1,000 widgets, and was therefore shipping 800 widgets as an accommodation to the buyer in light of the buyer's urgent need. Which of the following is the most accurate statement? (A) A contract was formed under the mailbox rule when the seller shipped the widgets. (B) A contract will be formed once the widgets arrive at the buyer's address. (C) No contract exists, because the notice of accommodation operates as a counteroffer. (D) No contract exists, because the buyer's accommodation is not an acceptance.

(C) No contract exists, because the notice of accommodation operates as a counteroffer. The UCC governs contracts for the sale of goods. If the contract falls under the UCC, then the UCC's provisions will trump any contrary common law rules. Under the UCC, an offer can be accepted by the offeree communicating that acceptance or by the prompt shipment of the goods. When the seller ships nonconforming goods, this operates as an acceptance of the offer and a simultaneous breach of contract. However, the shipment of nonconforming goods will not constitute an acceptance of the offer if the seller notifies the buyer that the shipment is offered only as an accommodation to the buyer. In such circumstances, the shipment instead constitutes a counteroffer, which the buyer is free to accept or reject. Here, the seller notified the buyer of an accommodation in light of the buyer's urgent need. As such, the notice will operate as a counteroffer.

A heartsick lover knew he was being foolish, but he just could not stop himself. The woman he was in love with was so beautiful, and she was so lonely with her husband in prison. She had told the lover that her husband was extremely jealous. She said that her husband was in prison because he had shot her last lover when he learned of their affair. About four months after he started seeing the woman, the lover received a telephone call from her. She seemed distraught, and cried, "My husband escaped from prison this evening, and he knows about us!" As expected, the husband called on the lover that evening. The lover shot the husband and was prosecuted for his murder. The lover claims self-defense. At trial, he attempts to testify to the woman's statement that her husband was in prison because he had shot her last lover when he learned of the affair. If the statement is offered to prove that the husband was in prison because he had shot the woman's lover when he learned of the affair, should the statement be admitted? (A) Yes, under the state-of-mind exception. (B) Yes, as nonhearsay. (C) No, as hearsay not within any exception. (D) No, because it is self-serving.

(C) No, as hearsay not within any exception. Under FRE 801(c), hearsay is: (1) any statement other than a statement made by the declarant while testifying at the trial or hearing in which the statement is proffered; (2) which is offered into evidence in order to prove the truth of the matter asserted. Here, this is hearsay because the question tells us that the lover offers the statement for the express purpose of proving the truth of the matter asserted, i.e., that the husband had killed his wife's prior lover. The statement does not fall within any exception, thus it is not admissible.

Paula and Pete were hit by a truck driven by an employee of the D Corporation, a company that manufactures widgets. The accident occurred in California, where Paula and Pete were living at the time. D Corporation is incorporated in Delaware maintains its principal manufacturing plant in Florida, and is controlled from its offices in Michigan. After the accident, Paula and Pete moved to Miami, Florida, where they planned to retire. After moving, they contacted a lawyer, who filed suit against D Corporation in the appropriate U.S. District Court where Paula and Pete lived, alleging diversity jurisdiction. The suit alleged that each plaintiff had suffered personal injuries in excess of $75,000. D Corporation moved to dismiss the case for lack of subject-matter jurisdiction. Should D Corporation's motion be granted by the district court? (C) No, because diversity subject-matter jurisdiction exists. (B) Yes, because the plaintiffs were citizens of California when they were injured. (D) No, because D Corporation is a citizen of Delaware only. (A) Yes, because D Corporation's headquarters are in Florida.

(C) No, because diversity subject-matter jurisdiction exists. The requirements for diversity subject-matter jurisdiction are both satisfied based on these facts: (1) the plaintiffs are citizens of Florida, and D Corporation is a citizen of Delaware and Michigan (a corporation is deemed to be a citizen of the states in which it is incorporated and in which the officers control the corporation); and (2) each plaintiff is claiming damages in excess of $75,000. Note that it is irrelevant that the corporation's principal manufacturing plant is in Florida. Although the diversity statute links corporate citizenship to the corporation's principal place of business, the Supreme Court has interpreted that phrase to mean the corporation's "nerve center"--i.e., the place where the corporation is controlled.

Knowing that his niece was looking for a good used car, an uncle sent an email to his niece one Friday afternoon, promising to sell her his car for $1,000 if she accepted before the end of that weekend. After reading the email, the niece sent her uncle a reply email that stated, "The price is a little high. I'll give you $800." The uncle responded via email, "I will not accept $800; it is worth more than that." The next day, the niece emailed her uncle and said, "I changed my mind, I will pay $1,000." On Sunday, the uncle responded that he refused to sell the niece his car. If the niece brings a cause of action against her uncle to enforce the agreement, will she prevail? (A) Yes, because she accepted the uncle's terms. (B) Yes, because she accepted the uncle's terms before the end of the weekend. (C) No, because she made a counteroffer. (D) No, because the uncle rejected her counteroffer.

(C) No, because she made a counteroffer. An offeree's power of acceptance can be terminated if the offeree refuses to accept the offer. A counteroffer made on the same subject matter operates to simultaneously reject the initial offer, terminating the offeree's power of acceptance, while making a new offer. Not all statements or questions about an offer are considered counteroffers. An offeree may test the waters by making a "mere inquiry" about the offeror's willingness to negotiate without creating a counteroffer and terminating the power of acceptance. Here, the niece's email, "The price is a little high. I'll give you $800." goes beyond a mere inquiry, as it modified the terms of the initial offer from $1,000 to $800. By doing so, the niece effectively rejected the offer for $1,000 and made a counteroffer for $800. This terminated her power of acceptance.

The plaintiff is an Alabama corporation that markets computer products of the defendant, a nationwide computer manufacturer with its headquarters in New York. The marketing agreement contained a forum-selection clause providing that any disputes arising out of the contract could only be brought in a court located in the Southern District of New York. After business relations took a turn for the worse, the plaintiff sued the defendant in the Northern District of Alabama. The complaint alleged that the defendant had breached the dealership agreement, causing more than $1 million in damages. The defendant moved the district court to transfer the case to the Southern District of New York. The Alabama court refused to do so, finding that transfer was controlled by Alabama law and that Alabama did not view contractual forum-selection clauses as enforceable. Specifically, the court found that Alabama disfavored such clauses in order to protect state court jurisdiction. Was the Alabama court's ruling correct? (A) Yes, because the Alabama court based its ruling on Alabama law. (B) Yes, because such forum-selection clauses are invalid. (C) No, because the clause was evidence of what the parties believed was convenient. (D) No, because the defendant was a resident of New York.

(C) No, because the clause was evidence of what the parties believed was convenient. Venue is considered appropriate where either party consents to the venue. Importantly, contractual forum-selection clauses are evidence of what the parties believed would be a convenient forum. Here, the contract that served as the basis of the suit had a forum-selection clause stating that all claims had to be brought in the Southern District of New York. While the clause was not dispositive, the court would take it into account with other factors regarding appropriate transfer, such as the convenience of witnesses, ease of access to evidence, the location of the claim, and local interest of the community.

Acting on a tip that a suspected drug dealer was selling drugs out of a hotel room, police asked the hotel manager for permission to enter the room. The manager gave the police permission to enter and a key to the room. When the police entered the room using the key, they found the suspected drug dealer in the middle of packaging bags of a white powder. The police arrested the suspected drug dealer and seized the powder, which subsequent forensic tests showed to be cocaine. When the drug dealer was charged with possession of narcotics with intent to distribute, he challenged the admissibility of the cocaine at trial. Should the court admit the cocaine into evidence? (A) Yes, because the cocaine was in plain view. (B) Yes, because the hotel manager consented to the search. (C) No, because the drug dealer had a reasonable expectation of privacy in the hotel room. (D) No, because there were no exigent circumstances present.

(C) No, because the drug dealer had a reasonable expectation of privacy in the hotel room. The Fourth Amendment generally requires a warrant before the police may search a person's home. The Supreme Court has held that the Fourth Amendment's warrant requirement applies in a number of areas outside the home, including a rented hotel room, and that a hotel manager lacks authority to consent to the search of a guest's room [Stoner v. California, 376 U.S. 483 (1964)]. None of the other exceptions to the warrant requirement apply here, so the search was unreasonable. The exclusionary rule thus applies to the cocaine, making it inadmissible.

Police investigating the unlawful sale of copyrighted videos received a tip that much of the copying was taking place in the offices of a small law firm in the suburbs. Based upon the sworn statement of a paralegal at the firm, stating that the copying operation was taking place in the firm's mailroom, police officers obtained a warrant to search the premises for evidence of an illegal video piracy operation. In executing the warrant, the police approached the loading dock area of the firm and, without alerting the occupants of the building, kicked open the back door and burst inside with guns drawn. The subsequent search of the law firm's offices produced a large number of illegally copied videos and the arrest of most of the members of the criminal operation. Before trial, the defendants seek to have the evidence of the search excluded as violating their Fourth Amendment rights. Must the court exclude the evidence? (A) Yes, because the police did not knock and announce their presence before executing the warrant. (B) Yes, because the manner in which the warrant was executed was unreasonable under the Fourth Amendment. (C) No, because the knock-and-announce rule does not always bar the admission of exclusion of evidence obtained in violation of the rule. (D) No, because the police do not need to knock and announce their presence before executing a warrant.

(C) No, because the knock-and-announce rule does not always bar the admission of exclusion of evidence obtained in violation of the rule. In executing search warrants, police must generally knock and announce their presence to the inhabitants of the place to be searched before entering forcibly. The main exception to this rule is the presence of exigent circumstances, especially those in which the police could be harmed or evidence could be lost if the police were to knock and announce their presence. In this case, there was no evidence of exigent circumstances, and the police violated the Fourth Amendment by failing to knock and announce. However, unlike most violations of the Fourth Amendment, the knock-and-announce rule does not automatically trigger the exclusionary rule, but requires instead a broader inquiry that takes into account the social costs of exclusion of the evidence. This answer is therefore correct.

Police investigating some burglaries received a tip that the stolen goods were being held at 23 Maple Avenue. They obtained a warrant, but a typographical error on the warrant authorized a search of 32 Maple Avenue instead. The police executed the warrant properly at 32 Maple Avenue. In the course of their search, the police discovered a large bag of heroin on a chair in an upstairs bedroom. The police apologized for searching the wrong house and then arrested the homeowners for felony possession of an unlawful substance. Before trial, the homeowners challenged the admissibility of the heroin. Should the court exclude the heroin as evidence? (A) Yes, because the police lacked the warrant to search the home. (B) Yes, because the police search of the home was unreasonable. (C) No, because the police acted in good faith when searching 32 Maple Avenue. (D) No, because the heroin was in plain view and the police thus did not need a warrant.

(C) No, because the police acted in good faith when searching 32 Maple Avenue. Although the Fourth Amendment normally requires a valid warrant for the police to search a home, one of the exceptions to the warrant requirement is the so-called good-faith doctrine. This doctrine provides that evidence that is unlawfully obtained due to police error (even negligent error) is admissible if the police mistake was due to good faith. In this case, the police thought they were searching the right house, but the typographical error on the warrant led them to search the wrong house in good faith. The proceeds of such a search are therefore admissible.

Police received a tip from an informant that a ring of thieves were hoarding stolen property in the house owned by the ringleader's uncle. The ringleader frequented the uncle's house to sell the stolen goods out of the basement and could be found there every evening after 10:00 pm. The police obtained an arrest warrant and went to the uncle's house to arrest the ringleader the next evening. The uncle refused entry to the police, who pushed past him to find the ringleader. When police entered the living room, they observed bricks of marijuana on the table. Police arrested the uncle and the ringleader and charged them both with narcotics offenses. The uncle's motion to suppress the evidence of the drugs was granted. The ringleader files a similar motion seeking suppression of the drugs. Should the ringleader's motion be granted? (A) Yes, because the police did not have a valid search warrant for the uncle's house. (B) Yes, because there was no probable cause to believe that the drugs belonged to the ringleader. (C) No, because the ringleader had no standing to contest the legality of the search. (D) No, because the drugs were observed in plain view and there were exigent circumstances.

(C) No, because the ringleader had no standing to contest the legality of the search. Standing under the 4th Amendment is personal to the defendant. Thus, in order for the ringleader to have standing to vindicate his own rights under the 4th Amendment, he must have a reasonable and legitimate expectation of privacy in the place searched or the item seized. The ringleader does not have a legitimate expectation of privacy in someone else's home and therefore, does not have standing.

A landlord and a doctor entered into a written lease agreement for the lease of an apartment owned by the landlord. Under the terms of the lease agreement, the doctor was to rent the apartment from the landlord for a period of one year. On the day the lease commenced, the doctor found he could not gain access to the apartment, because the landlord had changed the locks. Has the landlord breached his duty to deliver possession of the premises to the doctor? (A) Yes, under the American rule, because the landlord has not delivered actual possession of the premises to the doctor. (B) Yes, under the minority rule, because the landlord has constructively evicted the doctor. (C) No, under the minority (American) rule, because the doctor has a legal right to possess the premises. (D) No, under the majority (English) rule, because a lease for a term of years may be terminated at will.

(C) No, under the minority (American) rule, because the doctor has a legal right to possess the premises. Under the facts presented, the doctor has a legal right to possess the premises by virtue of having entered into the lease agreement with the landlord, but the doctor does not have actual possession of the premises because he has been locked out. Under the majority (English) rule, a landlord fulfills his duty to deliver possession of the premises by delivering actual possession of the leased premises. In contrast, under the minority (American) rule, a landlord fulfills his duty to deliver possession of the premises merely by delivering the legal right to possession. Thus, under the American rule, the landlord has not breached his duty to deliver possession.

A defendant was on trial for drug distribution. The arresting officer was called by the prosecution to describe in detail the events that led to the defendant's arrest. The officer described the night in question, where he was, and what he observed. He was asked to provide a list of what he confiscated from the defendant after he was arrested. The officer testified that the defendant has a large amount of money and many small packets of drugs in several pockets throughout his clothing. When asked specifically about the number of drugs packets, the officer was unable to recall. The officer was then shown his police report. The officer quickly flipped to the third page of the report, then looked at the prosecutor. The prosecutor asked if the officer was now able to remember how many bags of drugs were taken from the defendant. The officer indicated that he was, but before he could answer the question, the defense attorney objected to the officer's testimony. How should the court rule? (A) Sustain the objection, because the officer lacks a present memory of the incident. (B) Sustain the objection and require the report to be admitted into evidence, because the jury is just as capable as reading the report as the officer. (C) Overrule the objection, because the officer's memory was refreshed. (D) Overrule the objection, because the report is admissible as a past recollection recorded.

(C) Overrule the objection, because the officer's memory was refreshed. The prosecutor is refreshing the officer's recollection. To do so, he can use anything available, because it is not being admitted as an exhibit, and so it is not used substantively. If the attempt to refresh fails, then extrinsic evidence is required, at which point past recollection recorded would be a valid hearsay exception. However, because the officer's memory was refreshed, the judge should overrule the defense attorney's objection.

A plaintiff wanted to sue two defendants for damages stemming from the defendants' reprinting of her novel in violation of federal copyright laws. The defendants reprinted and sold the novel in the Northern District of State A. One defendant resided in the Eastern District of State A, while the other resided in the Western District of State A. The plaintiff resided in the Northern District of State B. Where would venue be proper for this action? (A) The Northern District of State A only. (B) The Western District of State A or the Eastern District of State A. (C) The Northern District of State A, the Western District of State A, or the Eastern District of State A. (D) The Northern District of State A, the Western District of State A, the Northern District of State A, or the Eastern District of State B.

(C) The Northern District of State A, the Western District of State A, or the Eastern District of State A. Pursuant to 28 U.S.C. Section 1391, venue is appropriate: (1) where any defendant resides, provided that all defendants are within the same state; (2) in the district where a substantial portion of events occurred; or (3) in the district where any defendant can be found, if there is no other district in which the action could be brought. Here, the action was based on a federal copyright violation, a federal question. Venue would be appropriate in the Northern District of State A because that is where the defendants reprinted and sold the novel. Also, venue would be proper in the Western and Eastern Districts of State A because those are the districts where the defendants live.

A man from State A rented a car and drove to State B for a work meeting. On his way back to State A, the man ran into the back of a State B driver's car, injuring the State B driver. The State B driver then brought suit against the State A man in State B federal court claiming $100,000 in damages. The State A man argued that the case should be dismissed because the State B federal court lacked sufficient jurisdiction over him. Which of the following statements is most accurate? (A) The State B court should dismiss the case, because driving the car in State B does not provide sufficient minimum contact for the court to have jurisdiction over the State A man. (B) The State B court should dismiss the case, because the rental of the car occurred in State A. (C) The State B court should hear the case, because the man impliedly consented to being sued in State B by driving a car in the jurisdiction. (D) The State B court should hear the case, because diversity is complete and the amount in controversy is over $75,000.

(C) The State B court should hear the case, because the man impliedly consented to being sued in State B by driving a car in the jurisdiction. Consent can be implied, and one of the most common forms of implied consent is by driving on the roads of a state. Courts consider persons to have given implied consent to the laws regulating roads, and thus, if a defendant has a car accident on the road in that state, a court has personal jurisdiction over the defendant in an action arising from that accident.

A seller sent the following terms in a writing to a buyer: "I will provide 100 units of furniture for a price of $50 per unit. This offer is good for 60 days from today." The seller dated the letter and signed it at the bottom. The buyer received the letter and immediately wrote a response accepting the seller's terms, but did not immediately send the letter. A week after sending his letter to the buyer, the seller received an offer from a furniture wholesaler offering to buy as much furniture as the seller could make at $55 per unit. The seller wanted to accept the wholesaler's offer, but was concerned about the outstanding letter to the buyer. Which of the following statements is most accurate? (A) The seller can revoke his offer to the buyer as long as the buyer has neither accepted nor changed position in reliance on the offer. (B) The seller can revoke his offer, because there was no consideration provided that would make the offer irrevocable. (C) The buyer can accept the offer at any time within 60 days from the date of the letter, and the seller cannot revoke the offer. (D) The buyer has three months from the date of the letter to accept, because firm offers are irrevocable for 90 days.

(C) The buyer can accept the offer at any time within 60 days from the date of the letter, and the seller cannot revoke the offer. Under the UCC, a merchant can make a firm offer, which is an irrevocable offer, to buy or sell goods. No consideration is needed to make the firm offer irrevocable. In order to create a firm offer, the following requirements must be met [UCC Sec. 2-205]: (1) the offer must be made by a merchant; (2) the offer must be made in a writing signed by the merchant; and (3) the offer must expressly state by its terms that it will be held open. A firm offer can be made irrevocable for a maximum of 90 days; after that, the parties must create an option contract (which requires consideration) in order to make the offer irrevocable for a longer period. Here, because all of the requirements are met, the seller's offer to the buyer was an irrevocable firm offer. As such, the buyer can accept at any time within the 60-day period, and the seller cannot revoke the offer during this time.

A taxicab driver stopped at a convenience store to buy a snack. While he was handing his money to the store clerk at the cash register, a police officer entered the store. The taxi driver had received several traffic tickets from the police officer, and he felt that the police officer had been very rude to him. Seeking to gain some revenge, the taxi driver immediately began to taunt and berate the police officer, calling him a variety of profane and insulting names. The police officer was going through a series of personal difficulties and was feeling very emotionally fragile. As the taxi driver's insults rained down on him, the police officer suddenly snapped. He whipped out his gun and fired a shot at the taxi driver. The bullet missed the taxi driver but hit the store clerk, who was standing behind the taxi driver. The clerk survived the shooting but suffered a gunshot wound to his shoulder. In an action by the injured clerk against the taxi driver to recover for his personal injuries, what is the likely outcome? (A) The clerk will recover, because the taxi driver's conduct was the proximate cause of the clerk's injuries. (B) The clerk will recover, because it was foreseeable that the taxi driver's conduct would result in the clerk being shot. (C) The clerk will not recover, because the shooting was not a foreseeable consequence of the taxi driver's conduct. (D) The clerk will not recover, because the police officer intended to shoot the taxi driver, not the store clerk.

(C) The clerk will not recover, because the shooting was not a foreseeable consequence of the taxi driver's conduct. The taxi driver did not intend for the shooting to occur, so an intentional tort claim would not be successful here. The plaintiff could claim that the taxi driver acted negligently, but will be unable to prove that the taxi driver's conduct was a proximate cause of the shooting and resulting injury. Proximate cause exists only if the plaintiff was within the general class of people that one could reasonably foresee being hurt, and the plaintiff's injury was within the general type of harm that one could reasonably foresee occurring. Here, even if it was negligent for the taxi driver to insult and taunt the police officer, it was not foreseeable that the store clerk would be shot as a result of that negligence. In other words, no one could have foreseen that the police officer would snap and try to shoot someone who insulted him.

On July 1, a college senior entered into an oral agreement with a landlord to lease an apartment near her college campus for $750 per month for a period of one year, starting immediately. At the end of the year, having found a job nearby following her graduation, the college senior began looking for a new apartment. Because she did not find one right away, however, she remained in the apartment through the month of July. She did not, however, pay any rent to the landlord. What is the relationship between the parties as of August 1? (A) A month-to-month periodic tenancy has been formed. (B) Because the original lease agreement was not in writing, the Statute of Frauds will operate to invalidate the agreement, resulting in an at-will tenancy. (C) The college senior is a tenant at sufferance, and the landlord may immediately evict her. (D) The landlord cannot evict the college senior without providing at least 30 days' notice but may collect rent from the senior according to the lease terms.

(C) The college senior is a tenant at sufferance, and the landlord may immediately evict her. A tenancy at sufferance occurs when a tenant remains in possession of the leased premises ("holds over") after the end of the lease term. A tenancy at sufferance is not a true tenancy. In most states, if a residential tenant holds over, the landlord may recover possession of the premises and receive the reasonable rental value for the holdover period. Here, the college senior remained in possession of the apartment after the end of the lease term. If the college senior had paid her rent for the month of July on time and the landlord had accepted it, a month-to-month periodic tenancy would have been formed, requiring the landlord to provide notice before terminating the tenancy. However, because she did not do so, she remains on the property as a tenant at sufferance, and the landlord may immediately evict her and collect the reasonable rental value of the apartment for the month of July.

A police officer sees a robber point a pistol at a couple and take items from them before fleeing on foot. The officer chases the robber on foot. The robber, with gun still in hand, runs into his nearby apartment. This apartment is being leased by the robber. The officer follows the robber into the apartment and the officer realizes that there is no rear exit. The officer begins to search the apartment in an attempt to locate the robber. When the officer opens a hallway closet door, he sees a large quantity of drugs on the floor. The officer then finds the robber under the bed in the master bedroom. The officer arrests the robber for the armed robbery and possession of drugs with the intent to distribute. At a pre-trial motion, the robber's attorney argues that the drugs are inadmissible evidence because the officer did not have a warrant to search the property. How will the court rule? (A) The court should suppress the evidence, because the officer needed a warrant to enter the apartment. (B) The court should suppress the evidence, because the drugs were not in plain view. (C) The court should admit the evidence, because the officer was in hot pursuit. (D) The court should admit the evidence, because there were exigent circumstances.

(C) The court should admit the evidence, because the officer was in hot pursuit. Police may enter & search a private dwelling while in reasonable pursuit of a fleeing suspect. In such a circumstance, what would otherwise have been a warrantless search becomes lawful because the police are in actual "hot pursuit" of a fleeing suspect and actively attempting to apprehend him. While in that pursuit, they may seize not only evidence of the crime for which they are chasing the suspect but also any additional contraband that they find. Here, the officer was in hot pursuit of the robber and while attempting to apprehend him, the officer chased the robber into an apartment. The drugs were located in a closet while the officer was looking for the robber. Therefore, the drugs were lawfully seized contraband and can be admitted as evidence.

A man and woman attended a local art show. The art show featured a world-renowned photographer who was famous for taking pictures of people he saw on his daily walk around the city. The artist would then paint slogans on the photos. As the man and woman walked around the show, they discovered that the artist had taken a picture of them while they were dressed up in matching American flag shirts. Below the couple was painted the slogan "War Pigs." The woman became so upset that she fainted, injuring herself when she hit the ground. The man and woman joined as plaintiffs under Rule 20 and sued the artist in federal court. The man was a citizen of Connecticut, the woman was a citizen of Rhode Island, and the artist was a citizen of New York. The man's claim was for $10,000 under the Federal Misuse of Likeness Act, which made it a federal crime to use another person's image for monetary gain. The woman's claim was for $20,000 under state laws for intentional infliction of emotional distress. The artist moved to dismiss the woman's claim for lack of subject-matter jurisdiction. How should the court rule? (D) The court should deny the motion, because the woman's claim, in effect, alleged a violation of a federal law. (A) The court should grant the motion, because the amount in controversy was only $20,000. (B) The court should grant the motion, because the woman's claim was based on state law. (C) The court should deny the motion, because the woman's claim arose from the transaction or occurrence underlying the man's claim.

(C) The court should deny the motion, because the woman's claim arose from the transaction or occurrence underlying the man's claim. A federal court must have subject-matter jurisdiction over each cause of action alleged in the complaint. Nevertheless, claims that do not provide subject-matter jurisdiction may be combined with appropriate claims through the court's supplemental jurisdiction, provided that they arise out of the same case or controversy [28 U.S.C. 1367]. Claims arise out of the same case or controversy when they derive from: (1) the same transaction or occurrence; or (2) a common nucleus of operative facts. Here, the man was suing under a federal statute; thus, his claim was a federal question properly heard in federal court. The woman was suing based on a state law claim. While the woman's Rhode Island citizenship was diverse from the artist's New York citizenship, the amount in controversy was less than $75,000. Normally, her suit would be inappropriate for federal court. However, both the man's and the woman's claims arose out of a common nucleus of operative facts, because they were both based on the artist's use of their likenesses in his art show. The fact that the man and the woman suffered different injuries does not change the analysis.

A local politician and his girlfriend attended a local art show. The art show featured a world-renowned photographer who was famous for taking pictures of people he saw on his daily walks around the city. The artist would then paint slogans on the photos. As the politician and his girlfriend walked around the show, they discovered that the artist had taken a picture of them while standing in front of a foreign consulate. Below the couple was painted the slogan "Commie Pigs." The girlfriend became so upset that she fainted, injuring herself when she hit the ground. The politician and girlfriend sued the artist in federal court. The politician was a citizen of State C, the girlfriend was a citizen of State B, and the artist was a citizen of State A. The politician's claim was for libel in the amount of $70,000. The girlfriend's claim was for $20,000 under state laws for intentional infliction of emotional distress. The artist moved to dismiss the claims for lack of subject-matter jurisdiction. Which of the following is most accurate? (A) The court should deny the motion, because a federal court must hear the case as it involves defamation of a public figure. (B) The court should deny the motion, because there is complete diversity and the couple can aggregate their claims against the artist. (C) The court should grant the motion, because the federal court does not have jurisdiction over either claim. (D) The court should grant the motion as to the girlfriend's claim, because she is not a public figure.

(C) The court should grant the motion, because the federal court does not have jurisdiction over either claim. A federal court must have subject-matter jurisdiction over each cause of action alleged in the complaint. While it is true that the politician is a public figure, there must be proper subject-matter jurisdiction for a federal court to hear the case. There is no federal question presented simply by way of the politician being a public figure. While there is complete diversity, these two individuals cannot aggregate their claims. Multiple plaintiffs cannot aggregate their claims to meet the amount-controversy requirement.

A defendant shot and killed a victim at close range with a revolver. After the shooting death, the defendant was arrested and charged with first-degree murder. In this jurisdiction, first-degree murder is defined as "knowingly or willfully causing the death of another human being." Which of the following situations, if believed by the jury, would most likely result in the defendant's acquittal of the first-degree murder charge? (A) The victim, who was the defendant's brother, suffered from an incurable case of cancer and asked the defendant to kill him to put an end to his pain and suffering. (B) The killing was not done with premeditation or deliberation. (C) The defendant intended to kill himself, but the bullet grazed his head and struck and killed the victim. (D) The defendant intended to kill another man, but the victim unknowingly stepped in front of the other man and was struck and killed by the bullet.

(C) The defendant intended to kill himself, but the bullet grazed his head and struck and killed the victim. According to LaFave, "A is guilty of murder if he is actually the agent of B's death, notwithstanding the fact that he acted at B's request as where A shoots and kills B upon B's insistence that he wants to die now rather than continue to suffer from a serious illness." Criminal Law, p. 650. Suicide is not murder under the statute, since the defendant would be required to have knowingly or willfully caused the death of another human being, not of himself. By attempting suicide, the defendant did not knowingly or willfully cause the death of the victim.

A man and his friend decided to rob a bank. They entered the bank and quickly got into a shootout with the security guard. While the man was taking the money from the vault, the security guard was struck by a bullet and killed. The two men then left and, as the man was following his friend to the car, the man shot the friend in the back. This jurisdiction defines all murders as second-degree murder unless deliberation and premeditation can be shown, in which case the crime is elevated to first-degree murder. Manslaughter is defined as at common law. This jurisdiction also follows the agency theory for felony murder. Which of the following statements is most accurate? (A) The defendant is guilty of first-degree murder of the security guard and second-degree murder of the friend under the agency theory. (B) The defendant is guilty of first-degree murder of security guard and the friend under the agency theory. (C) The defendant is guilty of second-degree murder of the security guard and first-degree murder of the friend. (D) The defendant is guilty of second-degree murder of both the security guard and the friend, because both were killed during the felony, and neither death was premeditated.

(C) The defendant is guilty of second-degree murder of the security guard and first-degree murder of the friend. In this jurisdiction, murder is divided into two degrees for the purpose of imposing a more severe penalty for some murders than for others. First-degree murder includes intent-to-kill murder accompanied by premeditation and deliberation. Murder not falling within the definition of first-degree murder is considered second-degree murder. Here, the defendant's act toward the security guard is best described as felony murder. Even though he did not kill the guard (his friend did), under the agency theory of felony murder, a felon is responsible for the actions of his co-felons. Because the guard was shot by the friend, the defendant will be responsible. That killing was committed during the course of the armed robbery, and therefore, it will be felony murder, which is second-degree murder in this jurisdiction. Additionally, because the man intentionally shot his friend, that will be first-degree murder in this jurisdiction. Premeditation can occur in a moment, and here, the shooting was from behind, evidencing an intent to kill.

A police officer sees a robber point a pistol at a couple and takes items from them before fleeing on foot. The officer chases after the robber. The robber, with the gun still in hand, runs into a nearby apartment. This apartment is being leased by the robber. The officer follows the robber into the apartment and soon realizes that there is no rear exit. The officer begins to search the apartment in an attempt to locate the robber and finds a large number of drugs when he opens a hallway closet. The robber is eventually located under a bed in another room. The officer arrests the robber for the armed robbery and possession of drugs with the intent to distribute. At a pretrial motion, the robber's attorney argues that the drugs are inadmissible evidence because the officer did not have a warrant to search the property. How will the court rule? (A) The drugs are inadmissible because the officer needed the warrant to enter the apartment. (B) The drugs are inadmissible because the drugs were not in plain view. (C) The drugs are admissible because the officer was in hot pursuit. (D) The drugs are admissible because there were exigent circumstances.

(C) The drugs are admissible, because the officer was in hot pursuit. Police may enter and search a private dwelling while in reasonable pursuit of a fleeing suspect. In such a circumstance, what would otherwise have been a warrantless search becomes lawful because the police are in actual "hot pursuit" of a fleeing suspect and actively attempting to apprehend him. While in that pursuit, they may seize not only evidence of the crime for which they are chasing the suspect but also any additional contraband that they find. Here, the officer was in hot pursuit of the robber and while attempting to apprehend him, the officer chased the robber into an apartment. The drugs were located in a closet while the officer was looking for the robber. Therefore, the drugs were lawfully seized contraband and can be admitted as evidence.

A man purchased two tickets for a basketball game. The man then contacted his best friend, who agreed to accompany the man to the game. As they were driving to the game together, the man sped up to cross an intersection while the traffic signal was changing from yellow to red. As he reached the intersection, the man was traveling at twice the posted speed limit. Simultaneously, a car driven by a woman entered the intersection despite having a red light and collided with the man's car. The friend, who was not wearing a seatbelt at the time, suffered a broken arm and collarbone in the collision. This jurisdiction has adopted a modified comparative negligence statute. This jurisdiction has also adopted a statute requiring all occupants of motor vehicles to wear a safety belt, harness, or strap. The friend subsequently brought suit against the woman of the car that ran the red light to recover damages for his injury. What is the most appropriate result? (A) The friend should recover full compensation because he was not negligent. (B) The friend should recover full compensation because he was not in control of either vehicle that caused the accident. (C) The friend's recovery should be reduced because he was negligent per se. (D) The friend should recover nothing, because he was negligent per se.

(C) The friend's recovery should be reduced, because he was negligent per se. Here, the comparative negligence statute is critical. Had the plaintiff been wearing his seatbelt, there would be 100% recovery, because he did nothing wrong either actively or passively. However, when there is a comparative negligence statute, it will apply when there is a negligent plaintiff and one or more negligent defendants. While the plaintiff (the friend) was just a passenger who was riding in one of the cars and did nothing to cause the accident, he did fail to wear his seatbelt, which contributed to his harm. Therefore, his recovery will be limited.

A six-year-old girl was kidnapped outside her home in a small town. Following the girl's abduction, her parents publicly announced a $50,000 reward to anyone responsible for the apprehension of the kidnapper. The girl's abductor was described as a Caucasian male between the ages of 25 and 30 with curly blond hair. He also was identified as having a pentagram tattoo on his left arm. One afternoon, an off-duty police officer from a nearby town stopped at a local fast-food restaurant for lunch. The officer was aware of the kidnapping but was unaware of the reward. Once he entered the restaurant, the police officer noticed a man fitting the description of the kidnapper. The police officer proceeded to arrest the individual, who, in fact, turned out to be the girl's abductor. After the girl was found unharmed, the police officer learned of and requested the $50,000 reward. However, the girl's parents refused to pay the police officer the reward money. If the police officer brings suit against the girl's parents to recover the $50,000 reward, for whom should the court enter judgment? (A) The police officer, because he accepted the offer by apprehending the kidnapper. (B) The police officer, because he was off duty when the apprehension occurred. (C) The girl's parents, because the officer was unaware of the reward when he apprehended the kidnapper. (D) The girl's parents, because recovery of a reward by a police officer is against public policy.

(C) The girl's parents, because the officer was unaware of the reward when he apprehended the kidnapper. This question deals with the preexisting duty rule. In this question, the issue is whether an offeree can accept a reward that they were unaware was offered before they acted. The answer is no, if the requested action occurs before the offeree is aware. Because the officer was unaware of the reward offered at the time that the apprehended the kidnapper, he may not obtain it later.

A tenant entered into a two-year lease with a landlord for an apartment. Six months into the lease, the tenant fell asleep while smoking and caused a fire that destroyed the apartment. As a result, the tenant moved out. The next month, the landlord demanded that the tenant pay rent, but the tenant refused, citing the destruction of the property. The landlord sued to recover the unpaid rent. Who will prevail? (A) The tenant, because the property has been destroyed. (B) The tenant, because he has moved out of the property. (C) The landlord, because the tenant negligently caused the destruction. (D) The landlord, because the tenant leased only part of the property.

(C) The landlord, because the tenant negligently caused the destruction. At modern law, when leased property is destroyed, the tenant is relieved from the duty to pay rent, and there is no longer any distinction between a tenant who leases part or all of a building. However, there is an exception to this rule--where the tenant intentionally or negligently causes the destruction, the tenant will not be relieved from liability. Because the tenant here negligently caused the fire that destroyed the apartment, he will not be excused from paying rent.

One day, a traffic light in a busy intersection malfunctioned because of an electrical problem which could have been prevented if the state had performed reasonable maintenance. A driver was hit by another car because of the confusion created by the malfunctioning traffic light. While the driver's car was stalled in the middle of the road as a result of the accident, a trucker who had fallen asleep at the wheel crashed into the driver's car. The driver now seeks to bring suit in federal court against the various parties that caused him injury to his person and damages to his car. Assume that all parties are residents of the state except the driver, who lives in a neighboring state. Under which of the following circumstances would the federal court have jurisdiction over the case? (A) The lawsuit involves diverse litigants in which the amount in controversy is $55,000. (B) The lawsuit involves diverse litigants where the amount in controversy is $80,000, which includes property damage and medical expenses against the trucker as well as attorneys' fees and litigation costs in the amount of $10,000. (C) The lawsuit is a negligence action involving diverse litigants, and the driver has incurred property damages of $30,000 and medical expenses of $50,000. (D) The lawsuit involves diverse litigants, including the state, the trucker, and the other driver, and the claim against each defendant is $30,000.

(C) The lawsuit is a negligence action involving diverse litigants, and the driver has incurred property damages of $30,000 and medical expenses of $50,000. Federal district courts have original subject-matter jurisdiction over matters involving diverse litigants in which the amount in controversy is more than $75,000, exclusive of the costs and expenses associated with trial. A claimant may add together each amount in controversy against a particular defendant to meet the threshold. Thus, a federal district court would have jurisdiction over this action, in which property damages are $30,000 and medical expenses are $50,000, because the aggregate is over $75,000 and the litigants are diverse.

A statute regulating the practice of midwives in a state was passed at midnight at the end of the regular legislative session. There were few legislators in attendance at the time of the vote because most of the work of the legislature had been done and most members had gone home for the holidays, although a quorum existed. The sponsor of the legislation finally succeeded in persuading a core group to vote for the bill, which included a provision that the founder of the midwives' lobbying group would no longer be licensed under the state's laws. The group's founder was a practicing midwife who had lobbied successfully against health maintenance organizations and on behalf of a number of state laws guaranteeing patients the right to treatment in the area of pregnancy and childbirth. The bill's sponsor saw the midwives' successes as having driven up the cost of health care treatment throughout the state. If the group challenges the revocation of the founder's license in federal court, what is the most likely outcome? (A) The state will prevail because regulation of the group does not implicate interstate commerce. (B) The state will prevail because a license is a privilege that can be revoked without procedural due process. (C) The lobbying group will prevail because the revocation constitutes a bill of attainder. (D) The lobbying group will prevail because the revocation violates members' right to work under the Privileges and Immunities Clause.

(C) The lobbying group will prevail, because the revocation constitutes a bill of attainder. Sections 9 and 10 in Article I of the U.S. Constitution bar both federal and state legislatures from passing bills of attainder. A bill of attainder is a legislative act that inflicts punishment without a judicial trial upon named individuals or an easily ascertainable group for past conduct. The legislative act in this question named the person to be punished and revoked her license to practice midwifery in the state without giving her notice and a hearing. The revocation was unconstitutional, and the group should prevail.

On February 3, a florist left a note for one of his customers offering to pay $2,000 for his truck if the customer delivered the truck to the flower shop before 1:00 p.m. on February 13. The florist indicated that the customer needed to accept by delivering the truck because the florist had lots of deliveries for Valentine's Day. Two days later, the customer sent a letter to the florist indicating his acceptance of the offer to purchase. However, the letter was delayed by the post office and did not reach the flower shop until February 12. By then, having not heard back from his customer, the florist had purchased another used truck for $1,800. On February 13, the customer drove his truck to the florist, arriving at about 12:55 p.m., at which time the florist rejected the customer's truck. The customer then filed suit for breach of contract. Which of the following most accurately describes the significance the court will attach to the customer's letter accepting the offer to purchase the truck? (A) The customer's letter, upon its receipt by the florist, operated as an effective acceptance of the florist's offer to purchase. (B) The letter from the customer, upon the customer's mailing of the letter, created a binding unilateral contract between the florist and the customer for the purchase of the customer's truck. (C) The mailing of the customer's letter did not create a binding contract between the customer and the florist. (D) The customer's letter bound the florist and the customer to a unilateral contract as of February 12, when the florist received the letter.

(C) The mailing of the customer's letter did not create a binding contract between the customer and the florist. When an offeror requests acceptance by actual performance rather than by a promise to perform, the offer is considered an offer for a unilateral contract. An offeree's promise is insufficient to create acceptance of a unilateral contact; a unilateral contract can be accepted only by the offeree's performance of the requested act. Here, the florist's note to his customer was an offer for a unilateral contract, because it requested the customer's acceptance by performance of the act of delivery of the truck on or before the specified date and time. The customer's letter containing the promise to deliver the truck by the specified deadline did not operate to create an enforceable contract between the parties, as the customer could accept the florist's offer only by rendering the requested act of actually delivering the truck by the specified deadline, which the customer failed to do.

A popular electronics store is having a Black Friday super sale, with deep price reductions and giveaways for the first five people to make purchases. A man decided to enter the store the night before, hide out in one of the large appliance boxes that were lying in the stockroom, and then be the first person in the store the next morning. During the night, the man needed to use the restroom, and entered a door marked "Employees Only." The door led to a stairway that had a broken first step, and the man fell, injuring himself. The next morning, he was found with a broken leg, and arrested. If the man brings a civil suit against the electronics store, what is the likely result? (A) The man will recover, because he was an invitee who was injured on the premises. (B) The man will recover, because he was a licensee who was injured on the premises. (C) The man will not recover, because he was a trespasser on the premises. (D) The man will not recover, because he was a trespasser in an area marked "Employees Only."

(C) The man will not recover, because he was a trespasser on the premises. An invitee is a person who enters onto the defendant's land at the defendant's express or implied invitation, and who enters for a purpose relating to the defendant's interests or activities. Invitees are classified as either business invitees or public invitees. A business invitee is one who enters onto the defendant's land for a purpose related to the defendant's business activities or interests. An invitee may be regarded as a licensee, or even as a trespasser, if the invitee enters areas of the defendant's property to which his invitation does not extend, or if the invitee stays in a permitted area longer than was contemplated by the invitation. A defendant has no duty of care as to a trespasser whose presence is unknown to him. A defendant also has no duty to inspect his land to attempt to discover unknown trespassers. Here, although initially an invitee (assuming the store was open when the man entered), the man will be considered a trespasser for hiding in the stockroom without permission and staying overnight (assuming the store was closed overnight). The man exceeded the scope of his invitation in both time and place. Therefore, because the man's trespassing was unknown to the store, the store had no duty to the man, and he will not recover for his injuries.

On Monday, a man told a gardener, "I am having a party on Sunday and I want my house to look good. If you will promise to mow my lawn by Saturday, I will pay you $50." On Friday, the gardener arrived at the man's home just as the man was leaving for work and began to mow the man's lawn. The man said nothing to the gardener, but drove off as he saw the gardener unloading his lawn mower. When the man arrived home from work that evening, he noticed that only half of his lawn had been mowed. He then found a note from the gardener slipped into his mailbox which read, "Sorry, but I ran out of gas to power the lawn mower and did not have time to buy more gas to finish the job. I'm taking the weekend off, but I will be back Monday morning to finish the work." If the man brings suit against the gardener for breach of contract, who is likely to prevail? (A) The gardener, because he never accepted the offer made by the man. (B) The gardener, because he offered to cure the defective performance by finishing the job on Monday morning. (C) The man, because the gardener's part performance necessarily implied an acceptance and a promise that he would render complete performance. (D) The man, because under the doctrine of equitable estoppel, the gardener's part performance was evidence of his intent to honor the entire contract.

(C) The man, because the gardener's part performance necessarily implied an acceptance and a promise that he would render complete performance. As a general rule, where the offeree (the gardener) begins the performance contemplated, he thereby impliedly promises to complete it. However, in order that the act of part performance may be treated as implying a promise to complete, the following requirements must be present. First, the offer was for an entire contract, and not for a series of separate contracts. Second, that what is begun must be a part of the actual performance bargained for, and not mere preparation for performance. Third, that such implied acceptance is communicated to the offeror, or he had knowledge of it. Since the man saw the gardener beginning to mow his lawn, the gardener will have impliedly promised to mow the lawn by Saturday, and his failure to do so would be a breach.

An animal rights activist group always fought developers when construction projects threatened wildlife in the area. Some members of the group became involved in a battle with a corporation that owned the largest ski resort in town. The corporation obtained permits to expand its operations by adding additional trails on the mountain and increasing the square footage of its ski lodge. Members of the group resisted all expansion efforts in the proper public forums, but when the corporation's permits were approved, some of the more extreme members decided to take desperate action. Fringe members flooded the unoccupied ski lodge by opening the gates of a local dam during the off-season. The dam water flooded the lodge, rendering it a total loss and causing millions of dollars in damages. Thereafter, the state legislature enacted a law restricting the rights of members of the group in several ways, one of which was excluding members of the group from holding elected office in the state. A senator justified the legislation on grounds that "terrorists have no place in our placid state." A member of the group was in the middle of his campaign for mayor. The member did not participate in the flooding of the ski lodge. If the member challenges the constitutionality of the exclusionary legislation in court, what is the likely outcome? (A) The state will prevail, because the group is a subversive organization. (B) The state will prevail, because it acted within its police power. (C) The member will prevail, because the law is a bill of attainder. (D) The member will prevail, because he did not engage in violence as a member of the group.

(C) The member will prevail, because the law is a bill of attainder. A bill of attainder punishes named individuals or easily ascertainable members of a group without the benefit of judicial trial. In the Constitution, Article I, Section 9, Clause 3 states: "No bill of attainder...shall be passed"; and Article I, Section 10, Clause 1 provides: "No state shall pass any Bill of Attainder." These apply to states as well as the federal government. The state legislation in this question is a bill of attainder because it amounts to legislative punishment for membership in the animal rights activist group. Members of the group, such as the member in question, are prohibited from holding elective office without a judicial determination of their right to serve. The legislation is unconstitutional.

A man was driving to work in the northbound lane of a highway at the same time that a mother was driving her son to school in the southbound lane of the same highway. The son began to cry, which diverted the mother's attention from the road, causing her to drift slightly over the centerline of the highway. The man saw the mother's vehicle cross the line and enter his lane. The man did not move over, nor did he decrease his speed. As the mother drew closer, the man could have moved over to avoid the accident, but did not, and the mother's car hit the man's car. The mother was injured and her car suffered damage. The applicable jurisdiction has adopted a rule of contributory negligence. The mother subsequently brought suit against the man for her injuries. What is the most appropriate result at trial? (A) The mother cannot recover damages, because she was negligent. (B) The mother cannot recover damages, because she was negligent per se. (C) The mother can recover damages because the man could have slowed down or moved over. (D) The mother can recover damages because she acted reasonably given the circumstances

(C) The mother can recover damages because the man could have slowed down or moved over. Generally, in a contributory negligence jurisdiction, a plaintiff is precluded from recovery if she is found to have been even 1% negligent. However, the last clear chance doctrine provides a basis for recovery even where a plaintiff is otherwise contributorily negligent. If injury to the plaintiff could still have been avoided through a subsequent exercise of due care by the defendant, than the defendant is said to have had the last clear chance to avoid harm, and the plaintiff's contributory fault will not bar recovery. Here, because the man could have avoided the accident and chose not to, he had the last clear chance to avoid the harm, and so the mother, whose negligence would normally bar her recovery in a contributory negligence jurisdiction, can nevertheless recover.

Acme is a Georgia corporation that has its principal place of business in Alabama. Polly used to live in Alabama and work at Acme's Alabama plant. However, she was recently terminated from her employment with Acme. She consulted an attorney, who advised her that her termination was wrongful under a law recently enacted by Congress. The lawyer also informed Polly that Georgia had the most liberal procedures for prosecuting this type of action. Polly moved to Georgia, where she is now working as a waitress, and commenced an action in the appropriate state court against Acme for wrongful termination under the applicable U.S. law and a similar Georgia statute. Polly seeks $80,000 in damages. Acme filed a notice of removal in the U.S. District Court in Georgia. Polly then filed a motion for remand back to state court. How should the court rule on Polly's motion? (C) The motion should be denied because one of Polly's actions is a federal claim. (A) The motion should be granted because there is a lack of diversity. (B) The motion should be granted because Polly moved to Georgia only for the purpose of acquiring a more litigation advantageous forum. (D) The motion should be denied because there is no diversity of citizenship.

(C) The motion should be denied, because one of Polly's actions is a federal claim. Because Polly's claim is based upon a federal claim, Acme may remove the case to the appropriate U.S. District Court. It is irrelevant that Acme is a citizen of Georgia because defendants who are citizens of the forum state are only prohibited from removing the suit if federal jurisdiction would be based on diversity. Here, federal jurisdiction would be based on federal question, not diversity.

A man was injured while riding a bus. The bus company operates nationwide and was concerned that their reputation would be tarnished if the case was litigated, so they settled with the man out of court. In the settlement, the man was given free bus rides anywhere in the country for life. Several years after the settlement was reached, the National Transportation Safety Board passed a regulation precluding nationwide passenger carriers from providing free transportation. The ruling was retroactive and covered airlines, cruise ships, trains, and bus services. The defendant then notified the man that it could no longer allow him to ride the bus for free as originally agreed upon. The man subsequently brought suit in federal court against the company. The defendant moved that the case should be dismissed because the federal court lacked subject-matter jurisdiction. Which of the following is true? (A) The motion should be denied, because the matter involves interstate commerce, which is an exclusively federal power, and therefore should be heard in federal court. (B) The motion should be denied, because a central issue in the case is the federal statute. (C) The motion should be granted, because there is no federal question presented. (D) The motion should be granted, because defendants can remand a case to state court if they choose.

(C) The motion should be granted, because there is no federal question presented. A suit arises under the US Constitution & laws of the US only when the plaintiff's statement of his own cause of action shows that it is based upon those laws/the Constitution. It is not enough for a plaintiff to allege an anticipated defense to his cause of action and assert that the defense is invalidated by some provision of the US Constitution. These facts are similar to the Mottley decision [Louisville & Nashville Railroad Company v. Mottley, 211 U.S. 149 (1908)]. In that case, the court found that the defendant's allegations showed that in the course of litigation a question under the US Constitution would arise, but this did not satisfy the court's requirement for federal-question jurisdiction that the plaintiff's original cause of action arise under the Constitution.

A tall building was under construction in the downtown business district of a city. A lawyer took a car to the downtown area to go to a meeting with a client. She was running late for the meeting and could not find a legal parking spot, so she decided to park illegally in front of a fire hydrant. This was in violation of a local ordinance that prohibited parking within 50 feet of a fire hydrant. The lawyer figured that it was better to get a ticket than to miss her meeting with an important client. The spot where she parked was next to the site of the construction of the new building. While the lawyer was at her meeting, an accident occurred at the construction site. A large crane was being used to lift a load of bricks. A cable on the crane broke, and the bricks fell. Most of them landed on top of the lawyer's car. The load of bricks that fell on the lawyer's car caused the gas tank of the car to rupture and explode. Shrapnel from the explosion flew in all directions and injured a pedestrian who was walking on the sidewalk near the lawyer's car. If the pedestrian sues the lawyer and relies on the doctrine of negligence per se, which of the following, if true, is the lawyer's best defense? (A) Payment of a small fine is the only penalty provided in the ordinance for those who park too close to fire hydrants. (B) The police never issued a ticket to the lawyer for parking in front of the fire hydrant.(C) The purpose of the parking ordinance was to facilitate access to the hydrant by fire trucks, not to protect against accidents like the one that occurred when bricks fell on the lawyer's car. (D) The pedestrian would not have been injured if the construction company had properly maintained the crane.

(C) The purpose of the parking ordinance was to facilitate access to the hydrant by fire trucks, not to protect against accidents like the one that occurred when bricks fell on the lawyer's car. For a negligence claim, the jury usually must assess all the facts and circumstances to decide whether the defendant acted with as much care as a reasonable person. In most states, however, the doctrine of negligence per se provides a shortcut to proving that the defendant failed to exercise reasonable care. A defendant who violated a statutory standard of care will be treated as negligent per se, meaning that the defendant's conduct will automatically be deemed to be negligent. Breach of a statutory duty will be negligence per se only if two requirements are met: (1) the plaintiff must be a member of the class of persons meant to be protected by the statute; and (2) the plaintiff's injury must be of the type of harm the statute was designed to prevent. Here, if the defendant lawyer can show that the plaintiff pedestrian's injuries were not of the type intended to be prevented by enactment of an ordinance prohibiting parking within 50 feet of a fire hydrant, she will not be negligent per se. Exam Tip: A small minority of jurisdictions follow the rule that violation of a criminal statute is merely evidence of negligence, but not negligence per se. Students should follow the majority rule on the MBE, unless the facts say otherwise.

A state legislature, in an attempt to curb teen pregnancies, recently enacted a statute denying welfare assistance to parents for illegitimate children. A young woman was receiving welfare assistance after being awarded sole custody of her two children following her divorce several years earlier. She recently gave birth to a third child and applied for an increase in her welfare assistance as a result. However, her request was denied pursuant to the state law. The woman sued the state, claiming that the law was unconstitutional and violated the Equal Protection Clause. Which of the following correctly states the burden of persuasion in this case? (A) The woman has the burden of persuading the court that the statute is not necessary to further a compelling interest. (B) The woman has the burden of persuading the court that the statute is not substantially related to achieve an important government interest. (C) The state has the burden of persuading the court that the statute is substantially related to achieving an important government interest. (D) The state has the burden of persuading the court that the statute is rationally related to a legitimate government interest.

(C) The state has the burden of persuading the court that the statute is substantially related to achieving an important government interest. Intermediate scrutiny applies to government action using quasi-suspect classifications, such as gender and illegitimacy. Here, the statute treats legitimate and illegitimate children differently. As such, it will be subjected to the intermediate scrutiny standard. Under this standard, the burden is on the state to prove that the measure being challenged is substantially related to the achievement of an important government interest.

A woman was on trial for the murder of her husband. The prosecution alleged that the woman paid one of her neighbors $25,000 to kill her husband. Immediately after the killing, the neighbor boarded a bus and called his roommate. In a lowered voice, he whispered, "Pack your bags, we gotta get out of town for a while 'till the heat cools off. Don't worry about money, I got it covered." The neighbor's conversation was heard by a fellow passenger. The neighbor was arrested at the airport the following day. At trial, the prosecution attempted to call the neighbor to testify, but he invoked his Fifth Amendment right not to testify. The prosecution then called the passenger to testify. The wife objected. Which of the following is most accurate? (A) The statement is inadmissible because it does not directly incriminate the neighbor. (B) The statement is inadmissible because it does not directly incriminate the wife. (C) The statement is admissible, as a statement against interest. (D) The statement is admissible, as an excited utterance.

(C) The statement is admissible, as a statement against interest. If a declarant makes a statement against his penal interest and is unavailable for trial, the statement will qualify as a statement against interest and will be admitted as an exception to the hearsay rule. The facts indicate that the neighbor has invoked his Fifth Amendment right not to testify. The neighbor's statement also meets this exception's criteria that it was against the neighbor's penal interest when he said it, and that a reasonable person would not have made the statement if it were not true. Therefore, this is the best answer.

A rare coin dealer and a collector met at a stamp-and-coin fair that was held on a monthly basis. The dealer had a mint condition 1905 Silver Eagle in his inventory that was valued at well over $10,000. The collector desperately wanted to purchase the coin, but knew that he did not have enough money to make the purchase. The two spoke for a while, and before leaving, the following note was written from the dealer to the collector: "I will hold the 1905 Silver Eagle you were eyeing up at the fair today for sale to you alone until the next fair." The dealer signed and dated the note and handed it to the collector. If the collector wanted to purchase the coin at the following fair, is the offer still in force? (A) No, because there was no consideration provided to keep the offer open until the next fair. (B) No, because "until the next fair" does not state a sufficient period of time. (C) Yes, because it was a firm offer. (D) Yes, because the offer concerned the sale of goods.

(C) Yes, because it was a firm offer. This question concerns a sale of goods by a merchant. The writing from the dealer to the collector here constitutes a firm offer as defined by the UCC. A written offer made by a merchant that expressly states that it will be held open is effective and irrevocable for the stated period (up to a maximum of three months).

A doctor is on trial for battery of a former patient. He claims that he was acting in self-defense, as the client became enraged at the doctor's advice to no longer pursue his dream of attending law school. During trial, the doctor testified that he was warned by two of the patient's friends that the patient had a short temper and to "beware." The prosecution objected. How should the court rule? (A) The statement is inadmissible, as hearsay not within an applicable exception. (B) The statement is inadmissible, as a self-serving statement. (C) The statement is admissible, as it is not hearsay. (D) The statement is admissible, under the present sense impression exception to the hearsay rule.

(C) The statement is admissible, as it is not hearsay. In many civil and criminal cases, the jury may be required to determine the state of mind of a defendant based on circumstantial evidence. Thus, under the Federal Rules of Evidence, any statements or warnings heard or otherwise understood by a defendant are not hearsay, because they show a defendant's state of mind when he heard the statement. If the issue is whether the defendant (or some other person) actually heard the statement or warning, the statement will be admissible nonhearsay because the truth or falsity of the out-of-court statement is immaterial. Here, the doctor's testimony, that others had warned him that his patient has a short temper and to beware, could technically be regarded as involving out-of-court statements, but they are not being offered to prove the truth of the matter asserted. Instead, the fact that the doctor had been told of the patient's short temper and to beware is relevant as to whether the doctor had a good-faith belief that force was reasonably necessary in defense.

A landlord leased office space to a tenant for five years. The lease contained covenants whereby the tenant agreed to pay $750 rent per month to the landlord, and the landlord agreed to repaint the office once each year. At the beginning of the second year, the landlord refused to paint the office. What are the tenant's remedies? (A) The tenant may refuse to pay rent due to the landlord's breach of covenant. (B) The tenant may treat the lease as terminated if he vacates the premises. (C) The tenant may paint the office space himself and offset his subsequent rental payments by the cost of the paint and labor. (D) The tenant may not paint the office space, because this would constitute an ameliorative waste.

(C) The tenant may paint the office space himself and offset his subsequent rental payments by the cost of the paint and labor. A tenant may seek money damages for a landlord's breach of a term or covenant in the lease. Traditionally, a tenant may vacate the premises and terminate the lease if the tenant has been evicted by the landlord, actually or constructively. Today, statutes in many jurisdictions provide the tenant with the following statutory remedies: (1) withholding rent; or (2) repairing the premises and deducting the cost of repair from subsequent rent payments. Therefore, under the circumstances, this is the best answer choice.

A man sued his company claiming that he was denied his yearly bonus. The CEO claimed that the man was not a bonused employee. To prove this fact, he wished to introduce a record of all bonused employees. The CEO called the secretary who was in charge of payroll to testify to this record. The man objected claiming the report was hearsay. How should the court rule? (A) The testimony is inadmissible under the Confrontation Clause. (B) The testimony is inadmissible because the business record exception applies only to an actual entry in a business record. (C) The testimony is admissible as proof of the absence of an entry in a business record. (D) The testimony is admissible as habit evidence, provided there is corroborating evidence.

(C) The testimony is admissible as proof of the absence of an entry in a business record. In order to prove who was owed a bonus, the secretary must testify to a document that lists all of the bonused employees. Because this list is an out-of-court statement being offered for its truth, an exception must exist for it to be admissible. Here, since the CEO is attempting to prove that the man is not on the list, and therefore not owed a bonus, the absence of an entry in a business record is what is being proved. The elements are similar to the business record exception; however, it is being used to show what is not there, rather than what is. Because the man's absence from the bonus list is what is being sought to be proved, this exception provides for the list's admissibility.

A man was a convicted of sexual assault and required to register with the county of his residence within 30 days of his release from prison. After being paroled, the man rented an apartment, but failed to register. Three months later, during a routine traffic stop, the man was arrested for failing to register in violation of his parole. At trial, the prosecutor called the chief clerk of the county in which the defendant's apartment was located to testify. She proposed to testify that she did a diligent search of the country sex offense registry and that the defendant's name did not appear in the records. The defendant objected, claiming that her testimony was hearsay. How should the court rule? (A) The testimony is inadmissible, under the Confrontation Clause. (B) The testimony is inadmissible because the business record exception applies only to an actual entry in a business record. (C) The testimony is admissible, as proof of the absence of an entry in a business record. (D) The testimony is admissible because the clerk has firsthand knowledge of the country records.

(C) The testimony is admissible, as proof of the absence of an entry in a business record. In order to prove who has not registered as a sex offender, the clerk must testify to a document that lists all of the registered sex offenders. Because this list is an out-of-court statement being offered for its truth, a hearsay exception must exist for it to be admissible. Here, the prosecution is attempting to prove that the man is not on the list, and therefore not compliant with the law. As such, the absence of an entry in a business record is what is being proved. The elements are similar to the business record exception; however, it is being used to show what is not there, rather than what is there. Because the man's absence from the list is what is being sought to be proved, this exception provides for the clerk's testimony.

A woman who was missing one of her legs applied to be a city firefighter, but was rejected. The woman then contacted the station chief, who admitted that the woman was denied admission solely due to the fact that she was missing one leg. The woman then filed an appropriate action against the city asserting that her equal protection rights had been violated. Which of the following correctly states the applicable burden of persuasion? (A) The city has the burden to show that the woman's rejection furthers an important state interest. (B) The city has the burden to show that the woman's rejection is rationally related to a legitimate state interest. (C) The woman bears the burden to show the city's actions are not rationally related to a legitimate state interest. (D) The woman bears the burden to show the city's actions do not further a compelling state interest.

(C) The woman bears the burden to show the city's actions are not rationally related to a legitimate state interest. Discrimination based on a physical disability, such as a missing leg, implicates neither a fundamental right nor a suspect class. As such, rational basis scrutiny would be applied. The burden of persuasion would be on the plaintiff, the woman, to show that the city's actions are not rationally related to a legitimate state interest. Notwithstanding certain protections created by state governments and Congress (notably, the ADA) to prevent discrimination against persons with disabilities, the fact that the woman was rejected based on his physical disability per se does not warrant the use of heightened equal protection scrutiny.

Police received reliable information that a homeowner had a stolen x-brand stereo in his possession. The detectives submitted an affidavit to a neutral magistrate setting forth sufficient underlying circumstances for the issuance of a search warrant. Making a determination of probable cause, the magistrate issued a warrant for the x-brand stereo at the homeowner's address. The police arrived at the homeowner's dwelling, showed him the warrant, and came inside. In the living room, they noticed a y-brand stereo, which had the serial number removed. Upon further inspection, the police determined that the stereo had been stolen from a local electronics store during a recent burglary. The police placed the homeowner under arrest and instructed him to remain seated in the living room while they searched the rest of the home. One of the officers proceeded to the basement where she found the stolen x-brand stereo. She then decided to search the upstairs and came upon a stolen z-brand stereo in the homeowner's second-floor bedroom. The homeowner was subsequently prosecuted for receiving stolen property. At trial, the homeowner moves to prevent introduction of the stereos into evidence. His motion should be granted with respect to what evidence? (A) The y-brand stereo. (B) The z-brand stereo. (C) The y-brand and the z-brand stereos. (D) None of the stereos.

(C) The y-brand and the z-brand stereos. A search made under authority of a search warrant may extend to the entire area covered by the warrant's description. For example, if the warrant authorizes a search of "premises" at a certain described geographical location, buildings standing on that land may be searched. If the place is identified by a street number, the search may extend to those buildings within the curtilage and the yard within the curtilage. LaFave points out that the permissible intensity of the search within the described premises is determined by the description of the things to be seized. Here, the warrant covered the seizure of a stolen x-brand stereo at the homeowner's address. When the police arrived at the defendant's home, they noticed a y-brand stereo in the living room. The first question is whether the police, under the plain-view doctrine, were permitted to seize the y-brand stereo, which also turned out to be stolen. The plain-view doctrine is legitimate only where it is immediately apparent to the police that they have evidence before them; the plain-view doctrine may not be used to extend a general exploratory search from one object to another. In fact, in Arizona v. Hicks [480 U.S. 321 (1987)], it was held that full probable cause was needed to pick up an item of stereo equipment to ascertain its serial number (which revealed it was stolen). Based on Hicks, the y-brand stereo was unlawfully seized since the police needed to "further inspect" it to determine its stolen status. By the same token, the police did not have authority to continue to search the home after discovering the stolen x-brand stereo (specified in the warrant). When the purpose(s) of the warrant have been carried out, the authority to search is at an end. This answer choice is correct because the defendant's motion to exclude the y-brand and the z-brand stereos will be granted.

A man was prosecuted for armed robbery of a local convenience store. The robber had been extremely unkempt, with long hair and a scruffy beard. The store clerk identified him in a photo lineup. At trial, the alleged robber wore a suit and tie, was clean-shaven, and had short hair. The prosecutor then showed the store clerk a group of photographs. The clerk testified that she had previously told the prosecutor that the third photograph was a picture of the robber. The third photograph was a picture of the suspect, with long hair and a beard. Should the clerk's testimony be admitted? (A) No, because it violates the suspect's confrontation rights. (B) No, as hearsay not within any exception. (C) Yes, as a prior identification. (D) Yes, as a recorded recollection.

(C) Yes, as a prior identification. The evidence is admissible as a prior identification. Federal Rule of Evidence 801(d)(1)(C) classifies a prior identification of a person, made after perceiving that person, as nonhearsay if the declarant (eyewitness) is available to testify and be cross-examined. Thus, the store clerk may testify that she earlier identified the defendant, and the prior identification may be used as substantive evidence.

A patient sued his doctor for malpractice. The patient called an expert witness to testify that the drugs prescribed by the doctor were so experimental that it constituted negligence under accepted practices in the medical community. On cross-examination, the expert stated that Pharmacology: A Guide for the Practitioner was a reliable authority in her area of specialty. The expert testified that she, however, did not rely on that treatise in forming her opinion. The doctor's attorney then proposed to read a portion from the treatise, which stated that the drugs prescribed by the doctor are widely used by other physicians in treating patients. The patient's attorney objected. Will the court admit the evidence? (A) No, because the text of the treatise constitutes inadmissible hearsay. (B) No, because the expert did not rely on the treatise in forming her opinion. (D) Yes, to impeach the expert, but not as substantive evidence.

(C) Yes, as substantive evidence. FRE 803(18) provides that statements contained in treatises may be admitted into evidence during direct or cross-examination of an expert witness if: (1) the treatise is established as a reliable authority; and (2) the treatise is called to the attention of the expert witness during cross-examination or is relied upon by the expert in direct testimony. As an exception to the hearsay rule, statements in treatises admitted pursuant to FRE 803(18) may be used as substantive evidence. They may be read to the jury.

A man and his wife were injured by a truck driven by an employee of S Corporation, a company that manufactures widgets. The accident occurred in State C, where the man and his wife were living at the time. S Corporation is incorporated in State D, maintains its principal manufacturing plant in State C, and is controlled from its offices in State F. After the accident, the man and his wife moved to State F, where they planned to retire. After moving, they contacted a lawyer, who filed suit against S Corporation in the appropriate U.S. District Court where the man and his wife now lived, alleging diversity jurisdiction. The suit alleged that each plaintiff had suffered personal injuries in excess of $75,000. S Corporation moved to dismiss the case for lack of subject-matter jurisdiction. Should S Corporation's motion be granted by the district court? (A) No, because diversity subject-matter jurisdiction exists. (B) No, because S Corporation is a citizen of State D only. (C) Yes, because S Corporation's headquarters are in State F. (D) Yes, because the plaintiffs were citizens of State C when they were injured.

(C) Yes, because S Corporation's headquarters are in State F. The requirements for diversity subject-matter jurisdiction are not satisfied based on these facts, because the parties are not diverse. The plaintiffs are citizens of State F, and S Corporation is a citizen of State D (where it is incorporated) and State F (where the officers control the corporation). Because State F is "in common" amongst the parties, there is not complete diversity. Note that it is irrelevant that the corporation's principal manufacturing plant is in State C. Although the diversity statute links corporate citizenship to the corporation's principal place of business, the Supreme Court has interpreted that phrase to mean the corporation's "nerve center"--i.e., the place from where the corporation is controlled.

A state agency needed to select an officer to go undercover and infiltrate a white supremacist group whose members occupied a fortified compound within the state. A Hispanic officer with extensive undercover experience applied for the assignment but was rejected in favor of a much less experienced white officer. The agency admitted that race was the determinative criteria in filling the position. If the Hispanic officer sued, should the court uphold the department's decision? (A) No, because it was a violation of the Equal Protection Clause of the 14th Amendment. (B) No, because race cannot be used as a criterion in government job assignments. (C) Yes, because it was necessary to achieve a compelling interest. (D) Yes, because it was rationally related to a legitimate state interest.

(C) Yes, because it was necessary to achieve a compelling interest. The primary criterion for the government job assignment in this case is race. Under the Equal Protection Clause, that classification is only valid when necessary to achieve a compelling interest. This is one of the rare cases in which this standard is met. A Hispanic undercover agent would be totally ineffective penetrating an organization in which race is the primary criterion for membership. Therefore, the racial classification in this case will be upheld.

Knowing that his niece was looking at a red sedan on his used car lot, an uncle sent a signed email to his niece one Friday afternoon, promising to sell her the red sedan from his lot for $1,000, and stating that she could have the weekend to think about it and let him know. After reading the email, the niece sent her uncle a reply that stated, "The price is a little high. I'll give you $800." The uncle responded via email, "I will not accept $800; it is worth more than that." On Saturday, the niece emailed her uncle and said, "I changed my mind, I will pay $1,000." On Sunday, the uncle responded that he refused to sell the niece his car. If the niece brings a cause of action against her uncle to enforce the agreement, will she prevail? (A) No, because a firm offer is immediately revoked with a rejection. (B) No, because the niece did not provide any consideration to create an option contract. (C) Yes, because she accepted the uncle's terms before the uncle took any action contrary to the offer to the niece. (D) Yes, because the uncle was a merchant

(C) Yes, because she accepted the uncle's terms before the uncle took any action contrary to the offer to the niece. Not all statements or questions about an offer are considered counteroffers. This was a firm offer, because the uncle was a merchant and stated in a signed writing that he would hold the offer open for a specified period of time. As such, the offer was irrevocable during this period. When there is an irrevocable offer (either a firm offer under the UCC or an option contract at common law), the offer remains open even in the face of an outright rejection by the offeree. However, the offer will be revoked if the offeree rejects it and the offeror, in the face of the rejection, changes his position in reliance on the rejection. If the offeror has not changed his position in reliance, the offeree may still accept the offer during the period even after previously rejecting it. Therefore, the niece could still accept the uncle's offer here despite having rejected it by making a counteroffer, because the uncle has not changed his position in reliance on the niece's rejection.

An Illinois man wanted to gain personal jurisdiction over a California businessman who had been quite evasive in avoiding service in Illinois. He called the businessman's office, lied as to who he was, and asked to speak to the businessman. His secretary said he was not there, mentioned that he was set to board a flight, but said that he had a short layover in Chicago and asked if the businessman could call him back around 3:00 P.M. CST. The man then went online and found the businessman's flight. The man immediately dispatched a process server to the airport with a copy of the complaint. As the businessman was disembarking from the plane, the process server handed him a copy of the complaint and said, "Welcome to Chicago. You've just been served." Would an Illinois court have jurisdiction over the businessman? (A) No, because the tactics used to shock the conscience. (C) Yes, because the businessman was served while in the jurisdiction. (B) No, because the man employed fraudulent tactics to serve the businessman. (D) Yes, because the businessman was being evasive, which caused unfair prejudice to the plaintiff.

(C) Yes, because the businessman was served while in the jurisdiction. While a defendant cannot be fraudulently lured into a jurisdiction in order to be served, there is no rule that prevents the plaintiff from being disingenuous.

A businessman lived in Japan. He sent a letter to a California car dealership stating that if the dealership would extend his son the credit, he would guarantee the purchase price of any car. The dealership received the letter on February 9, and on February 10, sold a car to the son for $11,500. On February 11, the businessman died suddenly. Unaware of his death, the agency sent him a letter indicating its acceptance of the offer and notifying him of the sale. The dealership also noted that the son had a good credit rating and that the dealership would have extended him credit even without his father's guarantee. Two months later, the son also died suddenly, leaving a bankrupt estate. The dealership then sought to recover the balance of the car's purchase price from the businessman's estate. Would the dealership prevail in a suit against the businessman's estate for the balance owed on the car? (A) No, because the dealership would have sold the car to the son even without the businessman's guarantee of the purchase price. (B) No, because the businessman died before the dealership mailed the letter notifying him of its acceptance. (C) Yes, because the dealership accepted the businessman's offer before his death. (D) Yes, because the dealership foreseeably, justifiably, and reasonably relied on the businessman's promise.

(C) Yes, because the dealership accepted the businessman's offer before his death. A unilateral contract is created when the offeror requests acceptance of the offer by the performance of an act rather than by a promise to perform the act. The offer is accepted only when the offeree performs the requested act. An offer is revoked by operation of law upon the death of the offeror. Here, given that the businessman's offer requested acceptance by performance rather than by a promise to perform, the dealership accepted the offer when it performed the act that the businessman had requested (i.e., extending his son credit). Therefore, the businessman's offer created a unilateral contract, which can be enforced against his estate. The fact that the dealership notified the businessman of its acceptance of the offer after his death is not the controlling factor; what is controlling is that by performing the requested act, the dealership validly accepted the offer (by its performance of the requested act) before the businessman died.

One afternoon, a man was driving his car in the downtown section of a city. Already late for a business appointment, he was becoming very upset because of the heavy traffic. Suddenly, he decided to drive his car along the sidewalk to avoid the traffic jam. While he was speeding down the sidewalk, people were hastily scurrying to avoid his car. His car struck a pedestrian, who had tripped trying to get out of his way, killing her. Which of the following is the best theory regarding the man's criminal liability? (A) Felony murder, because he was committing an assault with a deadly weapon. (B) Transferred intent, because one is presumed to intend the natural and probable consequences of his acts. (C) Intentional killing, because the man's conduct was a substantial factor in causing the pedestrian's death. (D) Commission of an act highly dangerous to life, without an intent to kill but with disregard for the consequences.

(D) Commission of an act highly dangerous to life, without an intent to kill but with disregard for the consequences. Extremely reckless conduct, which creates what a reasonable person should recognize as an unjustifiably high degree of risk of death or serious bodily injury to another, constitutes depraved-heart murder when that conduct causes the death of another, even though there was no intent to kill or do serious bodily injury.

A landlord owned a home in a neighborhood that was zoned for mixed residential/commercial use, and which had been leased out for several years as a doctor's office. Following the doctor's retirement, the landlord rented the house to a family of four, who moved in immediately while the father began renovating the interior. A couple of months later, both of the family's children began getting sick. A subsequent inspection discovered that the doctor and his staff had allowed hazardous chemicals to seep into the floors and walls, and which were slowly evaporating over time. The landlord, who was not aware of the situation until notified by the father, refused to pay to have the floors and walls scrubbed clean of the chemicals. The family sued the landlord, claiming that he had breached the lease's implied warranty of habitability. Will the family prevail? (A) No, because the landlord did not know about the chemicals at the time it entered into the lease with the family. (B) No, because the doctor's lease was a commercial lease. (C) Yes, because the family notified the landlord of the hazardous chemicals within a reasonable time. (D) Yes, because the lease is not a long-term lease.

(C) Yes, because the family notified the landlord of the hazardous chemicals within a reasonable time. In order to have a valid claim against a landlord for breach of an implied warranty of habitability, the tenant must notify the landlord of defect, and the landlord must fail to repair the defect within a reasonable time. Only residential leases contain an implied warranty of habitability. While the doctor's lease was a commercial one, the family has a residential lease, and is therefore owed the implied warranty. Because the family notified the landlord of the defect and the landlord refused to make the necessary repairs, the family will prevail.

An investor bought a run-down house in a developing neighborhood, which he hoped to fix up and "flip" for a considerable profit. The investor hired a painter to paint the outside of the house. The painter was a friend of the investor's who was trying to start his own painting company. The painter agreed to paint the house at a discounted rate if the investor would allow him to put a sign in the front yard of the house advertising the painter's new business. The investor chose a color and told the painter, "I don't know anything about painting, so just do the job in your usual manner." The investor paid the painter in advance for his services. As the painter was painting the side of the house, he accidentally flung a bucket of paint from the ladder where he was working. The bucket hit the investor's neighbor in the head, causing the neighbor to suffer a concussion. The neighbor has asserted claims against the investor and the painter for his injuries. If the neighbor recovers from the investor under a theory of vicarious liability, does the investor have any recourse against the painter? (A) No, because payment by the investor was an acceptance of the work. (B) No, because the investor selected the painter to do the work. (C) Yes, because the judgment against the investor was based on vicarious liability. (D) Yes, because the investor's conduct was not a factual cause of the harm.

(C) Yes, because the judgment against the investor was based on vicarious liability. Where one defendant is held to be vicariously liable for the tort of another directly liable defendant, the first defendant may recover the entire amount of any damages paid to plaintiff from the second defendant, who was actually responsible for plaintiff's injury.

A tenant leased a house in a suburban neighborhood from a landlord. The lease was silent as to which party bore the responsibility to make repairs. One day, a group of neighborhood kids were playing baseball in the street when an errant ball went through a second-floor window of the tenant's house. The tenant replaced the broken window pane at a cost of $300 and deducted the amount from the next month's rental payment. The landlord then sued to recover the $300 from the tenant. Will the landlord prevail in recovering the $300 against the tenant? (A) No, because deduction of the cost of repairs was the appropriate remedy for breach of the warranty of habitability. (B) No, because the broken window constitutes a partial, constructive eviction. (C) Yes, because the lease did not specify that the landlord had the burden of making repairs. (D) Yes, because the tenant did not notify the landlord of the damage and give the landlord a reasonable time to make repairs.

(C) Yes, because the lease did not specify that the landlord had the burden of making repairs. Under the common law rule, in the absence of contractual terms to the contrary, a tenant has a duty to make minor repairs to keep the leased premises wind and water tight. This responsibility arises from the tenant's duty not to commit waste. In this case, given that the lease did not state which party would be responsible for making repairs, the default common law rule applies, and the tenant is responsible for repairing the window. Because liability for the broken window lies with the tenant, the tenant may not deduct the cost of making the repair from the rent payment.

A man was driving to work when his truck collided with a car driven by a woman, who was backing out of her driveway very quickly without looking or yielding to oncoming traffic. The man subsequently filed suit to recover damages from the woman, asserting that he had severely injured his left arm in the accident and that this injury had rendered him unable to work, as he was left-handed. The woman, in turn, claimed that the man's injury was completely unrelated to the car accident and was the result of the repetitive-motion syndrome. The man's employer required its employees to have an annual examination. The previous year, three months before the accident, the man had visited the doctor for his yearly exam. During the trial, the man testified that his injury had caused him severe pain and left him physically unable to work. He then called the doctor to the stand to testify that the man had told him during the annual exam that his previous arm problems had gone away. Should the doctor's testimony be admitted over the woman's objection? (A) No, because it is hearsay not within any exception. (B) No, because the man did not make the statement for the purpose of medical treatment. (C) Yes, because of the man-made statement for the purpose of medical treatment. (D) Yes, as a statement by a party opponent.

(C) Yes, because the man-made the statement for the purpose of medical treatment. Federal Rule of Evidence 803(4) allows the admission into evidence of a statement made to a healthcare professional for the purpose of a present medical diagnosis or treatment, describing past symptoms or history, as an exception to the rule excluding hearsay evidence. Here, the man visited the doctor for his general physical examination, to ensure that he was healthy and able to work. As such, his statement to the doctor would probably be regarded as one made for the purpose of treatment. Therefore, the statement should be admitted pursuant to this exception to the hearsay rule.

After a large chemical spill at an industrial worksite, an inspector for the Environmental Protection Agency investigated the incident. In his initial visit, the investigator took pictures of the scene and made some notes after conducting interviews. After a three-week vacation, the investigator compiled his findings and submitted them in a report. Two weeks after submitting his report, he died of a heart attack. His conclusion was that the worksite owner and operator were negligent in allowing the spill to happen and suggested that fines and penalties be assessed. In a subsequent civil case brought by a worker at the site against the owner and operator, the worker seeks to use the investigator's report. The owner objects. Is the investigator's report admissible? (A) No, because the report was not created contemporaneously with the chemical spill. (B) No, because the report was created in anticipation of litigation, and the inspector is now unavailable. (C) Yes, because the report was created by the inspector, who had a duty imposed by law. (D) Yes, because the inspector is now unavailable.

(C) Yes, because the report was created by the inspector, who had a duty imposed by law. Rule 803(8) creates a hearsay exception for most public records and reports. Properly certified official records from public offices are generally admissible if they are routine, factual, based on personal knowledge of public officials, and appear reasonably reliable. Investigative reports, reports with recommendations, and one-time reports prepared for a narrow purpose are generally admissible except against a defendant in a criminal case. No witness is required if the document is certified; it will be considered self-authenticated under Rule 902. Here, the report by the EPA inspector qualifies as a public record because it was created by a duty imposed by law, and therefore, it will be admissible.

A group of women having a bachelorette party were inside of a club when a shooting occurred behind their table. The bartender was arrested for aggravated assault for the shooting. At trial, the bride-to-be took the stand to testify. She admitted that she did not actually see the shooting but that she heard someone behind her yell, "Watch out! He's got a gun! The bartender's got a gun!" She testified that she heard this being yelled at the same time that she heard the gunshots. There was only one bartender, a male, employed by the bar that night. The person yelling never came forward to speak to the police about the shooting. Can the bride-to-be testify about what she heard? (A) No, because the declarant's statement it is hearsay not within any exception. (B) No, because the declarant is unavailable. (C) Yes, because the statement describes an event while the declarant was perceiving that event. (D) Yes, because there is sufficient corroboration presented, therefore this is a question of weight, not admissibility.

(C) Yes, because the statement describes an event while the declarant was perceiving that event. This answer is the most accurate because it presents a portion of the definition of the present sense impression exception to the hearsay rule. With this exception, a statement which describes or explains an event or condition made while the declarant was perceiving the event or condition, or immediately thereafter, are not excluded by the hearsay rule whether the declarant is available to testify or not. Here, the unidentified declarant yelled out statements regarding the shooter in a club. The statements essentially identified the shooter. The statements were made contemporaneous to the shooting and while the declarant perceived the shooting. Therefore, the Present Sense Impression exception would apply and allow the bride-to-be to testify about the statements in order to prove the truth of the matter asserted.

stockbroker became despondent because the economy was terrible and all of his clients had lost substantial amounts of money on their investments. The stockbroker decided to end his life by jumping off a bridge. He walked to the middle of the bridge, climbed over the railing, stood on the tiny ledge on the edge of the bridge, and prepared to jump. A taxi driver who happened to be crossing the bridge at that moment saw what the stockbroker was doing. The taxi driver slammed on his brakes, jumped out of his taxi, started running toward the stockbroker, and yelled, "Don't do it!" Pleased to hear that someone cared about him, the stockbroker changed his mind about committing suicide and decided that he wanted to live. At that moment, however, the stockbroker started to lose his balance, and his feet began to slip off the small, grimy ledge of the bridge. Just as the stockbroker started to fall, the taxi driver reached him and grabbed the back of his jacket. The taxi driver hung on tightly to the jacket and nearly was pulled over the side of the bridge, but he managed to hang on. Using every bit of strength he could muster, the taxi driver pulled the stockbroker back up and onto the bridge. The stockbroker was relieved to have been saved from falling. Unfortunately, the taxi driver suffered a severe injury to his back from the strain of hanging on to the stockbroker and pulling him back up onto the bridge. A statute in the jurisdiction makes attempted suicide a crime. If the taxi driver asserts a claim against the stockbroker to recover damages for his injuries, will he prevail? (A) No, because the stockbroker did not intend to harm anyone other than himself. (B) No, because attempted suicide was a criminal offense in the jurisdiction. (C) Yes, because the stockbroker put himself in a position of peril. (D) Yes, because he succeeded in saving the stockbroker's life.

(C) Yes, because the stockbroker put himself in a position of peril. Under the rescue doctrine, the fact that someone will try to come to the rescue when others are in danger is regarded as a foreseeable occurrence. As Judge Cardozo put it, "danger invites rescue" [Wagner v. International Railway, 232 N.Y. 176 (1921)]. A rescuer who is injured during the rescue attempt, therefore, can recover against the person whose negligence created the need for someone to come to the rescue. This doctrine applies even when the defendant endangers no one's safety but his own. In this question, the stockbroker put himself in a position of peril and can be held liable for the rescuer's injuries.

A baseball card dealer and a collector met at an auction. The baseball card dealer had in his inventory a mint condition rookie card of a famous baseball player that was valued at well over $50,000. The collector desperately wanted to buy the card, but knew that he could not make the purchase until he sold a few other cards that he currently owned. The two spoke for a while, and before leaving, the dealer wrote the collector the following note: "I, will hold the rookie card that we were talking about for sale to you alone for the next six months, in return for the $500 consideration I received from you today." The dealer signed and dated the note and handed it to the collector. If the collector saw the dealer at another auction five months later and wanted to purchase the card, is the offer still in force? (A) No, because the collector is not a merchant. (B) No, because the UCC only makes a firm offer irrevocable for a maximum of three months. (C) Yes, because there was consideration to support a common law option contract. (D) Yes, because UCC firm offers are irrevocable for six months.

(C) Yes, because there was consideration to support a common law option contract. This question concerns a sale of goods by a merchant. If this were a UCC firm offer, it would not need consideration to be effectively irrevocable; however, it would only be irrevocable for a maximum of three months. Ordinarily, the written offer made by the dealer was effective, but would have become revocable after three months. However, by adding the consideration, as the parties did here, they created a valid common law option contract, which can be irrevocable for longer than three months.

On August 1, a student entered into an oral agreement with a landlord to lease an apartment for $500 per month, starting immediately and running through the end of the year. At the end of the year, the student failed to move out, instead paying the rent for the following month on time and directly to the landlord, who accepted the rent by depositing it into his bank account. What is the relationship between the parties as of January 1? (A) The student is a tenant at sufferance, and the landlord may immediately evict him. (B) The student's payment, and the landlord's acceptance, of the rent formed an at-will tenancy, in which either party can immediately terminate the lease. (C) Because the original lease agreement was not in writing, the Statute of Frauds will operate to invalidate the agreement, resulting in an at-will tenancy. (D) A month-to-month periodic tenancy has been formed.

(D) A month-to-month periodic tenancy has been formed. A lease for a term of years has a definite beginning and end date (e.g., the lease expires on a set date or remains in effect only for a set number of months or years). No notice is required to terminate a lease for a term of years at the end of the specified duration. A periodic tenancy may be created expressly or by implication, as in the case with a holdover tenant. Here, the five-month oral lease agreement, running from August 1 through December 31, was a lease for term of years, which ended on December 31. The tenant then became a holdover tenant on January 1. Because the student paid the monthly rent for January and the landlord accepted the payment, a month-to-month periodic tenancy has now been formed.

A 68-year-old man drove his car into a restaurant while attempting to leave the parking lot. A police investigation showed that the man confused the accelerator for the brake pedal, causing his car to jump the curb and break through a large window. No one was hurt in the accident. However, the man's license was revoked pursuant to a state law requiring all drivers over the age of 65 who are involved in a serious accident to have their licenses revoked until they take another driving test. The man is a retired doctor who now volunteers by driving to the homes of patients who are otherwise unable to receive medical treatment. The man was now unable to attend to these patients due to the revocation of his license. He brought suit against the state challenging the automatic revocation of his license without his first having an opportunity to present evidence on the accident at the restaurant. How should the court rule? (A) For the man, because a pre-termination hearing is required. (B) For the man, because his Fourteenth Amendment rights have been violated. (C) Against the man, because he is now retired. (D) Against the man, because he received adequate due process.

(D) Against the man, because he received adequate due process. The issue here is one of procedural due process. A pre-termination hearing is not required in this situation. As such, choice (A) is incorrect. Choice (C) is incorrect, because it is irrelevant to the man's procedural due process claim. His license to practice is not being taken, but rather, his license to drive. Choice (B) is incorrect regardless of whether it is interpreted as a Fourteenth amendment procedural or substantive due process challenge, because the man will not be entitled to an opportunity to present his case before the suspension of his license.

A landlord and a tenant entered into a residential lease agreement that described the lease term as "continuing until the landlord, in his sole discretion, terminates the lease." Under the majority rule, what kind of tenancy has been created by the lease agreement? (A) A periodic tenancy. (B) A life estate held by the tenant. (C) An at-will tenancy that may be terminated by the landlord only. (D) An at-will tenancy that may be terminated by either the landlord or the tenant.

(D) An at-will tenancy that may be terminated by either the landlord or the tenant. A tenancy that has no fixed duration and lasts only as long as the landlord and tenant desire is an at-will tenancy. In this case, the lease agreement had no fixed duration and provided that only the landlord would have the power to terminate the lease. However, most courts will interpret this language as creating an at-will tenancy that can be terminated by either party. Therefore, this answer choice is correct

An owner planned to renovate her office building and requested several contractors to submit bids to perform the work. One of the contractors in turn sought bids from several subcontractors to determine the overall cost of the job. An electrician submitted a bid to the contractor that he used in computing the bid he sent to the owner. The contractor notified the electrician to the effect. On November 1, the contractor submitted his bid to the owner in which he offered to perform the renovation work for $75,000. On November 4, the electrician notified the contractor that he was unable to perform the electrical work on the owner's project. On November 5, the owner sent the contractor a signed confirmation letter wherein she stated, "I hereby accept your offer to perform the renovation work for the quoted cost of $75,000." Thereafter, the contractor hired another electrician to complete the electrical work at a cost of $5,000 above the electrician's bid. The contractor explained the situation to the owner and informed her that the overall cost of the job would have to be increased to $80,000. The owner responded that she would hold contractor to his original bid of $75,000 and would not be responsible for any additional costs. The contractor then performed the renovation work, but the owner has not yet paid him anything. Which of the following best supports a claim for $5,000 by the contractor against the electrician? (A) The electrician made an offer that the contractor accepted by using the electrician's bid in computing the bid he submitted to the owner. (B) The electrician's bid was a firm offer that was irrevocable, because both the electrician and the contractor were merchants. (C) The contractor made an offer to the electrician that the electrician accepted when he submitted his bid. (D) An option contract was created because the contractor used the electrician's bid in computing the bid he submitted to the owner and notified the electrician of that fact.

(D) An option contract was created because the contractor used the electrician's bid in computing the bid he submitted to the owner and notified the electrician of that fact. When a general contractor about to submit a bid on a construction project secures a bid from a subcontractor for a definite part of the proposed work, and uses the bid to determine that part of his cost, he often finds after the principal contract is awarded to him that the subcontractor refuses to go through with the job. He must then find another to do the job, usually at a price much higher than the promised figure. Can he recoup his loss from the defaulting subcontractor? Yes, the subcontractor is bound under the doctrine of promissory estoppel. Thus, a promisor who induces a substantial change of position by the promisee in reliance on the promise is estopped from denying its enforceability as lacking consideration. The reason for the doctrine is to avoid an unjust result. This answer is therefore correct because Restatement of Contracts 2d, Section 87(2) provides, "An offer which the offeror should reasonably expect to induce action or forbearance of a substantial character on the part of the offeree before acceptance and which does induce such action or forbearance is binding as an option contract to the extent necessary to avoid injustice."

A woman leased an apartment from a landlord. When the woman arrived to take move in, she found that the former tenant was still in the apartment, and refused to leave. As a result, the woman refused to pay any rent to the landlord. The landlord then sued the woman to enforce the lease. How should the court rule? (A) For the landlord, because the American rule requires him to provide only the right to possess the apartment. (B) For the landlord, because the English rule requires him to provide only the right to possess the apartment. (C) For the woman, because the American rule requires the landlord to provide her with both the right to possess and actual possession of the apartment. (D) For the woman, because the English rule requires the landlord to provide her with both the right to possess and actual possession of the apartment.

(D) For the woman, because the English rule requires the landlord to provide her with both the right to possess and actual possession of the apartment. The majority rule is the English rule, which requires that a landlord provide not just the right to possession, but also actual possession of the leased property. Therefore, it is the landlord's responsibility to evict the former tenant and deliver actual possession of the ranch to the woman.

A reliable police informant telephoned the police to report seeing two men in a blue car selling narcotics outside a local movie theater. The informant gave a detailed description of the two men and the license number of the vehicle. A patrol car was immediately dispatched to the movie theater. A few minutes later, the police saw a car matching the description given parked about two blocks from the movie theater. When the police approached, they saw one man fitting the description provided. The police proceeded to the car and opened the doors of the vehicle. The police then pried open a locked tool case that was situated in the rear of the vehicle. Inside, the police found an envelope that contained about one gram of cocaine. They then placed the driver of the vehicle under arrest. Immediately thereafter, the police opened the glove compartment and found a small amount of marijuana. The driver of the van was subsequently prosecuted and charged with two counts of possession of controlled dangerous substances. He filed a motion to exclude the cocaine and marijuana from evidence. How will the court most likely rule on his motion? (A) Granted, as to the cocaine only. (B) Granted, as to the marijuana only. (C) Granted, as to both the cocaine and the marijuana. (D) Denied, as to the cocaine and the marijuana.

(D) Denied, as to the cocaine and the marijuana. In California v. Acevedo [500 U.S. 565 (1991)], the Supreme Court held that police may search an automobile and the containers within where they have probable cause to believe contraband or evidence is contained. Accordingly, in United States v. Ross [456 U.S. 798 (1982)], the Court held that if "probable cause justifies the search of a lawfully stopped vehicle, it justifies the search of every part of the vehicle and its contents that may conceal the object of the search." Since the police had probable cause to search the entire vehicle for drugs, the police may lawfully search both the tool case and the glove compartment.

A police officer pulled over a car after observing it drive through a stop sign. The officer walked up to the driver's side of the car, ordered the driver out of the vehicle, and told him to place his hands on the trunk of the car. As the driver was getting out of the car, the officer noticed a gun in his waistband. The driver stated that he did not have a permit for the gun. The officer then ordered the passenger in the front seat to also get out of the car and place his hands on the trunk. As the passenger exited the car, he dropped a small bag of heroin on the ground. The district attorney charged the driver with illegal possession of a firearm and the passenger with illegal possession of a controlled substance. The driver filed a motion to suppress the gun and the passenger filed a motion to suppress the bag of heroin. How should the court rule on the motions? (A) Grant the driver's motion, but deny the passenger's motion. (B) Grant the passenger's motion, but deny the driver's motion. (C) Grant both motions. (D) Deny both motions.

(D) Deny both motions. The court should deny both motions to suppress. Under Pennsylvania v. Mims [434 U.S. 106 (1977)], an officer at a routine traffic stop has the right to order the driver to exit the car. Additionally, under Maryland v. Wilson [519 U.S. 408 (1997)], an officer also has the right to order a passenger to exit the vehicle when making a routine traffic stop. As such, answer choices (A), (B), and (C) are incorrect.

n a 2010 lawsuit, a state university was held to have unlawfully refused applicants into the engineering masters program on account of their race in the past. The university agreed to hold 20 places in each class for black students, pursuant to a consent decree entered that same year. In 2012, a white male student applied to the engineering masters program at a state university and was rejected. The student's engineering aptitude test score and grade point average was above 15 of the 20 black students admitted. If the student brings suit to require that he be admitted because the university has unlawfully discriminated against him because of his race, what would the court do? (A) Grant relief, because a state may not use race as a criterion in making admissions decisions. (B) Grant relief, because a classification solely by race, even to achieve a worthy purpose, is not necessary to satisfy a compelling state need, and therefore violates the Equal Protection Clause. (C) Deny relief, because a state may consider race as a factor in admissions when it is attempting to aid disadvantaged minorities and penalizes no particular group. (D) Deny relief, because the racial classification is designed to remedy past unlawful discrimination.

(D) Deny relief, because the racial classification is designed to remedy past unlawful discrimination. Racial quotas may be used to remedy past unlawful racial discrimination. This case is different from University of California Regents v. Bakke [438 U.S. 265 (1978)], because the University of California had not been found guilty of past discrimination and was not under a court order to integrate, whereas the state university could be required to use the quota system because of its past discrimination.

Police received information from an undercover police officer that she had just seen two men (whom she described) in a red four-door sedan selling marijuana near the city's largest high school. A few minutes later, two police officers saw a sedan fitting the description half a block from the high school. The driver of the sedan matched the description of one of the men described by the undercover officer. The two officers stopped the sedan and searched the driver. In the pocket of the driver's jacket, the police found a small bottle of pills that they recognized as narcotics. They also looked in the trunk, where they found a quantity of cocaine in a sealed envelope. The driver was arrested and charged with unlawful possession of narcotics. The driver moved to suppress the use of the cocaine found in the trunk as evidence. How should the court rule? (A) Grant the motion, because the search was without probable cause or a warrant. (B) Grant the motion, because the search went beyond the wingspan of the driver. (C) Deny the motion, because this was a search incident to a lawful arrest. (D) Deny the motion, because the police had probable cause to search the entire vehicle.

(D) Deny the motion, because the police had probable cause to search the entire vehicle. A search without a warrant is valid if the search is of an automobile that is capable of being moved by its occupants and the police have probable cause to believe that it is carrying contraband, or where other exigent circumstances require an immediate search. Here, the police had probable cause to believe that drugs were being dealt from the sedan, because a few minutes earlier, an undercover police officer had identified two men in a similar car as selling marijuana near the city's largest high school. The police stopped the defendant's car, which fit the description, half a block from the high school. The driver of the car matched the description of one of the men described by the undercover officer. Because the requirements of the automobile warrant exception had been met, the police could search the entire car. Therefore, the cocaine found in the trunk will be admissible.

A legislature, determined to do something about impatient drivers who put others at risk, enacts a law that driving in the breakdown lane without a legitimate reason to do so is a misdemeanor. The same act, if it proximately causes serious injury, is a felony. Several months later, the defendant was stuck in traffic and tried to go around the traffic bottleneck by driving in the breakdown lane. After driving in the breakdown lane for about a quarter of a mile, the defendant's vehicle crested a hill. Just past the crest, he saw a stalled car in the breakdown lane. The defendant could not stop in time, so he tried to swerve back into the right-hand lane of the highway. His car struck a minivan in the right-hand lane, which flipped over, killing the driver. What is the most serious crime of which the defendant can be convicted? (A) Involuntary manslaughter. (B) Depraved-heart murder. (C) Intent-to-cause-serious-bodily-injury murder. (D) Felony murder.

(D) Felony murder. Felony murder is an intentional or accidental killing proximately caused during the commission or attempted commission of a serious or inherently dangerous felony. (The standard "inherently dangerous" crimes are known by the mnemonic BARRK--burglary, arson, robbery, rape, and kidnapping.) The majority of states classify an unintentional killing done during a felony that is committed in a dangerous manner a felony murder as well. Here, the facts tell us that the defendant's unlawful act (driving in the breakdown lane, death resulting) is a felony in that state. Although it is not one of the five "inherently dangerous" felonies that generally subject a defendant to felony murder charges, the legislature was aiming at the same kind of hazard to life when it passed the legislation. As such, it is likely that the defendant can be convicted of felony murder.

A gang member got into a fight with another young man. Their friends broke up the fight. The gang member, however, remained angry. Later that day, he decided he would kill the young man. The gang member placed a loaded gun in his coat pocket and went outside looking for the young man. He had no idea where to find the young man, so he decided to patrol the neighborhood. After an hour or so, he passed a local bar and decided to go inside and have a drink. While at the bar, he consumed twelve bottles of beer and became very intoxicated. He left the bar and began his search again. Ten minutes later, the gang member ran into the young man on the sidewalk. The two men got into a verbal argument during which the young man threatened to cause bodily injury to the gang member's parents and siblings. The gang member became angry and pulled out his gun and shot the young man to death. When the gang member woke up the next morning, the effect of the alcohol had mostly worn off. He had only a hazy recollection of what happened the night before, and he could not remember killing the young man. What is the most serious crime of which the gang member could be properly convicted? (A) Voluntary manslaughter, because he acted under adequate provocation. (B) Depraved-heart murder, because he acted with wanton indifference to human life. (C) Second-degree murder, because his intoxication prevented him from acting with premeditation or deliberation. (D) First-degree murder, because he acted with premeditation and deliberation.

(D) First-degree murder, because he acted with premeditation and deliberation. A defendant is guilty of first-degree murder if the killing is premeditated and deliberate. In this case, the facts indicate that the gang member decided to kill the victim. He took a loaded gun and went looking for the young man. Accordingly, he acted with premeditation and deliberation, and he is guilty of first-degree murder.

A landlord entered into a written residential lease agreement with a tenant for a mountain cabin. According to the terms of the lease, the landlord agreed to make any necessary repairs to the cabin within a reasonable time except for ordinary wear and tear, which the tenant was responsible for. Several years later, part of the cabin's roof collapsed during one of the mountain's ordinary windstorms. The tenant called the landlord and informed him of the damage, but the landlord refused to make the repairs. A couple weeks later, a loosened piece of wood fell from the damaged area and landed on the tenant's shoulder, causing him injury. The tenant sued the landlord. For what claim(s), if any, may the tenant be awarded damages? (A) For breach of the lease, but not for the tenant's injuries. (B) For the tenant's injuries, but not for breach of the lease. (C) For breach of the lease as well as for the tenant's injuries. (D) For neither breach of the lease nor the tenant's injuries.

(D) For neither breach of the lease nor the tenant's injuries. The tenant is entitled to damages for neither breach of the lease nor the tenant's injuries. The landlord here agreed only to make repairs for damages beyond ordinary wear and tear. There is nothing in the fact pattern to indicate that this windstorm was anything out of the ordinary. As such, this is most likely ordinary wear and tear, for which the tenant, and not the landlord, was responsible. Because the tenant was responsible for making the repairs, the landlord will not be liable for the tenant's injuries that resulted from the damage not being repaired, either.

A retired farmer leased his ranch to a man. When the man arrived to take possession, he found that a ranch hand was occupying the land illegally. The man refused to pay rent to the retired farmer. The retired farmer then sued the man to enforce the lease. How should the court rule? (A) For the retired farmer, because the American rule requires the farmer to provide only the right to possess the ranch. (B) For the retired farmer, because the English rule requires the farmer to provide only the right to possess the ranch. (C) For the man, because the American rule requires the farmer to provide him with both the right to possess and actual possession of the ranch. (D) For the man, because the English rule requires the farmer to provide him with both the right to possess and actual possession of the ranch.

(D) For the man, because the English rule requires the farmer to provide him with both the right to possess and actual possession of the ranch. The majority rule is the English rule, which requires the landlord to provide not just the right to possession, but also actual possession of the property. Therefore, it is the retired farmer's responsibility to evict the ranch hand and deliver actual possession of the ranch to the man.

The police suspected that a teenager, a 16-year-old high school student, had committed a series of burglaries. Two officers went to the teenager's high school and asked the principal to call him out of class and to then search his backpack. While the officers waited, the principal took the teenager into the hall, where she asked to look in his backpack. When the teenager refused, the principal grabbed it from him, injuring the teenager's shoulder in the process. In the backpack, she found jewelry, which she turned over to the officers. The teenager was subsequently charged with committing the burglaries, and now moves to suppress the use of the jewelry as evidence at trial. How should the court rule? (A) Deny the motion, because the search was incident to a lawful arrest. (B) Deny the motion, because school searches are reasonable if conducted by school personnel, on school grounds, on the basis of reasonable suspicion. (C) Grant the motion, because the search was conducted with excessive force. (D) Grant the motion, because the search was conducted without probable cause or a warrant.

(D) Grant the motion, because the search was conducted without probable cause or a warrant. The 4th Amendment applies only to government, not private, conduct. However, when a private party acts at the direction of a government agent, any search conducted and evidence seized will be subject to Fourth Amendment scrutiny. The police asked the principal to call the teenager out of class and to then search his backpack. There was no warrant or probable cause for this action. Therefore, the motion to suppress the jewelry as evidence should be granted.

A defendant, who is divorced, lives with her 11-year-old daughter. One night, the defendant went out to dinner and left her daughter at home. Upon leaving the house, the defendant locked the door to her bedroom. While the defendant was gone, the daughter found a spare key to her mother's bedroom and opened the door. While rummaging through her mother's dresser, she found a bag containing white powder. Concerned that the bag might contain drugs, the daughter called her father, who still owned the home along with the defendant, to ask his advice. The father instructed her to call the police. The daughter then telephoned the police station and reported what she had found. A patrol car was immediately dispatched to her home. The daughter allowed the police to come inside the home. The officers then asked her where the bag that she described to the 911 operator was located, and if she could take them to it. The daughter led the officers to the mother's bedroom and gave them the bag, which they took to the station house for a lab test. After the results confirmed that the bag contained cocaine, the defendant was arrested and charged with illegal possession of a controlled dangerous substance. The defendant's attorney filed a motion to prevent the cocaine from being admitted into evidence. How should the court rule on the motion? (A) Denied, because the daughter, and her father, as co-owner, had the authority to allow the police inside the home and to make the seizure. (B) Denied, because the daughter lived with her mother, and she had equal control over the premises. (C) Granted, because the daughter, as a minor, did not have the apparent authority to permit the police to enter the home. (D) Granted, because the daughter did not have the apparent authority to permit the police to enter the defendant's bedroom.

(D) Granted, because the daughter did not have the apparent authority to permit the police to enter the defendant's bedroom. The consent search exception to the 4th Amendment warrant requirement is being tested here. Any person with an apparent equal right to use or occupy the property may consent to a search, and any evidence found may be used against the occupants [United States v. Matlock, 415 U.S. 164 (1973)]. For third-party consent, joint access and control of the premises is required. In our problem, first and foremost, did the daughter have authority "in her own right" to allow the police to search her mother's bedroom? The answer is no. Although the child may have had the apparent authority to permit the police to search common areas of the home (e.g., living room or kitchen), the daughter clearly did not have authority to allow the police to search her mother's bedroom. Moreover, because the defendant locked the bedroom door, she did not assume the risk that her daughter would gain entry.

A college student was a committed vegan, eating only plant-derived foods, and the thought of all the animals slaughtered to provide meat for people nauseated her. One night, in order to draw attention to the slaughter of innocent animals, the student went to the butcher's section of the supermarket where she worked as produce manager and sprinkled a nausea-inducing chemical on all the meat, fowl, and fish products. Despite being careful, the student accidentally put too much powder in one package of hamburger. When the woman who bought the meat served it to her family, her husband, who was unusually susceptible, died as a result of ingesting the chemical. In the student's jurisdiction, murder is defined as "the unlawful killing of a human being with malice aforethought. Such malice may be express or implied. It is express when there is manifested a deliberate intention unlawfully to take away the life of another person. It is implied when no considerable provocation appears or when the circumstances attending the killing show an abandoned and malignant heart. All murder which is perpetrated by willful, deliberate, or premeditated killing or which is committed in the perpetration of or attempt to perpetrate arson, rape, robbery, or burglary is murder of the first degree. All other kinds of murders are of the second degree." If prosecuted for the criminal homicide of the woman's husband, of what crime should the student should be found guilty? (A) First-degree murder, because she willfully, deliberately, and with premeditation killed the victim. (B) First-degree murder, because the homicide occurred during perpetration of a felony. (C) Second-degree murder, because the circumstances show that she acted with an abandoned and malignant heart. (D) Involuntary manslaughter, because she did not intend to kill anyone

(D) Involuntary manslaughter, because she did not intend to kill anyone. Where there is no intent to kill, in order to constitute murder, a homicide must be either felony murder (which may be 1st-degree), 2nd-degree murder (intent to seriously injure/wanton and willful misconduct), or involuntary manslaughter (criminal negligence or misdemeanor manslaughter). In this case, the facts clearly indicate that the student had no intent to kill or seriously injure; she sought to make a political point by causing people who ate the affected foods to feel nauseated. Thus, the student could be guilty of felony murder, depraved-heart murder, or involuntary manslaughter. The statute given by the problem lists four felonies which, if a homicide occurs during the commission of them, could give rise to 1st-degree felony murder. Because the student committed none of these (arson, burglary, rape, or robbery), that basis for 1st-degree murder is eliminated. In choosing between depraved-heart murder & involuntary manslaughter, one must examine the degree of risk created by the conduct. Acting with a depraved heart is usually and more precisely defined as "wanton and willful misconduct"--conduct which the defendant knows will create a very high risk of death or serious bodily injury. Criminal negligence is usually regarded as equivalent to gross negligence--i.e., creating a high risk of death or injury--and if such conduct results in a homicide, the appropriate label is involuntary manslaughter. Under the circumstances of this question, the student's conduct appears to have created a high rather than a very high risk of injury, and is closer to gross negligence than to wanton and willful misconduct. For that reason, she is guilty of involuntary manslaughter.

One evening, the defendant was playing football with his son in the front yard of his house. The night before, a rainstorm had felled a large branch from a tree in his neighbor's yard, and the branch now lay across the fence that divided the defendant's and his neighbor's properties. When the defendant's wife came outside to tell the defendant and his son that dinner would be ready soon, the defendant launched into a tirade about the fallen branch. His wife responded that she and the neighbor had discussed the branch over coffee that morning and that she had told the neighbor that the defendant would take care of it. Seething with anger, the defendant made no response and continued playing catch with his son. Several minutes later, the defendant's neighbor stepped outside, and the defendant immediately began yelling about the branch. The irritated neighbor responded, "Your wife is a liar and she shouldn't have told me you'd move the branch if you're not going to." Infuriated, the defendant hurled the football in his hand at the neighbor's head. The football hit the neighbor's neck at an odd angle, breaking his neck and killing him instantly. Of what crime can the defendant be found guilty? (A) First-degree murder. (B) Second-degree murder. (C) Voluntary manslaughter. (D) Involuntary manslaughter.

(D) Involuntary manslaughter. Involuntary manslaughter is an unintentional killing resulting without malice aforethought caused by criminal negligence or during the commission or attempted commission of an unlawful act. Criminal negligence requires that a defendant's conduct involved a high degree of risk of death or serious injury beyond the standard of ordinary tortious negligence. Here, the neighbor's death occurred during the defendant's commission of the unlawful act of hurling the football at the neighbor's head (i.e., battery). From the facts stated, however, it does not appear that the defendant intended to kill, or even to inflict serious injury upon, his neighbor by throwing the football at him. An unintended death which occurs as a result of a battery that is not an attempt to inflict serious bodily injury is considered involuntary manslaughter.

A staff assistant for a state agency was convicted in federal court of taking bribes from a foreign government for the purpose of influencing an upcoming vote on a waterworks bill. He was sentenced to probation. The staff assistant had served in the agency long enough to become fully qualified for his pension upon retirement under the terms of an agreement between the pensions board and the union. The staff assistant retired and immediately started receiving monthly pension checks. Subsequently, the governor signed into law an act which provided in part: "Section 8. Any member of a state agency's staff who is convicted of bribery shall not be entitled to receive any retirement or other benefit or payment of any kind from the state. Such conviction shall be considered a breach of the staff member's employment contract." The staff assistant received a letter from the state which stated that pursuant to this new act, the state was immediately discontinuing pension benefit payments to him on account of his bribery conviction. The staff assistant then contacted an attorney, who challenged the discontinuance of benefits on the grounds that the new law was unconstitutional. To counter one of the attorney's possible arguments regarding the unconstitutionality of Section 8 of the act, which of the following arguments would serve as the state's best rebuttal? (A) The staff assistant was afforded an opportunity to express his views about the new legislation at public hearings prior to the enactment of the statute. (B) Deprivation of pension benefits is not cruel and unusual punishment. (C) A letter sent through ordinary mail is sufficient notice to satisfy due process for the discontinuation of pension benefits. (D) It is implicit that one of the conditions of the state's contract of employment with a state agency staff member is that he shall not engage in bribery.

(D) It is implicit that one of the conditions of the state's contract of employment with a state agency staff member is that he shall not engage in bribery. This question requires a 2-step analysis to select the best alternative. 1st, you must determine the constitutional issue involved. 2nd, you must apply the appropriate constitutional principle to the Q asked--i.e., the state's best rebuttal to a constitutional challenge to the pension forfeiture statute. This answer choice is correct because the argument concerning a condition of employment contract with an agency staff member affects the Contract Clause of the Constitution. The staff member's attorney could validly challenge the constitutionality of the statute, alleging unconstitutional impairment of the obligation to contract. The pension forfeiture statute would be violative of the Contract Clause because under the circumstances, the staff member has satisfied the conditions of retirement eligibility (he became fully qualified for his pension on retirement, as stated in the facts). His retirement pay has ripened into a full contractual obligation and has become a vested right. Therefore, the pension forfeiture statute would be an unconstitutional impairment of his vested right to receive retirement benefits (his pension).

After weeks of deliberation, the defendant decided to rob a local liquor store. The defendant purchased a ski mask and then went into the liquor store, put his gun into the face of the clerk, and demanded the money from the register. Terrified, the clerk gave him the money from the register. The defendant pocketed the money, grabbed a bottle of tequila off the shelf for his celebration, and then ran back to the car and drove off. About an hour later, the defendant was slowly approaching his hideout in the outskirts of town when a young child suddenly darted in front of the defendant's car. He applied the brakes but couldn't stop in time. The car struck the child, killing her. For the child's death, the defendant should be found guilty of which, if any, of the following crimes? (A) Felony murder. (B) Involuntary manslaughter. (C) Voluntary manslaughter. (D) No crime.

(D) No crime. The child's death was unintentional and did not occur as a foreseeable result during the commission of the robbery. The death of the child occurred an hour later, and the defendant was not speeding at the time. Her act of suddenly darting in front of the defendant's car was not a foreseeable consequence of his getaway from the robbery. Therefore, the defendant may not be convicted of felony murder. The defendant lacked the requisite mental state for involuntary manslaughter, namely, gross or criminal negligence. He was driving slowly and applied his brakes at the time of the accident. Nor may he be convicted of voluntary manslaughter since the killing was not intentional. Thus, choice (D) is correct; the defendant will be guilty of no crime.

A husband came home early from work one day. He walked to the bedroom and, upon opening the door, saw his wife and another man in bed together. Visibly upset, the husband told the other man to leave. As the man was reaching for his pants, he pulled out a gun, fearing that the husband was about to attack him. When the husband saw the gun, he jumped on the man and angrily twisted his neck, breaking it and killing him. The husband is guilty for which, if any, of the following crimes? (A) Murder. (B) Voluntary manslaughter. (C) Involuntary manslaughter. (D) No crime.

(D) No crime. In this question, the facts clearly indicate that the other man was the aggressor. When the husband caught the other man in bed with his wife, the husband did not threaten the other man with bodily harm or injury. It was only after the other man brandished his weapon that the husband acted in self-defense and killed him, thus he has not committed any crime.

A stockbroker leased his apartment to a diplomat pursuant to a lease with a one-year term. Shortly after moving into the apartment, the diplomat installed central air conditioning in the apartment. The lease did not address renovations or improvements such as the diplomat's installation of air conditioning. In most jurisdictions, is the diplomat's action permissible under the lease? (A) Yes, because the diplomat improved the value of the premises. (B) Yes, because the lease does not prohibit it. (C) No, because it constitutes permissive waste. (D) No, because it constitutes ameliorative waste.

(D) No, because it constitutes ameliorative waste. A tenant commits ameliorative waste when he acts in a way that increases the value of the premises by permanently altering them. In this case, the diplomat's act of installing central air conditioning would be considered ameliorative waste. Under the law of most jurisdictions, a tenant (except possibly a tenant for the long-term) may not commit ameliorative waste. Therefore, under the majority rule, the diplomat's act was not permissible.

A teenage brother and sister had heard that a locally notorious immigrant, who had become extremely wealthy through a number of ethically questionable business ventures, did not trust American banks and kept all his money in a suitcase in his garage. One night, the teenagers decided to sneak into the man's garage and steal the suitcase full of cash. They jumped the fence surrounding his home and attempted to jimmy the garage door. They had just managed to enter the garage when his dog, alerted by the noises coming from the garage area, began barking furiously, causing the man to come out to investigate. In a panic, the brother grabbed a bicycle that was sitting in the garage and beckoned his sister to get on. They rode away as fast as they could, with the sister perched on the bike's handlebars, as the man chased after them yelling for help. As they turned a corner, however, they were met by a policeman who heard the commotion. Sensing that a crime had been committed, he ordered them to stop, but the brother kept pedaling. The policeman fired a shot at the tire of the bicycle in order to stop the fleeing teenagers, but the bullet struck and killed the sister. Can the brother be charged for his sister's death? (A) Yes, for felony murder, as they committed a burglary. (B) Yes, for involuntary manslaughter, because the brother was negligent in causing his sister's death. (C) No, because they had already left the scene of the felony. (D) No, because the deceased was a co-felon.

(D) No, because the deceased was a co-felon. Felony murder is an intentional or accidental killing proximately caused during the commission or attempted commission of a serious or inherently dangerous felony, such as burglary, arson, robbery, rape, and kidnapping (known by the mnemonic "BARRK"). For a defendant to be guilty of felony murder, the resulting death must be a foreseeable outgrowth of the defendant's actions. Courts have generally applied the foreseeability requirement very liberally, and for purposes of felony murder, most deaths are considered foreseeable. If a death occurs while the defendant is fleeing from the scene of the felony, he may still be guilty of felony murder. However, under the Redline limitation, a felon is not guilty of felony murder where the killing constitutes a justifiable homicide, such as where the police or the victim shoots one of the co-felons. In this case, inasmuch as a policeman shot the sister, his co-felon, the brother will not be criminally liable for his sister's death.

Police observed a customer leave the bar with a plastic cup in his hand and get into his car. The customer pulled into traffic and drove up onto the curb and then made a U-turn going the wrong way on a one way street. The officer pulled the car over and arrested the driver for DUI. The customer was handcuffed and seated in the back of the police car. The officer then conducted a search of the vehicle and found the plastic cup filled with beer under the driver's seat. The defendant filed a pre-trial motion contesting the legality of the search. Should the motion be granted? (A) Yes, because the defendant was already handcuffed and detained in the police car before the search took place. (B) Yes, because the officer did not have probable cause to look under the driver's seat. (C) No, because the search was conducted incident to a lawful arrest. (D) No, because the driver was a recent occupant of the vehicle and the officer had reason to believe that the car contained evidence of the crime for which the defendant was arrested.

(D) No, because the driver was a recent occupant of the vehicle and the officer had reason to believe that the car contained evidence of the crime for which the defendant was arrested. The recent occupant rule, formulated by SCOTUS in the Gant decision [Arizona v. Gant, 556 U.S. 332 (2009)], permits the officer to search a vehicle even if the defendant has been cuffed and detained in the police vehicle. In Gant, the court held "Police may search a vehicle incident to a recent occupant's arrest only if the arrestee is within reaching distance of the passenger compartment at the time of the search or it is reasonable to believe the vehicle contains evidence of the offense of arrest." What this means is that there are two situations in which the police may search the car of a recent occupant following his arrest: (1) if the arrestee is within reaching distance of the car; or (2) if the police believe the car contains evidence of the arrest. However, the search of the passenger compartment may only be conducted if they reasonably believe that evidence of the offense for which the defendant has been arrested is in the vehicle. In this case, the police observed the defendant leave a bar holding a plastic cup and drive in violation of traffic laws. Police certainly had a reasonable belief that evidence of drinking would be found in the passenger compartment, and therefore, the search was lawful.

The owner of a seaside cottage had leased the cottage to a renter for the past three summers. Last year, on October 1, the owner sent the renter a letter stating, "Because you have been such a great tenant, I will rent you the seaside cottage on the same terms as last year. I need your answer by June 1." The renter, uncertain as to when he would be taking his annual seaside vacation, decided to put the letter aside for the time being, intending to respond to it by June 1. On May 15, the owner received an offer from the renter's cousin to rent the seaside cottage at a higher price than the renter had paid the prior year. The owner accepted. On May 20, the renter telephoned the owner and said, "My cousin tells me that he has rented the seaside cottage from you. However, you said that I had until June 1 to decide, and even though your rents are too high, I have decided that I want the cottage for the summer." The owner replied that he had already rented the cottage to the cousin. The renter filed a lawsuit seeking to enforce the contract. Will the renter succeed in his enforcement lawsuit? (A) Yes, because the renter relied on the owner's statement that he needed an answer by June 1. (B) Yes, because the renter contacted the cottage owner on May 20. (C) No, because the renter's response was not the mirror image of the offer and constituted a counteroffer. (D) No, because the owner had already rented the cottage to a different tenant.

(D) No, because the owner had already rented the cottage to a different tenant. An offer is a manifestation of intent by the offeror to be bound by the contract that is communicated to the offeree with definite and certain terms. Here, the owner wrote, "I will rent you the seaside cottage on the same terms as last year. I need your answer by June 1," thereby demonstrating his intent to be bound by a contract. Although the letter itself did not specify the terms, the reference to "the same terms as the previous year" is sufficiently definite to form an enforceable offer. The owner's letter also included the stipulation that he must have a response from the renter by June 1. Generally, promises to keep an offer open are enforceable as long as the offeree pays separate consideration for the option. In this case, the renter did not pay separate consideration for the option, and, as such, the owner was free to revoke the offer at any time before June 1, unless the doctrine of promissory estoppel applies to make his promise enforceable. In order for the doctrine of promissory estoppel to apply, the promise must have been made with the reasonable expectation of inducing reliance, and the offeree must have in fact relied and suffered some detriment as a result of the reliance. Even assuming that the owner made the promise with the reasonable expectation of inducing reliance, there is nothing to indicate that the renter suffered a detriment as a result of his reliance. The renter did not turn down another vacation rental, choose not to make other vacation plans, or incur a cost in expectation of renting the cottage that would demonstrate detriment to the renter. The offer was effectively revoked by the owner's conduct in renting the seaside cottage to the cousin, and by the communication of that revocation to the renter by the cousin. Therefore, there is no contract to enforce.

A saleswoman and an architect entered into an agreement whereby the architect would lease an apartment owned by the saleswoman. The architect moved into the apartment as provided under the lease agreement. However, during the first several months of the lease term, and despite numerous requests from the architect, the saleswoman refused to repair a leaky faucet in the apartment's bathroom. The lease did not address whose responsibility it was to make such repairs. Under the common law, is the saleswoman required to make the repairs? (A) Yes, because under the common law, the landlord has a duty to repair without exception. (B) Yes, because a leaky faucet constitutes ordinary wear and tear. (C) No, because the failure to repair a leaky faucet does not constitute a waste. (D) No, because the saleswoman has no duty to make any repairs to the premises.

(D) No, because the saleswoman has no duty to make any repairs to the premises. Under the common law estate theory, the landlord had no duty to make any repairs to the premises during the lease term. By contrast, the tenant had a duty not to commit waste, which included a duty to repair the premises (including minor repairs to keep the building wind tight and watertight).

A defendant was on trial for drug distribution. The arresting officer was called by the prosecution to describe in detail the events that led to the defendant's arrest. The officer described the night in question, where he was, and what he observed. He was asked to provide a list of what he confiscated from the defendant after he was arrested. The officer testified that the defendant has a large amount of money and many small packets of drugs in several pockets throughout his clothing. When asked specifically about the number of drugs packets, the officer was unable to recall. The officer was then shown his police report. The officer read over his report for several minutes. The prosecutor then asked if the officer was able to remember how many bags of drugs were taken from the defendant. The officer responded that he could not. At this point, the prosecutor directed the office to page three of his report and asked him to read the fifth line, which said, "Confiscated from the defendant's jacket were three purple packets with an off-white powdery substance, and confiscated from his right front pants pocket were 33 purple packets with an off-white powdery substance, that were placed on property receipt #43567." The defense attorney objected to the officer's testimony. How should the court rule? (A) Sustain the objection, because the officer lacks a present memory of the incident. (B) Sustain the objection, and require the report to be admitted because the jury is just as capable as reading the report as the officer. (C) Overrule the objection, because the officer's memory was refreshed. (D) Overrule the objection, because the report is admissible as a past recollection recorded.

(D) Overrule the objection, because the report is admissible as a past recollection recorded. The attempt to refresh the officer's memory was unsuccessful. Therefore, the prosecution needs to introduce the evidence by laying a foundation of the officer's report, and then have that portion of the report read into the record. Although the out-of-court statement is being offered for its truth, the past recollection recorded hearsay exception will provide a means of admissibility.

Until 1954, a state required segregation in all public and private schools, but all public schools are now desegregated. Other state laws, enacted before 1954 and continuing to the present, provide for the free distribution of textbooks to students in all public and private schools. In addition, the state accredits schools and certifies teachers. One private school that offers elementary and secondary education in the state denies admission to all non-Caucasians. Which of the following is the strongest argument against the constitutionality of the free distribution of textbooks to the students at the private school? (A) No legitimate educational function is served by the free distribution of textbooks. (B) The state may not aid private schools. (C) The Constitution forbids private, as well as public, bias in education. (D) Segregation is furthered by the distribution of textbooks to these students.

(D) Segregation is furthered by the distribution of textbooks to these students. SCOTUS has held that a state program in which the state bought textbooks and lent them to students in both public schools and private schools, including private schools that practiced racial discrimination, was unconstitutional [Norwood v. Harrison, 413 U.S. 455 (1973)]. The majority opinion stated that a state cannot give significant aid to schools that practice racial or other invidious discrimination.

A fast-food ethnic restaurant chain wanted to construct a restaurant on Route 44, a business-zoned, secondary road running through the town. It purchased a buildable lot with sufficient frontage and submitted engineering and architectural plans to the appropriate town authorities. The building inspector, backed by the town planning board, denied the restaurant chain's application for a building permit, stating the following reasons in a written decision: (1) Route 44 cannot sustain any further traffic entering and exiting businesses on the road; (2) Route 44 currently has 17 fast-food restaurants, five of which are in the immediate vicinity of the chain's planned location; and (3) the location of the entrance to the planned chain eatery is too close to a dangerous intersection. In a letter to the town planning board members and building inspector, the fast-food chain requested an opportunity to address each of the stated reasons for the denial of the building permit, but the town authorities responded that the decision was final. The chain then sought a declaratory judgment of its right to address the permit denial. If the chain obtains declaratory relief, what provision would provide the strongest argument? (A) Substantive due process. (B) Equal protection. (C) The Takings Clause. (D) The 14th Amendment.

(D) The 14th Amendment. Under the 5th & 14th Amendments, citizens are protected against the deprivation of life, liberty/property without the due process of the law. Procedural due process guarantees notice & the right to be heard. Because procedural due process is not an answer choice in this question, the 14th Amendment, which extends the right to due process to actions by state governments, is the best answer.

A city was experiencing a severe water shortage. To mitigate the problem, the city enacted an ordinance limiting total water use and prohibiting the use of water by private citizens for any purpose other than that of personal hygiene, cooking, and personal consumption. Use of water for landscaping, gardening, washing cars, and the like was strictly prohibited and punishable by fines of up to $5,000. A homeowner had a thriving vegetable garden which required daily watering. Because she limited her personal water use, she felt justified in limited use of sprinklers to water her garden. The sprinklers were timed to water the garden between 6:00 and 6:20 p.m. every night, before the homeowner got home from work. As her garden was in her backyard, the homeowner was relatively confident that her water use would go unnoticed by town officials, and it did. However, the sprinklers did not escape the attention of a boy and his friend, the seven- and eight-year-old boys who lived on either side of the homeowner. Noting that the homeowner's sprinklers went off like clockwork every night at 6:00 p.m., just when temperatures in the desert town were peaking, the boy and his friend made a habit of putting on their bathing suits and sneaking into the homeowner's backyard to run through the sprinklers, unbeknownst to their mothers and to the homeowner. One evening, in his enthusiasm, one of the boys tripped over the sprinkler spigot and scratched his ankle. The wound went unnoticed by his mother until several days later, when she saw that her son was limping. She took him to the doctor only to learn that he had developed a staph infection and required immediate hospitalization. The boy was hospitalized for over a month before he recovered sufficiently to be released, but his infected leg remained weak and his medical bills were extensive. The boy's mother filed suit against the homeowner to recover her medical expenses and lost wages, citing the homeowner's violation of the ordinance. The court accepted the statute as written. Which of the following statements is most accurate? (A) The boy's mother will prevail, because the homeowner's violation of the ordinance constituted negligence per se. (B) The boy's mother will prevail, because the attractive nuisance doctrine applies. (C) The boy's mother will not prevail, because she was negligent in failing to control her child. (D) The boy's mother will not prevail, because the homeowner did not breach a duty of care or commit negligence per se.

(D) The boy's mother will not prevail, because the homeowner did not breach a duty of care or commit negligence per se. The unexcused violation of a legislative enactment or administrative regulation which is adopted by the court as defining the standard of conduct of a reasonable man, is negligence itself, or as commonly phrased, negligence per se. The violation of a criminal statute constitutes negligence per se where the plaintiff can show that: (1) he was a member of the class of persons the statute was designed to protect; and (2) he suffered the type of harm the statute was designed to prevent. In such instances, statutory violation will be considered sufficient to prove duty and breach of duty, but a plaintiff is still required to show causation and damages. Here, the doctrine of negligence per se does not apply. For one, the statute involved was a civil ordinance, not a criminal statute. Secondly, the type of harm suffered by the boy (an injury occurring when tripping over a sprinkler spigot) was not type of harm that the ordinance was designed to prevent. Neither does it appear that the homeowner breached a duty of reasonable care. As such, the boy's mother will not prevail, either on grounds of negligence per se or ordinary negligence.

A man from State B was injured while driving a vehicle that he purchased from a manufacturer of hybrid technology cars whose principal place of business and place of incorporation were in State A. The man brought a state-law products liability action against the manufacturer in State B, claiming damages of $100,000. Two weeks after being served, the manufacturer removed the case to federal court. The man now seeks to have the case remanded back to state court. How should the court rule? (A) The case should be remanded, because it revolves exclusively around state law. (B) The case should be remanded, because a federal court lacks subject-matter jurisdiction to hear the matter. (C) The case should remain in federal court, because removal is the right of the defendant. (D) The case should remain in federal court, because the defendant is permitted to remove a case that could have originally been brought in federal court.

(D) The case should remain in federal court, because the defendant is permitted to remove a case that could have originally been brought in federal court. While a plaintiff is the master of his complaint (and decides the forum in which he will file a lawsuit), the defendant is not without any say in the matter. The procedure for removal allows a defendant to remove certain cases filed in state court to federal court. A notice of removal must be filed within 30 days after the defendant's receipt of the initial pleading (or an amended pleading, if the pleading as amended gives rise to the grounds for removal) "through service or otherwise," or within 30 days after service of the summons on the defendant, if the initial pleading is not required to be served on the defendant, whichever period is shorter. Here, the plaintiff could have sued in federal court, because there is complete diversity and the amount-in-controversy requirement is satisfied. Therefore, the defendant is allowed to remove within 30 days, which the facts indicate occurred here.

D-Corp, a Michigan corporation, sells its automobiles through independent dealers. D-Corp's principal place of business is in Detroit. Each dealer has a separate dealership agreement, which they respectively executed with D-Corp at the time they were appointed to be dealers. Their agreements entitle them to sell all automobiles manufactured by D-Corp in their areas. This year, D-Corp has produced a new sports car that is a radical departure from its previous automobiles. D-Corp decided that it will not sell this car to its existing dealers, but will instead enter into agreements with new dealers to handle the new sports car line. Twelve dealers in California jointly filed actions for breach of contract, seeking an injunction against D-Corp in the appropriate U.S. District Court. D-Corp moved to dismiss the case for lack of subject-matter jurisdiction, and alternatively, moved to sever the actions. How should the court rule on D-Corp's motions? (B) The court should grant D-Corp's motion to dismiss, because the plaintiffs are not diverse. (A) The court should grant D-Corp's motion to sever, because the claims arise out of the same transaction or series of transactions, and there are common issues of law and fact to all of the claims. (C) The court should deny D-Corp's motion to sever, because plaintiffs are compelled to join their actions if their claims arise out of the same transaction or series of transactions. (D) The court should deny D-Corp's motion to dismiss, because the diversity requirement has been met in this case.

(D) The court should deny D-Corp's motion to dismiss, because the diversity requirement has been met in this case. The fact that 2 or more plaintiffs are citizens of the same state, or 2 or more defendants are citizens of the same state, does not destroy diversity. Diversity requires only that no plaintiff be a citizen of the same state as any defendant. As such, the case will not be dismissed for lack of subject-matter jurisdiction.

Plaintiff, a citizen of Utah, was involved in an auto accident with Defendant, a citizen of Minnesota. The accident occurred in Utah. Pursuant to the applicable Utah long-arm statute, Plaintiff sued Defendant and Insureco, Defendant's auto insurance company, in a Utah state court of general jurisdiction for $80,000. Insurance Co, which was joined with Defendant under applicable state law, is incorporated in Delaware and has its principal place of business in Utah. Both defendants were served with the process on the same day. Twenty-nine days after being served, Insureco and Defendant filed a joint notice of removal to the applicable U.S. District Court. Plaintiff then filed a motion for remand. Should the court grant or deny Plaintiff's motion? (B) The court should deny Plaintiff's motion because Defendant is not a citizen of Utah. (D) The court should grant Plaintiff's motion because Insureco's principal place of business is in Utah. (A) The court should deny Plaintiff's motion because the amount-in-controversy requirement is satisfied. (C) The court should grant Plaintiff's motion because the notice of removal was not filed in a timely fashion.

(D) The court should grant Plaintiff's motion, because Insureco's principal place of business is in Utah. A corporation is deemed to be a citizen of the state in which it is incorporated and the jurisdiction in which it has its principal place of business. As a result of this rule, removal would not be possible for two reasons. The first reason is because Insureco's presence in the dispute destroys complete diversity. The second reason is because defendants sued in their home states cannot remove if the basis for removal would be diversity of citizenship.

A citizen of State A was injured in an auto accident by a citizen of State B. The accident occurred in State A. Pursuant to the applicable State A long-arm statute, the plaintiff sued the defendant and the defendant's auto insurance company in a State A state court of general jurisdiction for $80,000. The defendant's auto insurance company, which was joined with the defendant under an applicable state law, is incorporated in State D and has its principal place of business in State A. Both defendants were served with process on the same day. Twenty-nine days after being served, both defendants filed a joint notice of removal to the applicable U.S. District Court. The plaintiff argued that a federal court did not have jurisdiction and then filed a motion for remand. How should the court rule on the plaintiff's motion? (A) The court should deny the plaintiff's motion, because the amount-in-controversy requirement is satisfied. (B) The court should deny the plaintiff's motion, because there is complete diversity and the amount in controversy is satisfied. (C) The court should grant the plaintiff's motion, because the notice of removal was not filed in a timely fashion. (D) The court should grant the plaintiff's motion, because the defendant auto insurance company's principal place of business is in State A.

(D) The court should grant the plaintiff's motion, because the defendant auto insurance company's principal place of business is in State A. A corporation is deemed to be a citizen of the state in which it is incorporated and the jurisdiction in which it has its principal place of business. As a result of this rule, removal would not be possible for two reasons. The first reason is because the auto insurance company's presence in the dispute destroys complete diversity. The second reason is because defendants sued in their home state cannot remove if the basis for removal would be diversity of citizenship.

The defendant owned a 10-acre farm with an old well that was no longer used. His house was located on the middle of the property 1000 yards from the well. An anonymous tip led the police to believe that there was a gun that killed a murdered drug dealer that was hidden in the bottom of that well. The police searched the well on the defendant's property without a warrant and found the gun. The defendant is charged with murder. Which of the following is true? (A) The defendant had a Fourth Amendment right to a reasonable expectation of privacy because the gun was not visible from off the property. (B) The defendant had a Fourth Amendment right to a reasonable expectation of privacy on his entire plot of land, including the well where the police found the gun. (C) The defendant had no Fourth Amendment right to a reasonable expectation of privacy as to the gun because he had discarded the gun. (D) The defendant had no Fourth Amendment right to a reasonable expectation of privacy in the well, as it is outside the curtilage of the defendant's home.

(D) The defendant had no Fourth Amendment right to a reasonable expectation of privacy in the well, as it is outside the curtilage of the defendant's home. A defendant may assert his Fourth Amendment rights only when he has a reasonable expectation of privacy, which exists where the defendant has standing and where the objects to be seized are not "held out to the public." Any unoccupied or undeveloped area outside of the curtilage, or the living space directly around the home, is not protected by the 4th Amendment. Here, the police searched an area which was well away from his home. As such, the defendant did not have a reasonable expectation of privacy in this well, and this answer choice is correct.

The defendant owned a large plot of land in a rural area, and his house sat on the middle of the property. On the left side of the defendant's property sat an undeveloped wooded area, and on the right side of the defendant's property sat a guest house and a large garden. Recently, the defendant was involved in the robbery of a convenience store, which turned deadly when the store clerk was shot. The defendant took the gun that was used in the robbery and hid it in the woods on his property. After following a number of leads, police suspected that the defendant had played a part in the robbery and murder at the convenience store. They went to his house a number of times, but he refused to let them in or talk to them. The police searched the wooded area on the defendant's property without a warrant and found the gun. Which of the following is true? (A) The defendant had a Fourth Amendment right to a reasonable expectation of privacy because the gun was not visible from off the property. (B) The defendant had a Fourth Amendment right to a reasonable expectation of privacy on his entire plot of land, including the wooded area where the police found the gun. (C) The defendant had no Fourth Amendment right to a reasonable expectation of privacy as to the gun because he had discarded the gun. (D) The defendant had no Fourth Amendment right to a reasonable expectation of privacy in the wooded area on his property, as it is outside the curtilage of the defendant's home.

(D) The defendant had no Fourth Amendment right to a reasonable expectation of privacy in the wooded area on his property, as it is outside the curtilage of the defendant's home. A defendant may assert his 4th Amendment rights only when he has a reasonable expectation of privacy, which exists where the defendant has standing and where the objects to be seized are not "held out to the public." Any unoccupied or undeveloped area outside of the curtilage, or the living space directly around the home, is not protected by the 4th Amendment. Here, the police searched the undeveloped area on the left side of the defendant's property, which was away from his home. As such, the defendant did not have a reasonable expectation of privacy in this wooded area, and this answer choice is correct.

A sailor got into a fight with a marine while at a bar. The fight was quickly broken up by some other patrons. The sailor, however, remained angry. The sailor went back to the same bar the following night. While inside the bar, someone put a substance in the sailor's soda that made him extremely intoxicated. He soon left the bar and, 10 minutes later, ran into the marine on the sidewalk. The two men got into a verbal argument, during which the marine threatened to cause bodily injury to the sailor's girlfriend. The sailor became angry and pulled out a gun, shooting the marine to death. When the sailor woke up the next morning, the effect of the alcohol had mostly worn off. He had no recollection of what happened the night before. He did not remember having a gun, or using it to shoot the marine. This jurisdiction defines all murder as second-degree murder, but elevates intentional killings and those committed during an inherently dangerous felony to first-degree murder. Manslaughter is defined as at common law. Which of the following statements is most accurate? (A) The sailor is guilty of first-degree murder, because he intentionally killed the marine. (B) The sailor is guilty of second-degree murder, because, due to his intoxication, he could not formulate the specific intent to kill. (C) The defendant is guilty of voluntary manslaughter, because there was sufficient adequate provocation by the marine due to the threats made concerning the sailor's girlfriend. (D) The defendant is not guilty of homicide, because he lacked the state of mind required to commit the crime.

(D) The defendant is not guilty of homicide, because he lacked the state of mind required to commit the crime. Whereas voluntary intoxication may, in limited circumstances, prevent a defendant from forming the specific intent required for certain crimes, involuntary intoxication may be a complete defense to a variety of charges. Involuntary intoxication will excuse what would normally be criminal conduct if it: (1) prevents the defendant from understanding what he or she is doing; (2) causes the defendant to be unable to differentiate between right and wrong; (3) makes the defendant incapable of complying with the law; or (4) otherwise leaves the defendant lacking the culpable state of mind required for a conviction. Here, because the sailor became extremely intoxicated as a result of a substance that someone else put into his drink without his knowledge, his involuntary intoxication will operate as a defense to the homicide charge.

A state has a policy of hiring new state attorneys on a probationary basis for a two-year period. Prior to the end of those two years, the lawyer can be let go for any reason. After those two years, the lawyer receives the added job security of only being able to be fired "for cause". The state has followed no consistent pattern with respect to the termination of probationary employees. Some were fired with no explanation or warning. Others were given hearings after notice, during which their job performance as probationary employees was discussed. If a probationary employee who was terminated without a hearing challenged the constitutionality of the state's action, how should the court rule? (A) The employee would succeed, because she was denied a notice and hearing. (B) The employee would succeed, because the state's action is an act of attainder. (C) The employee would not succeed, because a state's procedure with respect to state employees is a matter reserved to the state under the Tenth Amendment. (D) The employee would not succeed, because she does not have a right to notice and a hearing protected by the Due Process Clause of the Fourteenth Amendment.

(D) The employee would not succeed, because she does not have a right to notice and a hearing protected by the Due Process Clause of the 14th Amendment. An employee who has no expectation of continued employment (i.e., one who has been hired on a probationary basis) has no due process rights regarding termination. Here, because probationary employees have no expectation of a permanent job when they are hired by the state, they have no property right, and therefore, no entitlement to a hearing, so the state's discharge procedure need not be consistent.

A father took his son to an outdoor concert put on by a famous pop singer. The father left his son in his seat and went to the bathroom. While in the bathroom, the father saw a man placing what he believed to be a bomb inside a trash can. The father immediately went back to his seat, grabbed his son, and left the concert. On the way to his seat and to the exit, the father passed several security guards, but said nothing to them as to what he had seen. Shortly after leaving, the bomb exploded, killing several concertgoers and injuring dozens more. The father felt bad for what transpired, and confessed to a police officer, who then arrested him. In this jurisdiction, homicide is defined as at common law. Which of the following is most accurate? (A) The father is guilty of manslaughter, because his failure to act was grossly negligent. (B) The father is guilty of depraved-heart murder, because his conduct evidenced a wanton disregard for the value of human life. (C) The father is guilty of murder, because he believed the man was placing a bomb inside the garbage can. (D) The father is not guilty of any crime.

(D) The father is not guilty of any crime. The father did not have a legal, enforceable duty to warn the others about the bomb. As such, he committed no crime.

A six-year-old girl was kidnapped outside her home in a small town. Following the girl's abduction, her parents publicly announced a $50,000 reward to anyone responsible for the apprehension of the kidnapper. The girl's abductor was described as a Caucasian male between the ages of 25 and 30 with curly blond hair. He also was identified as having a pentagram tattoo on his left arm. One afternoon, while on duty, a police officer from a nearby town stopped at a local fast-food restaurant for lunch. Inside the restaurant, the police officer noticed a man fitting the description of the kidnapper. The police officer proceeded to arrest the individual, who, in fact, turned out to be the girl's abductor. After the girl was found unharmed, the police officer requested the $50,000 reward. However, the girl's parents have refused to pay the police officer the reward money. If the police officer brings suit against the girl's parents to recover the $50,000 reward, for whom should the court enter judgment? (A) The police officer, because he accepted the offer by apprehending the kidnapper. (B) The police officer, if he knew of the existence of the reward offer when he apprehended the kidnapper. (C) The girl's parents, if their intent was to make the offer open only to members of the general public. (D) The girl's parents, because recovery by the police officer is against public policy.

(D) The girl's parents, because recovery by the police officer is against public policy. This question deals with the preexisting duty rule. In this question, the issue is whether a police officer on duty is entitled to collect a reward for the apprehension of a criminal. The answer is no if the apprehension occurs while the officer is performing his regular duties. Under the preexisting duty rule, an officer is legally obligated to apprehend criminals as part of his job responsibilities. As a result, the police officer may not obtain a reward for the capture of the abductor. Note that if the law enforcement officer apprehends a criminal while on vacation, then there is no preexisting duty and he is entitled to collect the reward offer.

A homeowner said to a roofer, "My roof leaks. I think the old tiles are cracked. If you will replace them with all new tiles, I will pay you $5,000." The roofer replied, "Sure, if I can clear my busy schedule." The homeowner then remarked, "That's all right, but let me know soon." Three days later, the roofer drove his pickup truck to the homeowner's home and unloaded the materials and equipment needed to perform the roofing job. When the homeowner looked out his window and saw what was transpiring, he immediately ran outside and exclaimed, "Stop! The deal's off. I decided to repair the roof myself." In an action by the roofer against the homeowner for breach of contract, which of the following would provide the roofer with his best theory of recovery? (A) A bilateral contract was formed when the roofer purchased the materials and equipment needed to do the job. (B) A bilateral contract was formed when the roofer said, "Sure, if I can clear my busy schedule."I (C) The homeowner made an offer that proposed a unilateral contract, and the offer became irrevocable when the roofer purchased the materials and equipment needed for the job. (D) The homeowner made an offer that proposed a unilateral contract, and the roofer manifested an intent to accept the offer when he began performance by unloading the materials and equipment at the homeowner's house.

(D) The homeowner made an offer that proposed a unilateral contract, and the roofer manifested an intent to accept the offer when he began performance by unloading the materials and equipment at the homeowner's house. A unilateral offer which invites performance of an act as acceptance, rather than a return promise, becomes irrevocable as soon as the offeree has started to perform the act. Thus, this answer choice correctly describes the theory that would be most helpful to the roofer.

A man and his roommate got into an argument over a loan the man-made to the roommate. The roommate called the man a spoiled brat who always got his way. The man slapped the roommate across the face. The roommate then grabbed a carving knife from the kitchen counter and tried to stab the man. The man wrestled the knife away and stabbed the roommate in the chest, killing him. If the man is charged with homicide, which of the following statements is most accurate? (A) The man is guilty of murder, because deadly force was not justified. (B) The man is guilty of manslaughter, because deadly force was not justified. (C) The man is guilty of murder, because he was the initial aggressor. (D) The man is not guilty of any crime, because deadly force was justified.

(D) The man is not guilty of any crime, because deadly force was justified. The man provoked the attack by slapping the roommate across the face. However, the slap was nondeadly force. In a majority of jurisdictions, the roommate cannot respond to nondeadly force with deadly force, such as by using a knife. The roommate used excessive force in his response to the man's slap, and so the man can use deadly force to defend himself, and will not be responsible for criminal homicide under these circumstances.

A landlord owned a three-story home in fee simple and leased it to his friend for a term of 10 years. By the terms of the lease, the friend expressly covenanted to pay a monthly rental of $300 and to pay the taxes on the premises during the term of the lease. The lease also stipulated that the friend, as lessee, could not assign or sublease the said premises. The friend and his family lived in the house for two years. Then the friend, still owing the landlord three months back rent, assigned his leasehold interest in the property to a professor, who was unaware of the prohibition against assignments. This written assignment expressly provided that the professor would pay the landlord the monthly rental of $300, but was silent concerning the payment of taxes. The landlord never objected to this assignment. Six months after the assignment, the landlord conveyed his property interest in the premises to a new owner. After residing in the house for a year, the professor subleased the third floor to a student for a term of two years; the agreement provided that the professor would receive a monthly rental payment of $100. After the student had made regular rental payments to the professor for the first six months of the sublease, he defaulted. The following month, the professor sent the new owner $200 as payment for rent and notified him that he should collect the remaining $100 from the student, who refused to vacate the premises. In an action by the new owner against the student to recover the additional $100 for rent due, what is the most probable outcome? (A) The new owner will recover, because the landlord and sublessee are in privity of estate. (B) The new owner will recover, even though he never objected to the sublease. (C) The new owner will not recover, because the professor would only be held liable.This answer is incorrect. (D) The new owner will not recover, because the professor remains liable.

(D) The new owner will not recover, because the professor remains liable. Because the sublessee (the student) is not in privity of estate with the landlord, the sublessee is not subject to the covenants or terms of the leasehold agreement. Therefore, the professor will remain liable for the entire $300 rental.

The owner of a large social function hall hosts various banquets, meetings, and other types of community events in the social function hall. An entrepreneur is in the business of running bingo games sponsored by charities, a legal form of gambling in the state in which the social function hall is located. Without disclosing his intended use for the hall, on June 1, the entrepreneur entered into a one-year lease of the social function hall commencing on July 1. On June 15, a scandal erupted in the state in which the social function hall was located concerning corruption in bingo games sponsored by charities, and the state legislature passed legislation, effective June 30, outlawing bingo games. Upon passage of the legislation, the entrepreneur purported to cancel the lease. In a suit by the owner against the entrepreneur, what is the likely outcome? (A) The entrepreneur will prevail, because after the lease had been signed, the government made its subject matter illegal. (B) The entrepreneur will prevail, because the contract is voidable under the doctrine of frustration of purpose. (C) The owner will prevail, because the purpose of the contract was not illegal at the time that the contract was formed. (D) The owner will prevail, because the parties formed a valid contract supported by consideration.

(D) The owner will prevail, because the parties formed a valid contract supported by consideration. Because neither the doctrine of impossibility nor frustration of purpose will excuse the entrepreneur's performance, and the subsequent illegality of bingo games will have no effect on the lease, the owner will prevail.

A county decided to revamp its voting procedures in order to streamline the process. The county ordered new touch-screen voting machines, solicited more volunteers for election days, lengthened voting hours, made vote-by-mail available to all residents, and enacted various other rules to help make sure that election days ran more smoothly. One new rule required people to check in at the polls at the table labeled with their party affiliation: Republican, Democrat, or Other. A voter in that county objected to this particular rule. She was a Democrat who sent her child to a school in her neighborhood that was very heavily populated with Republicans. She served on the board of the parent-teacher organization at the school. The voter did not wish for her party affiliation to be public in case it might cause controversy for herself or her child. Assuming proper standing, if the voter challenges the constitutionality of the rule requiring voters to line up according to party affiliation at the polls, what is the likely outcome? (A) The rule will be invalidated, because it violates her right to privacy. (B) The rule will be invalidated, because it violates her procedural due process rights. (C) The rule will be upheld, because it passes strict scrutiny. (D) The rule will be upheld, because it passes the rational basis test.

(D) The rule will be upheld, because it passes the rational basis test. The court is likely to uphold this rule, because the county will only need to show a rational relationship between the rule and the county's objective to streamline voting procedures. Strict scrutiny applies when the government restricts the right to vote, a right that has been held to be fundamental under the Constitution. However, rational basis scrutiny applies to reasonable restrictions based on things like age or duration of residency. Moreover, voter registration requirements and regulation of the time, place, and manner of casting ballots are valid so long as they do not impose an undue burden on the right to vote. Here, there is no restriction at all on the right to vote, other than the concern that others will know your party affiliation. Because the county made vote-by-mail an option for everyone and because voter registrations are public records anyway, it seems very unlikely that this rule will be held to violate the Constitution. There is no constitutionally protected privacy right as to party affiliation.

Three friends decided to go bowling together. They made a friendly wager on the match, with the winner receiving a free beer from the other two. In the second frame of the first game, the first friend apparently got a strike when he knocked down all 10 pins. However, the second friend accused the first friend of fouling because his foot went over the line. The first friend denied fouling and wanted to mark down a strike on the scorecard. The second friend refused to give the first friend a strike and wrote in a zero on the scoring sheet. The first friend became enraged. The second friend then went to bowl his frame. As the second friend turned his back, the first friend approached from behind with a bowling ball in his hand. The first friend then raised the bowling ball, threatening to hit the back of the second friend's head. The second friend, who had his back turned, did not see the first friend's actions. The third friend saw what the first friend was about to do and could easily have warned the second friend, but the third friend remained silent. The first friend then struck the second friend on the back of the head with the bowling ball, causing a deep scalp wound. If the second friend asserts a cause of action against the third friend, what is the likely outcome? (A) The second friend will prevail, because the third friend was aware of the danger. (B) The second friend will prevail, on account of the third friend's omission to act. (C) The second friend will not prevail, because the first friend was responsible for his injury. (D) The second friend will not prevail, because the third friend was under no duty to warn the second friend of the danger.

(D) The second friend will not prevail, because the third friend was under no duty to warn the second friend of the danger. The fact that the defendant realizes or should realize that action on his part is necessary for another's aid or protection does not by itself impose a duty to take such action. There are situations where a special relationship between the parties (e.g., common carrier-passengers and innkeeper-guests) will impose a duty to act. However, in this question there is no special relationship between the parties. Despite the fact that the third friend was aware of the danger, he did not have a duty to warn the second friend. Thus, this answer is correct.

Concerned by the number of large commercial properties being purchased by foreign nationals, a state legislature enacted a statute prohibiting the sale of commercial properties in excess of 20,000 square feet to aliens. A resident alien who has lived in the state for several years entered into a contract with his business partner to buy a commercial manufacturing business in the state. The factory was 30,000 square feet in size. The resident and the business partner decide to test the constitutionality of the state statute, and so sought a declaratory judgment in federal court. Which of the following statements regarding the declaratory judgment action is most accurate? (A) The business partner has the burden of showing that there is no rational basis for the statute's purposes regarding large commercial properties. (B) The court will find that the resident, but not the business partner, has standing. (C) The resident has the burden of showing that the state has no compelling interest in carrying out the statute's purposes regarding large commercial properties. (D) The state has the burden of showing a compelling state interest in carrying out the statute's purposes regarding large commercial properties.

(D) The state has the burden of showing a compelling state interest in carrying out the statute's purposes regarding large commercial properties. When a state statute discriminates against a suspect class such as aliens, the burden is on the state to show that its actions are necessary to achieve a compelling state interest. In this case, the state statute may violate the Equal Protection Clause by prohibiting aliens from acquiring large commercial properties. The federal court hearing the resident and the business partner's case will apply the strict scrutiny test to the statute to determine whether the law violates the constitutional guarantee of equal protection.

The plaintiff has held various public offices for many years, and is currently a state representative. After a recent scandal broke involving inappropriate text messaging with a minor, the plaintiff was convicted of felony obscenity charges, and was sentenced to time served and placed on probation. A week after the conviction was announced, the plaintiff was informed that, pursuant to a state statute, he was being immediately suspended from office without pay until such time as he had exhausted his appeals. Should his conviction become final, another statute authorized the attorney general to file suit to permanently remove the plaintiff from office, following a hearing on the matter and appellate review of the hearing decision. The plaintiff filed suit to prevent his suspension, claiming that the state statute was an unconstitutional bill of attainder and a violation of his due process rights. How should the court rule? (A) The plaintiff will prevail, because the statute violated his procedural due process rights. (B) The plaintiff will prevail, because the statute was an unconstitutional bill of attainder. (C) The state will prevail, because the statute provided for a judicial hearing. (D) The state will prevail, because the statute was not a bill of attainder.

(D) The state will prevail, because the statute was not a bill of attainder. Sections 9 and 10 in Article I of the U.S. Constitution bar both federal and state legislatures from passing bills of attainder. A bill of attainder is a legislative act that inflicts punishment without a judicial trial upon named individuals or an easily ascertainable group for past conduct. However, courts have held that laws suspending felons from public office or denying them the right to vote are not penal in nature, as their purpose is more to protect the integrity of the political process rather than to punish the criminal [Spooner v. West Baton Rouge Parish School Bd., 709 F. Supp. 705 (M.D. La. 1989); Trop v. Dulles, 356 U.S. 86 (1958)]. As such, the suspension statute in this state is not an unconstitutional bill of attainder.

A woman called 911, telling the operator that her husband had hit her, threatened her children, and was now breaking objects in the living room. Once the husband saw her calling 911, he fled, and the caller told the operator where he went. The husband was arrested and charged with assault. At trial, the woman did not testify. Instead, the prosecution called the 911 operator to play the recording of what the woman said in her call. The husband's attorney objected to the admission of the wife's call to the police. How should the trial court rule? (A) The statement is inadmissible because in criminal cases, all evidence presented against the defendant must be subject to cross-examination in front of the trier of fact. B) The statement is inadmissible because it was testimonial in nature. (C) The statement is admissible, as a statement made while under the stress of a startling event. (D) The statement is admissible because it was not testimonial in nature and qualifies as an excited utterance.

(D) The statement is admissible, because it was not testimonial in nature and qualifies as an excited utterance. This call would be nontestimonial in nature (it is describing current events to help police deal with an emergency), and so everything the wife said, including the identity of the attacker, would be admissible in court. Therefore, even if she does not show up in court, the prosecutor can play the tape in order to get a conviction.

A landlord entered into a lease agreement with a tenant pursuant to which the landlord was required to pump out and empty the house's small septic tank every six months, on January 1 and July 1. During the second year of the lease, the landlord failed to pump the tank in July despite repeated complaints by the tenant, and as a result, the property was suffused with a foul order for the entire month. The tenant remained in the apartment. When the landlord finally arrived to pump the tank at the beginning of August and demanded the previous month's rent, the tenant refused to pay. The landlord then sued the tenant for the unpaid rent. Under the majority rule, what remedy is the tenant entitled to pursue? (A) The tenant is entitled to a rent abatement, because the landlord has breached the covenant of quiet enjoyment, but he is not relieved of all liability for rent. (B) The tenant may withhold the entire rent, because he has been constructively evicted. (C) The tenant may not withhold the entire rent, because he has merely been constructively evicted. (D) The tenant may not withhold the entire rent, because he remained in the apartment.

(D) The tenant may not withhold the entire rent, because he remained in the apartment. Under the majority rule, a tenant may treat a lease as terminated if the landlord breaches the covenant of quiet enjoyment (express or implied in every lease) by constructive eviction of the tenant. In order for a constructive eviction to exist: (1) the landlord must substantially and permanently interfere with the tenant's use and enjoyment of the premises; and (2) the tenant must move out. In this case, because the tenant did not move out, he has not been constructively evicted, and therefore may not withhold the rent.

A landlord and tenant entered into a residential lease agreement, which provided that the landlord was required to provide heat to the apartment during the winter months. During the entire month of December, there was no heat in the apartment, despite the fact that the tenant complained to the landlord on numerous occasions. The tenant remained in the apartment. Under the majority rule, what remedy is the tenant entitled to pursue? (A) The tenant may withhold the entire rent, because she has been constructively evicted. (B) The tenant is entitled to a rent abatement, because the landlord has breached the covenant of quiet enjoyment but is not relieved of all liability for rent. (C) The tenant may not withhold the entire rent, because the tenant has merely been constructively evicted. (D) The tenant may not withhold the entire rent, because she remained in the apartment.

(D) The tenant may not withhold the entire rent, because she remained in the apartment. Under the majority rule, a tenant may treat a lease as terminated if the landlord breaches the covenant of quiet enjoyment (express or implied in every lease) by constructive eviction of the tenant. In order for a constructive eviction to exist: (1) the landlord must substantially and permanently interfere with the tenant's use and enjoyment of the premises; and (2) the tenant must move out. In this case, given the tenant did not move out, she has not been constructively evicted and may not withhold the rent.

A landlord leased an apartment in a high-rise building to a tenant. The jurisdiction in which the leased premises were located had enacted a statute requiring landlords to provide running water in leased apartments for no less than 25 days out of each month. The lease agreement itself did not contain any term involving running water. In early June, while making repairs on another apartment, the landlord accidently burst a water pipe leading into the tenant's apartment. As a result, the tenant did not have running water for the last three weeks of the month, and was forced to move out of the apartment. The tenant then sued the landlord for violating the lease's covenant of quiet enjoyment. (A) The only remedies available to the tenant are those provided for in the statute, because the landlord has not violated the lease. (B) The only remedies available to the tenant are those provided for in the statute, because the tenant vacated the apartment. (C) The tenant is entitled only to a rent abatement, because she has been constructively evicted. (D) The tenant may treat the lease as terminated and withhold rent, because she has been constructively evicted.

(D) The tenant may treat the lease as terminated and withhold rent, because she has been constructively evicted. A constructive eviction occurs when: (1) the landlord acts to substantially and permanently interfere with the tenant's use and enjoyment of the premises; and (2) the tenant moves out. Withholding something required by statute will constitute constructive eviction. In this example, while a requirement to provide running water was not included within the terms of the lease itself, it was required by statute. Therefore, the landlord's failure to properly supply running water for the specified number of days constitutes a constructive eviction. In most jurisdictions, a tenant who has been constructively evicted may treat the lease as terminated and withhold the entire rent.

A concert pianist suffered a strained wrist, so he went to visit a doctor. The doctor explained to her that the fastest way to regain complete use of the wrist was to treat it with a series of injections of a drug that blocked pain and reduced inflammation. The concert pianist had discussed with the doctor her need to be ready for an important recital in three weeks. The drug was known to cause calcium buildup in joint tissue in approximately one to two percent of patients treated with it. For most individuals experiencing this side effect, the calcium buildup did not result in noticeable limitation of movement. If it did, the buildup could be surgically removed, and after a brief recovery period (one to two months), complete movement was restored. The doctor did not inform the concert pianist of these facts, and she began a course of treatment consisting of three injections per week for three weeks. At the end of the second week, the concert pianist received in the mail the current edition of a popular health magazine. To her dismay, it contained an article describing the dangers of using the drug that the doctor recommended, pointing out that calcium buildup in the knuckles could be devastating to a pianist. The concert pianist discontinued treatment immediately and was not able to perform in her upcoming recital due to pain in her wrist, but not because of any calcium buildup in her knuckles. The concert pianist sought treatment from another physician, who recommended a two-month hiatus from piano playing followed by six months of physical therapy. After that time, the concert pianist was able to resume her playing. Which of the following would be most relevant in determining whether the concert pianist has a valid cause of action against the doctor? (A) Whether the concert pianist would not have consented to the use of the drug if she knew about the risks to her mobility. (B) Whether a reasonable person would have considered the information about calcium buildup important. (C) Whether the doctor had weighed the risks and benefits of using the drug before prescribing it. (D) Whether the concert pianist suffered no harm from the use of the drug.

(D) Whether the concert pianist suffered no harm from the use of the drug. If the concert pianist had been injured by the drug injections, the doctor could have been held liable for failure to disclose the material risks involved. Physicians owe their patients the duty to inform them of the risks involved in a proposed medical treatment. The disclosure should include the diagnosis, the risks of the proposed treatment, the risks of alternative treatments, and the risk of no treatment. Most jurisdictions now define the duty to disclose in terms of the patient's right to know. These jurisdictions look to whether a reasonable person in the patient's position would attach significance to the information. If that reasonable patient would, then the information is material and disclosure is required. In addition to proving that material risks were not disclosed, the plaintiff must show that she was harmed by the treatment and that she would have withheld her consent had she known of the risk of that harm, which in fact occurred. In this case, the concert pianist has no cause of action against the doctor because she has not suffered any harm.

Four men entered into a lease as tenants of a five-bedroom house. They each had a separate bedroom and used the fifth bedroom as a laboratory to manufacture the illegal drug methamphetamine. One of the men was the so-called moneyman; he provided the funds to make the rental payments and to purchase the laboratory equipment. One of the other men was the chemist; he had both an undergraduate and graduate degree in chemistry and knew the formula and procedure for producing methamphetamine. Another of the men had sold drugs in the past and knew potential buyers of the methamphetamine. Finally, the fourth man was the enforcer; his role in the scheme was to make sure that no unauthorized persons entered onto the premises of the house. The four men had been involved in this drug venture for seven months when they were finally arrested and charged with conspiracy to manufacture and distribute methamphetamine, and distribution and manufacture of methamphetamine. During the trial, the prosecution wishes to introduce, as evidence against the moneyman, a statement made by the enforcer to the landlord at the time of the signing of the lease, in which the enforcer said, "No matter what you charge us for rent, you better O.K. it with the moneyman because he's the one who will really be paying it." The moneyman's attorney objects. The judge's ruling on admissibility will depend on what fact? (A) Whether the enforcer actually paid all of the rent. (B) Whether the enforcer had a legal obligation to pay the rent under the terms of the lease. (C) Whether the landlord may be considered a party opponent. (D) Whether the statement was made during the course of and in furtherance of the conspiracy.

(D) Whether the statement was made during the course of and in furtherance of the conspiracy. If the enforcer's statement was made during the course of and in furtherance of the conspiracy, the statement would be admissible as an opposing party's statement. Under FRE 801(d)(2)(e), "a statement is not hearsay if the statement is offered against a party and is....a statement by a co-conspirator of a party during the course and in furtherance of the conspiracy." The enforcer's statement concerning the fact that the moneyman would be paying the rent for the premises where the methamphetamine was being manufactured would certainly fall within the "course of and furtherance of" the conspiracy to manufacture and distribute methamphetamine. All other statements would be incorrect as to prove the admissibility of the enforcer's statement against co-conspirator, the moneyman.

A new husband and wife took a ferry ride on their honeymoon. As they were descending the observation deck, the husband slipped on some water on the stairs. The husband alleged that he sustained severe back injuries from the fall, and he sued the ferry owner. Although the ferry owner admitted that the husband was injured by slipping on the water, the ferry owner denied negligence and claimed that the husband was contributorily negligent. The wife died before the trial. At trial, the ferry owner called the ferry captain to testify that just before the husband fell, the captain heard someone call out, "Be careful, the stairs are wet." The husband offered into evidence his wife's deposition testimony that "I did not hear anyone warn my husband about the water on the stairs." Is the deposition testimony admissible? (A) No, because the wife is not subject to cross-examination. (B) No, because the testimony is hearsay not within any exception. (C) Yes, as a dying declaration. (D) Yes, as former testimony.

(D) Yes, as former testimony. This is the best choice because Federal Rule of Evidence 804(b)(1) provides that testimony given by a witness in another hearing (including a hearing in a different proceeding) or taken in a deposition in the same (or different) proceeding is excepted from the hearsay rule if: (1) the former witness is unavailable in the present proceeding; and (2) the party against whom the former testimony is offered (or his predecessor in interest) had an opportunity to develop the former testimony by direct, redirect, or cross-examination and had a similar motive to develop the former testimony by cross-examination. Here, the wife is unavailable at trial and was subject to examination by the ferry owner at the deposition. Therefore, the former testimony exception applies.

Strong Corporation is in the business of providing exercise instruction and exercise facilities and equipment based on techniques developed by Joe Strong in the 1920s. Strong Corporation owns two registered trademarks, "The Strong Way," which designated its exercise instruction services, and "Strong Studios," which designated facilities for exercise and physical conditioning. Strong Corporation wanted to sue The Strong Institute for trademark infringement for naming its business The Strong Institute and for selling videos and other equipment that purported to teach the Strong method. The Strong Institute was incorporated in Delaware and had its principal place of business within the Central District of Pennsylvania. All of The Strong Institute's studios are in Pennsylvania, and it does the majority of its business there. However, within the past six months, The Strong Institute started working on a plan to open studios in New York City. In furtherance of that plan, The Strong Institute had started advertising on New York City buses and sent targeted direct-mail brochures to New York City gyms and gym members. Strong Corporation wants to sue The Strong Institute in the Southern District of New York, where New York City is located and Strong Corporation resides. Is venue appropriate there? (A) No, because the majority of the trademark infringement occurred in Pennsylvania. (C) Yes, because Strong Corporation, the plaintiff, resides in the Southern District of New York. (D) Yes, because The Strong Institute targeted New York City through its advertising. (B) No, because The Strong Institute, the defendant, resides in the Central District of Pennsylvania.

(D) Yes, because The Strong Institute targeted New York City through its advertising. For civil actions where jurisdiction is based in part on a federal question, such as trademark, the available venues are: (1) the district where any defendant resides, provided that all defendants are located in the same state; (2) the district where a substantial portion of events or omissions giving rise to the claim occurred; or (3) the district where any defendant can be found. Courts have found that if a defendant targets a particular district by advertising and actively pursuing efforts to market the offending product there, such actions may be regarded as constituting a substantial part of events giving rise to the claim. Because The Strong Institute targeted the Southern District of New York through advertising and direct mail, venue would be proper in that district.

After an automobile accident, the plaintiff brought suit against the defendant in the United States District Court for State X, where venue was proper. After being served, the defendant consulted his attorney and they felt that a change of venue would be better. The defendant then filed a motion to transfer venue to the United States District Court for the District of State Y. Both parties agreed that venue would have been proper there, and that it would be more convenient for the parties and witnesses. The court scheduled a pretrial conference to settle the issue. Two weeks later at the pretrial conference, the plaintiff and defendant informed the court that they had agreed that the case should be transferred to the United States District Court for the District of State Z, which all parties agreed was not a proper venue. The plaintiff and defendant each thought that a State Z jury pool would be more sympathetic to their case. May the court transfer the case to the United States District Court for the District of State Z? (A) No, because that court is not a court where the case might have originally been brought. (B) No, because venue is proper in the district where the case was originally filed. (C) Yes, because a federal district court may transfer a case to another federal district court for the convenience of parties and witness and in the interest of justice. (D) Yes, because all parties have agreed that the court may do so.

(D) Yes, because all parties have agreed that the court may do so. The venue transfer statute at one time limited the court's options on transfer to districts where venue would have been proper originally. However, the statute now allows the court to transfer to any such district or to any district where all parties consent to venue.

A man pulled his truck onto the freeway and collided with a car. The driver of the car was seriously injured in the accident, and subsequently filed suit against the truck driver, alleging that he failed to yield as he entered the freeway from the entrance ramp. The truck driver, in turn, claimed that the driver of the car was speeding and had failed to make any attempt to avoid the accident even though the two lanes to the left of the entrance ramp were wide open. The truck driver's attorney called the truck driver to testify that when he approached the car immediately following the accident, the driver of the car stated over and over, "I should have slowed down or moved over." The driver of the car objected. Should the truck driver be allowed to testify as to the driver of the car's statement? (A) No, because it is inadmissible hearsay. (B) No, because the truck driver is an interested party and there is no corroboration of the statement. (C) Yes, because it shows awareness by the driver of the car that he was speeding. (D) Yes, because it is a statement by an opposing party.

(D) Yes, because it is a statement by an opposing party. Under FRE 801, a statement by an opposing party that is offered against that party is non-hearsay. Here, the car driver's statement constitutes an opposing party's statement, and will therefore be admissible against him at trial

A CEO was addressing a large group of shareholders at an annual meeting in which his opening remarks discussed the health of the business, some recent acquisitions, and his personal health. "It was a long trip back," he said, "but I am feeling great, ready to take on the world." This news immediately hit the media and stock prices jumped 2%, as there was speculation that the CEO was suffering from severe depression. The following morning, the CEO was found dead in his hotel room. After his death, the CEO's wife attempted to collect on his life insurance policy, but her claim was denied due to speculation that the CEO committed suicide. At trial, the wife sought to use the CEO's statement to the shareholders that he was feeling great and ready to take on the world. The insurance company objected. Should the trial judge admit the testimony? (A) No, because there was speculation that he suffered from depression. (B) No, because it was not made to a doctor. (C) Yes, because it is relevant to the CEO's medical condition. (D) Yes, because it is a statement of the CEO's then-existing physical condition.

(D) Yes, because it is a statement of the CEO's then-existing physical condition. FRE 803(3) allows for admission of an out-of-court statement describing the declarant's then-existing mental, emotional, or physical condition. This includes a statement of the declarant's then-existing state of mind (such as motive, intent, or plan) or emotional, sensory, or physical condition (such as mental feeling, pain, or bodily health), but not including a statement of memory or belief to prove the fact remembered or believed unless it relates to the validity or terms of the declarant's will. Here, the proponent (the CEO's wife) is attempting to show that her husband was not likely to have killed himself based on his comment the day before about how he was feeling. Because she wants to offer this statement for its truth and it discusses how the CEO was currently feeling, it falls within this exception.

A man was arrested for sabotaging an electrical plant and causing a blackout in the city. The police suspected that there was another electrical plant that was also a target for sabotage but could not make the man admit his guilt or help the police in any way. A rookie police officer feared the harm to the community if there was another blackout and searched the man's home and office without a warrant. He found a computer file with information on the next target and how to stop the sabotage. Five minutes later, other police officers who did not know of the rookie police officer's action and pursuant to a search warrant were conducting a systematic search of the man's property beginning with his house and then his office. Will the computer file be admissible as evidence in the man's subsequent criminal trial for sabotaging the electrical plant? (A) No, because it is "fruit of the poisonous tree" and therefore tainted. (B) No, because the officers who had the search warrant were not the ones who discovered the document. (C) Yes, because there was an urgent need to search the man's office in order to neutralize the threat to the electrical plant as quickly as possible. (D) Yes, because officers would have inevitably discovered the evidence when they searched the suspect's office pursuant to the search warrant.

(D) Yes, because officers would have inevitably discovered the evidence when they searched the suspect's office pursuant to the search warrant. In general, the evidence that the officer illegally obtained in violation of the defendant's Fourth Amendment rights would be excluded under the exclusionary rule. However, there are several exceptions to the exclusionary rule. One such exception exists where the evidence in question would have been inevitably discovered regardless of the illegal discovery. In this case, police had already obtained a search warrant for the man's home and office, and they were in the process of executing that search warrant. They simply hadn't gotten to the office yet. Therefore, since they would have inevitably discovered the computer document using the search warrant, it is admissible, even though it was illegally discovered by the rookie police officer. Thus, this answer choice is correct.

After a nine-year-old girl was kidnapped while walking home from school, police and community members began to search the neighborhood. While the search for the girl was ongoing, police also launched an extensive investigation of the kidnapping in an effort to identify a suspect. After several days, police identified a neighbor as a suspect in her kidnapping and brought him into the station for questioning. A rookie police officer had heard that after 48 hours the chance of rescuing a kidnapped child alive decreased dramatically. Since more than 48 hours had passed since the girl was kidnapped, the officer became desperate. He asked around until he learned the suspect's identity and took it upon himself to break into the suspect's office and search the files on his computer for evidence. The officer found a document on the computer which appeared to be a detailed description of the kidnapping, so he printed the document and returned to police headquarters. At the same time, pursuant to a search warrant, other officers on the case were conducting a systematic search of the suspect's property beginning with his house and then his office. Will the computer file be admissible as evidence in the suspect's subsequent criminal trial for the kidnapping? (A) No, because it is "fruit of the poisonous tree" and therefore tainted. (B) No, because the officers who had the search warrant were not the ones who discovered the document. (C) Yes, because there was an urgent need to search the suspect's office in order to locate the girl as quickly as possible. (D) Yes, because officers would have inevitably discovered the evidence when they searched the suspect's office pursuant to the search warrant.

(D) Yes, because officers would have inevitably discovered the evidence when they searched the suspect's office pursuant to the search warrant. In general, the evidence that the rookie officer illegally obtained in violation of the defendant's Fourth Amendment rights would be excluded under the exclusionary rule. However, there are several exceptions to the exclusionary rule. One such exception exists where the evidence in question would have been inevitably discovered regardless of the illegal discovery. In this case, police had already obtained a search warrant for the suspect's home and office, and they were in the process of executing that search warrant. They simply hadn't gotten to the office yet. Therefore, since they would have inevitably discovered the computer document using the search warrant, it is admissible, even though it was illegally discovered by the rookie police officer. Thus, this answer choice is correct.

The plaintiff, a man from State A, was injured when the defendant, a citizen of State B, ran a red light in State A, causing $60,000 in damages. After a pretrial conference, the plaintiff and defendant were arguing in the parking lot over the cause of the accident. The defendant pushed the plaintiff to the ground and stormed off. The plaintiff sought medical attention after the incident. The plaintiff believed that he suffered $20,000 in damages from the incident in the parking lot. The plaintiff ultimately filed suit against the defendant for negligence and battery in a State A federal court. The defendant objected, claiming that the federal court did not have jurisdiction to hear the cases. May the federal court hear the case? (A) No, because even though the plaintiff's damages arose out of the same case or controversy, both claims arose from violations of state law. (B) No, because the plaintiff cannot aggregate unrelated claims. (C) Yes, because the court has supplemental jurisdiction over the battery action. (D) Yes, because the elements for diversity jurisdiction are met.

(D) Yes, because the elements for diversity jurisdiction are met. The plaintiff can bring his case in federal court if there is a federal question or diversity of citizenship. Here, there is no federal question involved. However, the plaintiff and the defendant are from different states, and the amount in controversy exceeds $75,000. The amount in controversy can be aggregated even if the claims are unrelated, so long as they are asserted by the same plaintiff against the same defendant. Here, both claims are brought by the same plaintiff against the same defendant. Therefore, aggregation is proper.

The plaintiff owned a very successful seafood restaurant. At the end of the previous season, the plaintiff became concerned about competition from a new restaurant approximately two miles away, and so he decided to hire a quality-control administrator who would help increase customer satisfaction and profitability. As one of his first new directives, the quality-control administrator created a spreadsheet and instructed the staff to keep tallies of any service-related issues. He had three headings on the spreadsheet: (1) food sent back based on quality; (2) complaints about service in general; and (3) comments about price. About three months later, the plaintiff learned that his seafood supplier was being paid by the competitor to deliver subpar seafood to the plaintiff and keep the best product for the competitor. The plaintiff then brought suit against the competitor for intentional interference with a business relationship. In order to prove damages, the plaintiff called the quality-control administrator to testify as to the column on the spreadsheet that showed the number of meals that were sent back due to food quality. Is the quality-control administrator allowed to testify as to the spreadsheet? (A) No, because the purpose of keeping the record was not sufficiently business-related. (B) No, because the spreadsheet constitutes hearsay, not within any exception. (C) Yes, because the spreadsheet constitutes a past recollection recorded. (D) Yes, because the spreadsheet is a business record.

(D) Yes, because the spreadsheet is a business record. Hearsay is an out-of-court statement offered into evidence to prove the truth of the matter asserted. Here, the tallies are clearly written testimony being presented for the purpose of proving that the supplier was supplying the plaintiff's restaurant with inferior seafood. Therefore, the spreadsheet is inadmissible hearsay unless it falls within a recognized hearsay exception. Because the spreadsheet was created for the purpose of improving the plaintiff's business and not to further his litigation, it was made in the ordinary course of the plaintiff's business. As such, it comes within the business records exception to the hearsay rule.

A man crashed into a woman's car. Shortly after the accident, a police officer arrived at the scene, and the woman told the officer that the man had run the red light and swerved into her car. The police officer wrote her statement in the "Accident Report Form," which he was required to complete after every accident. The woman sued for personal injuries sustained in the accident. On direct examination, her attorney called her to testify and asked her to recall the events of the night. Is the woman's testimony admissible? (A) No, because a party is precluded from introducing their own out-of-court statements at a subsequent trial. (B) No, because it is hearsay not within any exception. (C) Yes, because the officer wrote down the statement as part of his official duties. (D) Yes, because the woman has firsthand knowledge of the events of the night in question.

(D) Yes, because the woman has firsthand knowledge of the events of the night in question. The woman is not being asked to recount her out-of-court statement, but rather, to testify as to what she remembered seeing on the night in question. As long as the witness has firsthand knowledge, she is a proper witness to the events, and can testify as to what she saw. Therefore, there is no hearsay problem.

A city with a majority nonwhite population has historically had a nonwhite police commissioner. After a recent election, the new city mayor appointed the first white police commissioner in decades. Two months into the job, the new commissioner instated a "qualification exam" requirement for all current police officers and those seeking to become officers. The stated purpose of this exam was to "ensure that only the most qualified people are upholding the laws of this city and state." Current officers who failed the exam would have their employment terminated, and new applicants would need to pass the exam before being considered to join the police academy. For those who failed the exam, an appeals process was also put in place, allowing the applicant to argue that they were qualified for the position despite failing the written exam. In the wake of the new requirements, 80% of the nonwhite police officers failed the exam and were fired, while only 20% of the white officers failed. Of those who appealed, over 75% of white officers had their appeals granted, while only 10% of nonwhite officers had theirs granted. After six months, it was found that white applicants passed the test 68% of the time, while only 12% of nonwhite applicants passed. Half of the white applicants who appealed were admitted to the police academy, while no nonwhite applicant's appeal was granted. Is the city's qualification exam requirement constitutional? (A) No, because tests that disadvantage racial minorities receive strict scrutiny and are presumptively unconstitutional. (B) No, because the test has a disparate impact on the success of racial minority candidates. (C) Yes, because a state's police powers includes the absolute right to prescribe qualifications for city police officers. (D) Yes, because there is no evidence of intentional race discrimination in the administration of the test.

(D) Yes, because there is no evidence of intentional race discrimination in the administration of the test. In Washington v. Davis [426 U.S. 229 (1976)], the Supreme Court held that race-neutral qualifications for state offices did not trigger heightened (i.e., intermediate or strict) scrutiny in the absence of evidence of a discriminatory purpose. In Davis and other cases, the Court has made clear that a disproportionate impact on distinct racial groups, standing alone, is insufficient to show intentional discrimination and trigger heightened scrutiny. However, when a facially neutral law involves discretionary decisions, and those decisions are made in a discriminatory manner, the Supreme Court has found that there is purposeful discrimination in the application of the law [Yick Wo v. Hopkins, 118 U.S. 356 (1886)].

A customer has banked with the same area bank for the last 10 years. On several occasions, the bank merged with or was bought out by a larger national bank, but in each instance, the customer was able to iron out the ensuing logistical difficulties and maintain his fee-free banking services, in large part because of a very large account he has at the bank. Now, the bank has merged again. Two days after the merger, the customer receives several statements documenting substantial fees. The customer is enraged. He contacts the newly appointed bank manager, who indicates that that she is not willing to help the customer oppose the fees. The customer requests the name of her supervisor, and the manager tells him that in fact, she herself is the regional manager and oversees the entire state. The manager then hangs up on the customer. Infuriated, the customer walks over to the bank, enters, and walks straight to the manager's office. He fires a single shot at the woman standing in the office, killing her. The woman in the office was not the manager but another customer waiting for the manager to return from the vault. The customer is facing charges for the shooting death. Of which of the following crimes is the defendant guilty? (A) First-degree murder. (B) Second-degree murder. (C) Voluntary manslaughter. (D) Involuntary manslaughter

A) First-degree murder. First-degree murder includes intent-to-kill murder committed with premeditation and deliberation, felony murder, and, in some jurisdictions, murder accomplished by lying in wait, poison, or torture. If a murderer engages in any reflection or premeditation prior to the homicide, even if the reflection is cursory and brief, he may be guilty of first-degree murder. Here, although the person killed was not the defendant's intended victim, the defendant's factual mistake as to the identity of the person in the manager's office does not relieve him of responsibility for his purposeful act of killing. Rather, the defendant's intent to kill the manager is transferred to the person killed. Because the defendant acted with premeditation and deliberation, first-degree murder is the crime of which the defendant is guilty.

A man lived next door to a woman. Late one night, the man overheard the woman scream "Ellen, put down the knife, please don't do it!" The next day, the woman was found dead in her bedroom. Ellen was arrested and charged with murdering the woman. At trial, the man proposed to testify to the woman's statement. Upon objection by the defendant's counsel, how should the court rule on the man's testimony regarding the woman's statement? (A) Admissible as a dying declaration, if the jury determines that the woman believed that her death was imminent. (B) Admissible as a dying declaration, if the judge, by a preponderance of the evidence, determines that the woman believed that her death was imminent. (C) Inadmissible, because the probative value is substantially outweighed by the danger of unfair prejudice. (D) Inadmissible, as hearsay not within any recognized exception.

B) Admissible as a dying declaration, if the judge, by a preponderance of the evidence, determines that the woman believed that her death was imminent. Under FRE 104(a), "Preliminary questions concerning the qualification of a person to be a witness, the existence of a privilege, or the admissibility of evidence shall be determined by the court." The judge is not bound by the rules of evidence at this point, except those with respect to privileges. To the extent that these inquiries are factual, the judge acts as a trier of fact. The judge's decision is final and is not subject to a contrary determination by the jury. On the other hand, the jury is to determine how much, if any, probative value or "weight" to accord to the admitted evidence, as well as to decide issues of credibility (i.e., whether to believe, wholly or in part, the witness's testimony). The judge not only decides factual issues, he also determines the applicability of any technical evidentiary rules. He decides, for example, if a dying declarant had a sense of impending death; if an entry was made promptly in the regular course of business; if there was the necessary state of excitement to qualify a declaration as an excited utterance; if a witness is unavailable; or if an original document is unavailable so as to justify the admission of a copy under the best evidence rule. Finally, note that preliminary questions of fact are ordinarily resolved in both criminal and civil trials by using a preponderance of the evidence standard. The witness's testimony as to the victim's statement will be admissible as a dying declaration if the judge, by a preponderance of the evidence, determines the victim believed her death was imminent. Thus, this answer choice is correct.

A plaintiff was seriously injured in a collision involving three trucks. A sideswipe occurred between the plaintiff's pickup truck and a trailer truck as they passed in opposite directions. The sideswipe caused the plaintiff's truck to careen down the road and into the path of a second trailer truck. The driver of the second trailer truck was immediately killed. Both of the trailer trucks were owned by the same transport company. At trial, the plaintiff called a witness to the collision to testify that he heard an unidentified female witness scream, "Oh my God, that trailer truck sideswiped that little pickup truck." How should the trial judge rule regarding the witness's testimony? (B) Admissible as an excited utterance. (C) Inadmissible, because it contains inadmissible opinion evidence. (D) Inadmissible, as hearsay not within any recognized exception.

B) Admissible, as an excited utterance. Under FRE 803(2), the statement of the unidentified bystander, made in the presence of the witness, should qualify as an excited utterance or spontaneous exclamation within the meaning of that exception to the hearsay rule. The theory of this exception is simply that there has been some occurrence startling enough that temporarily stills the capacity of reflection and produces utterances free of conscious fabrication.

While walking down an escalator in a department store, a man was hit from behind by an employee who was taking some boxes to the trash compactor. The man fell and suffered injuries as a result. While in the hospital, the doctor indicated that the accident itself should only have caused minor injuries. The doctor then determined the man's bones had never healed properly after falling out of a tree as a child and breaking his pelvis and both of his legs. As a result, the injuries the man now suffered were more severe. At trial, the jury made the following findings: (1) the man was 10% at fault; (2) the employee was gainfully employed by the department store at the time of the injury; and (3) the man's childhood injuries were self-inflicted. To what damages will the man be entitled? (A) He will be entitled to only those damages that can be attributed to the employee's accident, minus his own percentage of negligence. (B) He will be entitled to all damages sustained as a result of the accident because the department store is liable for the full consequences of the injuries sustained by the man. (C) He will be entitled to damages that a normal person would have sustained in a similar accident. (D) He will recover nothing because his childhood injuries were self-inflicted.

B) He will be entitled to all damages sustained as a result of the accident, because the department store is liable for the full consequences of the injuries sustained by the man. Under the so-called thin-skulled or eggshell plaintiff rule, a defendant is liable for the full consequences of a plaintiff's injury, even though, due to the plaintiff's peculiar susceptibility to harm (of which the defendant was unaware), those consequences were more severe than they would have been in a normal person. Here, the employee was negligent, so the question then goes to damages. Applying the eggshell plaintiff rule, the defendant must take the plaintiff as he finds him. The man's childhood condition caused him to suffer more severe injuries than a normal person would have incurred. Even though the employee and employer were unaware of the man's condition, they will be liable for the full extent of the injuries sustained.

A man crashed into a woman's car. Shortly after the accident, a police officer arrived at the scene, and the woman told the officer that the man had run the red light and swerved into her car. The police officer wrote her statement in the "Accident Report Form," which he was required to complete after every accident. The woman sued for personal injuries sustained in the accident. On direct examination, her attorney calls her to testify as to her statement to the police officer. Is the woman's statement admissible? (A) No, because a party is precluded from introducing their out-of-court statements at a subsequent trial. (B) No, because it is hearsay not within any exception. The woman's statement was made out of court and is now being offered for the truth of the matter asserted therein (i.e., that the man had run a red light and swerved into her car). The statement is therefore hearsay not within any applicable exception. Thus, the statement is inadmissible. (C) Yes, because it constitutes a prior consistent statement. (D) Yes, because the woman is in court testifying to an out-of-court statement, and is therefore subject to cross-examination.

B) No, because it is hearsay not within any exception. The woman's statement was made out of court and is now being offered for the truth of the matter asserted therein (i.e., that the man had run a red light and swerved into her car). The statement is therefore hearsay not within any applicable exception. Thus, the statement is inadmissible.

Blue, an independent taxi driver, took his car to Aamco Transmissions to have the transmission repaired. Owens, Aamco's employee, was working on the transmission when the hydraulic lift broke. This caused the car to plummet on top of Owens, injuring him. Blue, who happened to be standing nearby, saw the incident and rushed over to help Owens. As Blue tried to free Owens from the metal entanglement, the hydraulic pump (an apparatus connected to the lift) suddenly burst, propelling a sharp metallic fragment into Blue's stomach. Blue received a deep laceration wound but was not otherwise injured. Blue then initiated suit against Aamco based on negligent maintenance of the hydraulic lift. At trial, Blue called the chief inspector for the state Department of Occupation Safety and Health Administration (OSHA), who testified about an official report of the accident as prepared by an OSHA investigator. Blue proposes to introduce a segment of the report that quotes Owens' statement, made to the OSHA investigator in an interview at Aamco's shop, as to the cause of the accident. Should the trial judge admit the report over Aamco's objection? (A) Yes, as a past recollection recorded. (B) Yes, as an opposing party's statement incorporated in a public record. (C) No, because such documents must be offered in their entirety. (D) No, because it is hearsay not within any exception.

B) Yes, as an opposing party's statement incorporated in a public record. A vicarious statement is a statement by an employee or agent made within the scope of the employment relationship and which is now being offered against the employer. The facts state that Owens, Aamco's employee, made a statement to the chief inspector for OSHA. That statement was contained within an OSHA accident report. Because the OSHA accident report qualifies as a public record, this is the correct answer.

A truck driver for a local beer distributor was pulling onto the freeway when he collided with a car driven by a bank teller. The bank teller and her passengers were seriously injured in the accident, and they filed suit against the truck driver, alleging that he failed to yield as he entered the freeway from the entrance ramp. They also alleged that he had exceeded by five hours the company's 12-hour limit for drivers per day, and that he should not have been driving at all, as he was taking medication that caused dizziness and fatigue. The truck driver, in turn, claimed that the bank teller was speeding and had failed to make any attempt to avoid the accident even though the two lanes to the left of the entrance ramp were wide open. The truck driver's attorney seeks to introduce the testimony of the state trooper who first arrived on the scene that, when he approached the bank teller's car immediately following the accident, she stated over and over, "I should have slowed down or moved over." The bank teller objects. Is the trooper's statement admissible? (A) Yes, because it was overheard by an officer during his investigation. (B) Yes, because it is a statement by the bank teller. (C) No, because it does not admit any wrongdoing that is clearly a violation of the motor vehicle law. (D) No, because it is hearsay.

B) Yes, because it is a statement by the bank teller. Under Federal Rule of Evidence 801, a statement by a party-opponent that is offered against that party is not hearsay. Here, the bank teller's statement that she should have slowed down or moved over constitutes an opposing party's statement, and will therefore be admissible against her at trial.


Conjuntos de estudio relacionados

Topic Review 4 Multiple choice Fnce 125

View Set

Biochem Chapter 7 Study Questions

View Set

Module 1: Role of human resources

View Set

AJ2, Kormányzati rendszerek, Országgyűlés (5.téma)

View Set

Week 5: Chapter 48 Drugs Affecting Blood Coagulation

View Set

Alternative measures of labor underutilization

View Set

Ch. 16 Control of Gene Expression

View Set

MS3 - Ch. 38: Rheumatic Disorders

View Set

Chapter 4a Exam: Life Insurance - Types of Policies

View Set